Tax Final

Lakukan tugas rumah & ujian kamu dengan baik sekarang menggunakan Quizwiz!

Marshall sells his entire interest in a passive activity for $350,000 on the installment method. His adjusted basis in the property is $280,000. The activity has a suspended loss of $40,000. In your computations, round any division to three decimal places. If Marshall receives a $80,000 down payment, he will recognize a gain of $___ and Marshall will deduct $___ of the suspended losses in the first year of the sale.

$16,000 [(350,000-280,000)/350,000] = 20% x 80,000 = $16,000 $9,160 = (16,000/70,000) = .229 * 40,000 = 9,160

In 2018, Pedro receives a gift of real estate with an adjusted basis of $900,000 and a fair market value of $890,000. No gift tax was paid on the transfer. Pedro's basis for a gain is $___ and his basis for a loss is $___. If Pedro later sells the property for $885,000, his recognized ___ is $___.

$900,000, $890,000, loss, $5,000

Aiden performs services for Lucas. Which, if any, of the following factors indicate that Aiden is an employee, rather than an independent contractor?

Aiden is paid based on hours worked.

Late in the tax year, the Polks come to you for tax advice. They are considering selling some stock investments for a loss and making a contribution to a traditional IRA. In reviewing their situation, you note that they have large medical expenses and a casualty loss (in a Federal disaster area), with neither being covered by insurance. What are the tax implications of selling the stock and making a contribution to a traditional IRA? The loss on the sale of stock will: The contribution to a traditional IRA will:

Both: Decrease AGI, which in turn will increase the deductions for the medical expenses and casualty loss.

Oriole Corporation has active income of $45,000 and a passive activity loss of $23,000 in the current year. Under an exception, Oriole can deduct the $23,000 loss if it is a personal service corporation.

False

Personal expenditures that are deductible as itemized deductions include medical expenses, Federal income taxes, state income taxes, property taxes on a personal residence, mortgage interest, and charitable contributions.

False

The amount of loss for partial destruction of business property is the decline in fair market value of the business property.

False

There is a Federal excise tax on hotel occupancy.

False

If depreciation is claimed, it should be supported by completing ___

Form 4562

In 2018, Mary had the following items: Salary $30,000 Personal use casualty gain 10,000 Personal use casualty loss (after $100 floor) 17,000 Other itemized deductions 4,000 Assuming that Mary files as head of household (has one dependent child), determine her taxable income for 2018.

Salary $30,000 Personal use casualty gains in excess of personal use casualty losses ($10,000 - $10,000) $0 Adjusted gross income $30,000 Less: Deductions Itemized deductions Casualty loss (not allowed; not in Federally-declared disaster area) $0 Other itemized deductions 4,000 Total itemized deductions $4,000 Standard deduction (larger than itemized deductions) (18,000) Taxable income $12,000

Answer the following regarding the gift tax exclusion. a. Patel is single and gives each of his six grandchildren $30,000 this year (2018). Patel's gift tax exclusion is ___. b. Assume Patel is married and elects to gift-splitting with his wife. Patel's gift tax exclusion is ___

a. $90,000 = (6 x $15,000) b. $180,000 = [(6 x 2) x $15,000]

There is an important distinction between tax credits and tax deductions. ___ reduce taxable income on which the tax liability is based. ___ reduce the tax liability dollar for dollar.

Tax deductions, Tax credits

A mother sells a valuable collection of antiques to her daughter for $1,000. Select from the dropdown list the judicial concept the IRS might invoke to question this transaction.

The arm's length concept

Indicate whether the following are the canons of taxation identified by Adam Smith. Select "Yes" or "No", whichever is applicable. a. Equity b. Neutrality c. Convenience d. Certainty e. Economy

a. Yes b. No c. Yes d. Yes e. Yes

In the following independent situations, select either "Yes" or "No" to indicate whether the taxpayer's tax position is likely to change. a. John started renting out a spare room in his home. b. Theresa quit her job as a staff accountant and has established her own practice as a CPA. c. Paul's employer transferred him from its California office to an office in Florida.

a. Yes b. Yes c. Yes

Marvin is the executor and sole heir of his aunt's estate. The estate includes her furnished home, which Marvin is considering converting to rental property to generate additional cash flow. For each item below, select either "Yes" or "No" to identify those which could be a tax problem for Marvin. a. Any rent Marvin receives from the property is taxed as income. b. Besides the real estate taxes, personal property taxes could be imposed on the furnishings. c. Marvin can expect an increase in the ad valorem property taxes levied by the local taxing authorities due to the commercial use of the property. d. Since the property was inherited, Marvin may not convert it to rental property.

a. Yes b. Yes c. Yes d. No

"Bracket creep" will not exist if there is only a single (flat) tax rate for the income tax.

True

As opposed to itemizing deductions from AGI, the majority of individual taxpayers choose the standard deduction.

True

In connection with the application of the kiddie tax, complete the following statements. a. The child generates only earned income. The kiddie tax ___ apply to earned income. b. The child reports a modest amount of unearned income. For 2018, the kiddie tax does not apply unless unearned income exceeds ___ c. The child is age 20, not a student, and not disabled. For the kiddie tax, the age coverage for a child is under __ or a full-time student under age __. d. The child is married. The kiddie tax ___ __ apply if the child is married and files a joint return.

a. does not b. $2,100 c. 19, 24 d. does not

The TJCA of 2017 prohibits employers from deducting qualified transportation fringe benefits provided to employees. If the employer provides the benefit, though, the employee may exclude it from income within the limits stated above.

True

The tax benefit received from a tax credit is never affected by the tax rate of the taxpayer.

True

When determining whether an individual is a material participant, participation by an owner's spouse generally counts.

True

Answer the following regarding the bad debt deduction. a. When can a bad debt deduction be taken for a nonbusiness debt? b. How is a nonbusiness bad debt classified?

a. In the year of total worthlessness b. Short-term capital loss

As a general rule, a deduction for personal expenditures is ___ by § 262 of the Code.

disallowed

Commercial Bank has initiated an advertising campaign that encourages customers to take out home equity loans to pay for purchases of automobiles. Are there any tax advantages related to this type of borrowing? The TCJA of 2017 ___ the itemized deduction for interest paid on home equity loans from 2018 through 2025. Assuming that the car is used only for personal purposes, the interest on a loan to buy it is ___.

eliminated, not deductible

An individual taxpayer sells some used assets at a garage sale. Why are none of the proceeds taxable in most situations? These assets are ___ assets. Losses on these assets ___ deductible, and the proceeds of the sale are ___; thus, the transactions are not taxable.

personal-use, are not, a return of capital

On December 1, 2018, Bigham Corporation pays a dividend of $4.00 on each share of its common stock. Vanessa and Gena, two unrelated shareholders, each own 5,000 shares of the stock. Vanessa has owned her stock for two years while Gena purchased her stock on November 3, 2018. How does each shareholder treat the $20,000 dividend from Bigham? The $20,000 that Vanessa receives is ___ and the $20,000 that Gena receives is ___.

subject to preferential 0%/15%/20% treatment, taxed at ordinary income rates.

Wolf Corporation has active income of $55,000 and a passive activity loss of $33,000 in the current year. Wolf cannot deduct the $33,000 loss if it is a closely held C corporation that is not a personal service corporation.

False

Complete the tax formula below. Tax Formula for Individuals

Income (broadly conceived) Less: Exclusions Equals: Gross income Less: Deductions for AGI Equals: Adjusted gross income Less: The greater of the standard deduction or itemized deductions Equals: Taxable income

Santiago and Amy are married and file a joint tax return claiming their three children, ages 12, 14, and 18, as dependents. Their AGI is $140,000. Santiago and Amy's child tax credit is $___

$4,000

A taxpayer can carry back any NOL incurred 2 years and then forward up to 20 years.

False

All income must be reported, and all deductions are allowed unless specifically disallowed in the Code.

False

All of a taxpayer's tax credits relating to a passive activity can be utilized when the activity is sold at a loss.

False

Once the actual cost method is used, a taxpayer cannot change to the automatic mileage method in a later year.

False

Only self-employed individuals are required to make estimated tax payments.

False

The principal objective of the FUTA tax is to provide some measure of retirement security.

False

The tax benefits resulting from tax credits and tax deductions are affected by the tax rate bracket of the taxpayer.

False

Alfred owned a term life insurance policy at the time he was diagnosed as having a terminal illness. After paying $18,300 in premiums, he sold the policy to a company that is authorized by the state of South Carolina to purchase such policies. The company paid Alfred $125,000. When Alfred died 18 months later, the company collected the face amount of the policy, $150,000. Alfred is required to include ___ in his gross income as a result of the sale of the policy.

$0

In the current year, Wells paid the following expenses: Premiums on an insurance policy against loss of earnings due to sickness or accident $3,000 Physical therapy after spinal surgery $2,000 Premium on an insurance policy that covers reimbursement for the cost of prescription drugs $500 In the current year, Wells recovered $1,500 of the $2,000 that she paid for physical therapy through insurance reimbursement from a group medical policy paid for by her employer. Disregarding the adjusted gross income percentage threshold, what amount could be claimed on Wells's current-year income tax return for medical expenses?

$1,000 [(2,000 - 1,500) + 500]

Research and experimental expenditures do not include the cost of consumer surveys.

True

If the amount of the insurance recovery for a theft of business property is greater than the asset's fair market value but less than it's adjusted basis, a gain is recognized.

False

A cash basis taxpayer must include as income the proceeds from the sale of an account receivable to a collection agency.

True

An education expense deduction may be allowed even if the education results in a promotion or pay raise for the employee.

True

Ashley received a scholarship to be used as follows: tuition $6,000; room and board $9,000; and books and laboratory supplies $2,000. Ashley is required to include only $9,000 in her gross income.

True

Sheila's appreciated property is involuntarily converted. She receives insurance proceeds equal to the fair market value of the property. Complete the statement below regarding the minimum amount Sheila must reinvest in qualifying property to defer recognition of realized gain. Sheila must reinvest an amount equal to the ___ realized.

net insurance proceeds

Coline has the following capital gain and loss transactions for 2018. a. Short-term capital gain $5,000 b. Short-term capital loss (2,100) c. Long-term capital gain (28%) 6,000 d. Long-term capital gain (15%) 2,000 e. Long-term capital loss (28%) (10,500) After the netting process, Coline has an overall ___ of $___.

net short-term capital gain, $400 (page 14-20)

Complete the following statements regarding business and nonbusiness bad debts. Lulu is a calendar year taxpayer. She owned stock in Kasper Corporation (a publicly held company). Lulu acquired the stock as an investment on November 1, 2017, at a cost of $23,000. On June 30, 2018, the stock became worthless when the company declared bankruptcy. Because the stock is deemed to have become worthless as of ___, Lulu has ___.

December 31, 2018, a long-term capital loss

Ninette, age 16, has dividend income of $2,800 and interest income of $935. She has no investment expenses. Determine the net unearned income for the purpose of the kiddie tax. Net unearned income is

$1,635 ($2,800 + $935) - ($1,050 less a $1,050 standard deduction)

Elijah and Anastasia are husband and wife who have five married children and nine minor grandchildren. For 2018, what is the maximum amount they can give to their family (including the sons- and daughters-in-law) without using any of their unified transfer tax credit?

$570,000 [(2 x 5) + 9] x $15,000 x 2 people

Elijah and Anastasia are husband and wife who have three married children and fourteen minor grandchildren. For 2018, what is the maximum amount they can give to their family (including the sons- and daughters-in-law) without using any of their unified transfer tax credit?

$600,000 = [(2 x 3) + 14] x $15,000 limit * 2 people

Adonías and Cierra are husband and wife and have five married children and nine minor grandchildren. In addition, Cierra has two sisters. For tax year 2018, what is the maximum amount they can give to the family (including sons- and daughters-in-law and the two sisters) without using any of their unified transfer tax credit?

$630,000 (5 married children + 5 spouses + 9 grandchildren + 2 sisters) x $15,000 (annual exclusion for 2018) x 2 donors (Adonías and Cierra, gift splitting) = $630,000

Complete the following statements regarding the legislative process. Federal tax legislation generally originates in the ___, where it is first considered by the ___. Tax bills originate in the ___, when they are attached as riders to other legislative proposals.

House of Representatives, House Ways and Means Committee, Senate

Answer the following question regarding Federal tax legislation. Where does Federal tax legislation generally originate?

The House Ways and Means Committee

Social considerations can be used to justify: a. Allowing excess capital losses to be carried over to other years. b. Allowance of a credit for child care expenses. c. Allowing a Federal income tax deduction for state and local sales taxes. d. Allowing accelerated amortization for the cost of installing pollution control facilities. e. None of these choices are correct.

b. Allowance of a credit for child care expenses.

State income taxes generally can be characterized by: a. No provision for withholding procedures. b. The same date for filing as the Federal income tax. c. Applying only to individuals and not applying to corporations. d. Allowance of a deduction for Federal income taxes paid. e. None of these choices are correct.

b. The same date for filing as the Federal income tax.

Tax functions that accounting and finance professionals may assist clients with include all but the following: a. Tax planning. b. Cash management to ensure timely payment of taxes. c. Tax evasion. d. Tax compliance.

c. Tax evasion.

In terms of probability, which of the following taxpayers would be least likely to be audited by the IRS? a. Taxpayer just received a $3 million personal injury award as a result of a lawsuit. b. Taxpayer has been audited several times before. c. Taxpayer is an employed electrician. d. Taxpayer owns and operates a check-cashing service. e. Taxpayer just won a $1 million slot machine jackpot at a Las Vegas casino.

c. Taxpayer is an employed electrician.

A landlord leases property upon which the tenant makes improvements. The improvements are significant and are not made in lieu of rent. At the end of the lease, the value of the improvements are not income to the landlord. This rule is an example of: a. The tax benefit rule. b. The arm's length concept. c. The wherewithal to pay concept. d. A clear reflection of income result. e. None of these choices are correct.

c. The wherewithal to pay concept.

When would an account receivable give rise to a bad debt deduction?

When the account receivable was previously included in gross income.

How are gains from the sale of § 1244 stock treated? Gains on the sale of § 1244 stock are treated as ___.

a capital gain

For self-employed taxpayers, travel expenses are deductions for AGI.

True

A father cannot claim a loss on his daughter's rental use property.

True

What is the difference between excise tax, sales tax and use tax? __ ______tax is restricted to a particular transaction while _ _____ tax covers a multitude of transactions. _ ___ tax is an ad valorem tax on the use, consumption, or storage of tangible property purchased outside the state but used in the state.

An excise, a sales, A use

Heather's interest and gains on investments for the current year are as follows: Interest on Madison County school bonds $600 Interest on U.S. government bonds 700 Interest on a Federal income tax refund 200 Gain on sale of Madison County bonds 500 Heather's adjusted gross income from the above is: or; Heather must report gross income in the amount of:

$1,400 (700 + 200 + 500)

Rebecca sells her personal scooter for $550. She purchased the scooter for $700 three years ago. In addition, Rebecca sells a painting for $1,200 that she acquired five years ago for $900. Rebecca has a ___ realized ___ on the scooter and a ___ realized ___ on the painting. Rebecca will recognize the gain or loss associated with ___

$150, loss, $300, gain, only the painting

Jarrod receives a scholarship of $35,200 from Riggers University to be used to pursue a bachelor's degree. He spends $21,120 on tuition, $1,760 on books and supplies, $7,040 for room and board, and $5,280 for personal expenses. Jarrod may exclude $___ from his gross income.

$22,880

Ted works for Azure Motors, an automobile dealership. All employees can buy a car at the company's cost plus 2%. The company does not charge employees the $300 dealer preparation fee that nonemployees must pay. Ted purchased an automobile for $29,580 ($29,000 + $580). The company's cost was $29,000. The price for a nonemployee would have been $33,900 ($33,600 + $300 preparation fee). What is Ted's gross income, if any, from the purchase of the automobile?

$240 ($300 * .8)

Mike, an attorney, earns $200,000 from his law practice and receives $45,000 in dividends and interest during the year. In addition, he incurs a loss of $50,000 from an investment in a passive activity acquired three years ago. What is Mike's net income for the current year after considering the passive investment? $___

$245,000

Complete the statements below regarding the real estate exception. The passive loss limits contain an exception related to real estate activities that is not restricted to real estate professionals. This exception allows individuals to deduct up to $___of losses from real estate rental activities against active and portfolio income. The potential annual deduction is reduced by ___ percent of the taxpayer's AGI in excess of $___. To qualify for the exception, a taxpayer must meet ___ of the following requirements: ___ participate in the real estate rental activity. Own ___ percent or more (in value) of all interests in the activity during the entire taxable year (or shorter period during which the taxpayer held an interest in the activity).

$25,000, 50%, $100,000, both, actively, 10%

Matt, a calendar year taxpayer, pays $11,000 in medical expenses in 2018. He expects $5,000 of these expenses to be reimbursed by an insurance company in 2019. In determining his medical expense deduction for 2018, Matt must reduce his 2018 medical expenses by the amount of the reimbursement he expects in 2019.

False

Monique owns a building that she leases to an individual who operates a grocery store. Rent income is $10,000, and rental expenses are $6,000. On what Form 1040 schedule or schedules are the income and expenses reported?

Schedule E

Milton is a resident of Mobile (Alabama) and is employed by Scaup Corporation. Because Scaup closed its Mobile office, Milton no longer has any nondeductible commuting expenses although he continues to work for Scaup. The most likely reason for this change is:

The taxpayer has an office in the home that qualifies as a principal place of business.

Which of the following would preclude a taxpayer from deducting student loan interest expense?

The taxpayer is taken as a dependent of another taxpayer.

A taxpayer may not deduct the cost of new curbing (relative to a personal residence), even if the construction is required by the city and the curbing provides an incidental benefit to the public welfare.

True

A taxpayer who always claims the standard deduction (i.e., does not itemize his or her deductions from AGI) may still be able to receive a tax benefit from any education expenses incurred.

True

A taxpayer who claims the standard deduction may be able to claim an office in the home deduction.

True

A taxpayer who itemizes must use Form 1040, and cannot use Form 1040EZ or Form 1040A.

True

A taxpayer who uses the automatic mileage method for the business use of an automobile can change to the actual cost method in a later year.

True

A theft of investment property can create or increase a net operating loss for an individual.

True

After Ellie moves out of the apartment she had rented as her personal residence, she recovers her damage deposit of $1,000. The $1,000 is not income to Ellie.

True

All personal property placed in service in 2018 and used in a trade or business qualifies for additional first-year depreciation.

True

Benefits collected under an accident and health insurance policy purchased by the taxpayer are excludible even though payments are a substitute for income.

True

Bill paid $2,500 of medical expenses for his daughter, Marie. Marie is married to John and they file a joint return. Bill can include the $2,500 of expenses when calculating his medical expense deduction.

True

In the current year, Kelly had a $35,000 loss from a real estate rental activity in which she is a 10% owner. If she is an active participant and if her modified AGI is $100,000, she can deduct $25,000 of the loss.

True

In the event that overwithholding of FICA tax occurs because the taxpayer has more than one employer, the excess amount should be claimed as a credit on the Federal income tax return of the employee.

True

The fair market value of property received in a sale or other disposition is the price at which property will change hands between a willing seller and a willing buyer when neither is compelled to sell or buy.

True

The key date for calculating cost recovery is the date the asset is placed in service.

True

The objective of pay-as-you-go (paygo) is to improve administrative feasibility.

True

The portion of property tax on a vacation home that is attributable to personal use is an itemized deduction.

True

The tax law specifically provides that a taxpayer cannot be temporarily away from home for any period of employment that exceeds one year.

True

This year Dena traveled 600 miles for specialized medical treatment that was not available in her hometown. She paid $90 for meals during the trip, $145 for a hotel room for one night, and $15 in parking fees. She did not keep records of other out-of-pocket costs for transportation. Dena can include $167 in computing her medical expenses.

True

Tomas participates for 300 hours in Activity A and 250 hours in Activity B, both of which are nonrental businesses. Both activities are active.

True

Under the "twelve month rule" for the current period deduction of prepaid expenses of cash basis taxpayers, the asset must expire or be consumed by the end of the tax year following the year of payment.

True

Upon the recommendation of a physician, Ed has a swimming pool installed at his residence because of a heart condition. If he is allowed to deduct all or part of the cost of the pool, Ed's increase in utility bills due to the operation of the pool qualifies as a medical expense.

True

Tonya is a cash basis taxpayer. In 2018, she paid state income taxes of $8,000. In early 2019, she filed her 2018 state income tax return and received a $900 refund.

If Tonya itemized her deductions in 2018 on her Federal income tax return and her itemized deductions exceeded the standard deduction by more than $900, she must recognize $900 income in 2019 under the tax benefit rule.

Complete the statements below regarding § 179 expense and expense recapture. ___ income recapture is required any time property, on which an expense has been taken under § 179, is no longer used predominantly ___. ___ is required when the expensed property is converted to personal use.

Ordinary, in a trade or business, Recapture

Complete the following regarding the hours a participant must work in a nonrental activity to be guaranteed material participation status. An individual must participate for ___ ___ hours.

more than, 500

Jackson Company incurs a $50,000 loss on a passive activity during the year. The company has active income of $34,000 and portfolio income of $24,000. If Jackson is a personal service corporation, it may deduct $34,000 of the passive activity loss.

False

Jake performs services for Maude. If Maude provides a helper and tools, this is indicative of independent contractor (rather than employee) status.

False

John owns and operates a real estate agency as a sole proprietor. On a full-time basis, he employs his 17-year old daughter as a receptionist and his 22-year old son as a bookkeeper. Both children are subject to FICA withholding.

False

Judy paid $40 for Girl Scout cookies and $40 for Boy Scout popcorn. Judy may claim an $80 charitable contribution deduction.

False

Kelly, who earns a yearly salary of $120,000, sold an activity with a suspended passive activity loss of $44,000. The activity was sold at a loss and Kelly has no other passive activities. The suspended loss is not deductible.

False

Leona borrows $100,000 from First National Bank and uses the proceeds to purchase City of Houston bonds. The interest Leona pays on this loan is deductible as investment interest subject to the investment interest limits.

False

Mallard Corporation pays for a trip to Aruba for its two top salespersons. This expense is subject to the overall limitation (50%).

False

Maria traveled to Rochester, Minnesota, with her son, who had surgery at the Mayo Clinic. Her son stayed at the clinic for the duration of his treatment. She paid airfare of $300 and $50 per night for lodging. The cost of Maria's airfare and lodging cannot be included in determining her medical expense deduction.

False

Marvin lives with his family in Alabama. He has two jobs: one in Alabama and one in North Carolina. Marvin's tax home is where he lives (Alabama).

False

Andre acquired a computer on March 3, 2018, for $2,800. He elects the straight-line method for cost recovery. Andre does not elect immediate expensing under § 179. He does not claim any available additional first-year depreciation. Click here to access the depreciation table to use for this problem. His cost recovery deduction for the computer is $___ for 2018 and $___ for 2019.

$280, $560 For 2018: $2,800 x 10% (Exhibit 8.5) = $280. For 2019: $2,800 x 20% (Exhibit 8.5) = $560.

Oleander Corporation, a calendar year entity, begins business on March 1, 2018. The corporation incurs startup expenditures of $64,000. In your calculations, round any division to 2 decimal places. Round your final answer to the nearest dollar. If Oleander elects § 195 treatment, the total amount of startup expenditures that it may deduct in 2018 is $___.

$3,556 [(64,000/180) * 10 months]

Maple Company purchases new equipment (7-year MACRS property) on January 10, 2018, at a cost of $430,000. Maple also purchases new machines (5-year MACRS property) on July 19, 2018 at a cost of $290,000. Maple wants to maximize its MACRS deductions; assume no taxable income limitations apply. What is Maple's total MACRS deduction for 2018?

$720,000 In 2018, the maximum § 179 deduction is $1,000,000. Additional first-year (bonus) depreciation could also be claimed. As a result, the maximum MACRS deduction for 2018 is $720,000 ($430,000 + $290,000).

During the past tax year, Aleshia identified $15,000 as a nonbusiness bad debt. In that tax year, Aleshia had $100,000 of taxable income, of which $12,000 consisted of short-term capital gains. During the current tax year, Aleshia collected $8,000 of the amount she had previously identified as a bad debt. Determine Aleshia's tax treatment of the $8,000 received in the current tax year. Aleshia includes $___ of the collection as ___.

$8,000, ordinary income

Select the tax that matches each of the following descriptions regarding the nature and purpose of the tax. • A tax is based on the right to do business in the state. • A tax is based on the importation of certain foreign goods. • A tax is applicable to trades or businesses, serving as a license. • A tax is based on the notion of a state's interest in its natural resources. • A tax is imposed on the export of certain commodities.

Franchise taxes Customs duties Occupational fees Severance taxes Export duties

Kurstie received a $800 state income tax refund this year. Kurstie deducted $3,000 of state income taxes paid in the prior year as part of her itemized deductions. Which of the following statements regarding the taxability of Kurstie's refund is true?

If Kurstie's itemized deductions exceeded the standard deduction by $200, then $200 of the refund is included in gross income.

Michael Sima, a sole proprietor craftsman, purchased an amount of equipment in the current year that exceeded the maximum allowable § 179 depreciation election limit by $20,000. Sima's total purchases of property placed in service in the current year did not exceed the limit imposed by § 179. All of the property (including the equipment) was purchased in November of the current year, and Sima elected to depreciate the maximum amount of equipment under § 179. Sima had bottom-line Schedule C income of $50,000 in the current year. Which method may Sima use to depreciate the remaining equipment in the current year?

MACRS mid-quarter convention for personal property.

Celeste makes a gift to Pierre of passive activity property having an adjusted basis of $18,000, suspended losses of $6,000, and a fair market value at the date of the gift of $30,000. Can Celeste deduct the suspended losses in the year of the gift? What is Pierre's basis in the property? $

No, $24,000

Dr. Werner is a full-time professor of accounting at Pelican University. During the year, he teaches continuing education programs for CPA groups in several cities. He also serves as an expert witness in numerous lawsuits involving accounting fraud. Complete the sentences below regarding Dr. Werner's job-related expenses by selecting the correct response from each dropdown list. ___ would likely be considered as Dr. Werner's tax home. Therefore, when he conducts continuing education programs or testifies as an expert witness while out of town, he ___ be allowed a deduction for travel expenses.

Pelican University, would

The deduction for personal and dependency exemptions has been suspended from 2018 through 2025.

True

Indicate the maximum tax rate for the following types of capital gains. a. Short-term gains (held for less than one year) b. Capital gains associated with certain depreciable property used in trade or business (held for more than one year) c. Capital gains associated with collectibles (held for more than one year) d. All other long-term capital gains

a. 37% b. 25% c. 28% d. 20%, 15% or 0%

Indicate whether the following personal expenditures are deductible "For AGI" or "From AGI" (as itemized deductions). a. Interest on student loans b. Contributions to Health Savings Accounts c. Penalty for early withdrawal from savings d. State, local, and foreign taxes on real property e. State income taxes f. Investment interest expense

a. For AGI b. For AGI c. For AGI d. From AGI e. From AGI f. From AGI

Both a value added tax (VAT) and a national sales tax have been criticized as being regressive in their effect. a. Complete the statement below regarding these taxes. Both the national sales tax and the VAT are taxes on ___that would ___ low-income taxpayers. b. How could this shortcoming be remedied? The regressive effect might be partly remedied by ___ on purchases by ___ taxpayers.

a. consumption, burden b. granting a credit or rebate, low-income

Kim owns an interest in an activity that produces $100,000 of income during the year. Complete the statements below regarding whether Kim would prefer to have the activity classified as active or passive. Passive activity income can be offset by ___ . Active income can be offset by ___. Therefore, Kim would prefer to have the activity classified as ___.

active, passive, and portfolio losses only active and portfolio losses passive

The Morgan family lives in Massachusetts. They moor their sailboat in Rhode Island. Complete the statement below which might be a plausible reason for this? The Morgan's are trying to avoid __ _______ ___ __ __________.

ad valorem tax on personalty

Sophia and Jacob are married and file a joint return. Their 2017 tax return included a Form 2106 (Employee Business Expenses) for each of them. Their 2018 tax return, however, included a Schedule C for Sophia. Select from the dropdown lists the appropriate response to complete the statement below that outlines their tax status for the two years. In terms of employment status, Jacob must have been ___ for both 2017 and 2018. Sophia was ___ for 2017 but ___ for 2018.

an employee, an employee, self-employed

Lena is a sole proprietor. In April of this year, she sold equipment purchased four years ago for $26,000 with an adjusted basis of $15,500 for $17,000. Later in the year, Lena sold another piece of equipment purchased two years ago with an adjusted basis of $8,200 for $5,500. What are the tax consequences of these tax transactions? Lena has ___ of $___ from the sale of the first equipment. Lena has ___ of $___ from the sale of the second equipment.

an ordinary gain, $1,500 (17,000 - 15,500), a § 1231 loss, $2,700 (8,200 - 5,500)

Fred and Wilma were divorced in year 1 (before 2019). Fred is required to pay Wilma $12,000 of alimony each year until their child turns 18. At that time, the payment will be reduced to $10,000 per year. In year 3, in accordance with the divorce agreement, Fred paid $6,000 directly to Wilma and $6,000 directly to the law school Wilma is attending. What amount of the payments received in year 3 is income to Wilma? a. $6,000 b. $10,000 c. $12,000 d. $0

b. $10,000 Alimony is taxable to the recipient and deductible by the payor. Child support is not taxable to the recipient and not deductible by the payor. Because the total payment decreases to $10,000 once Fred and Wilma's child turns 18, the $2,000 decrease is deemed child support. The fact that Fred pays the law school in accordance with the divorce agreement on Wilma's behalf does not change the fact that $10,000 is considered alimony.

Complete the following statement regarding tax research. A large part of tax research consists of determining the

intent of Congress

Last year, Aleshia identified $16,400 as a nonbusiness bad debt. In that tax year before considering the tax implications of the nonbusiness bad debt, Aleshia had $32,800 of taxable income, of which $3,280 consisted of short-term capital gains. This year, Aleshia collected $3,936 of the amount she had previously identified as a bad debt. Determine Aleshia's tax treatment of the $3,936 received in the current tax year. Aleshia includes $___ of the collection as ___.

$3,936, ordinary income

Are land improvements used in a trade or business eligible for cost recovery? Land improvements ___ eligible for cost recovery, because they have ___.

are, a MACRS class life of 15 years

One of the tax advantages of hiring family members to work in your business is that FICA taxes are avoided. Complete the statement below. This statement is _____ because ___employed in the family's/parent's ___business are exempted from FICA tax.

false, only children under 18, unincorporated

Elliott has the following capital gain and loss transactions for 2018. a. Short-term capital gain $1,500 b. Short-term capital loss (3,600) c. Long-term capital gain (28%) 12,000 d. Long-term capital gain (25%) 4,800 e. Long-term capital gain (15%) 6,000 f. Long-term capital loss (28%) (4,500) g. Long-term capital loss (15%) (9,000) After the netting process, Elliott has an overall ___ of $___.

net long-term capital gain, $7,200 (page 14-20)

A taxpayer who uses the automatic mileage method to compute auto expenses can also deduct the business portion of tolls and parking.

True

An inheritance tax is a tax on a decedent's right to pass property at death.

False

Complete the statements below regarding at-risk and passive losses. The at-risk limitation limits a taxpayer's ___ to the amount "at risk," which is the amount the taxpayer stands to ___ if the investment becomes worthless. The passive loss rules require the taxpayer to segregate all income and losses into three categories: ___. In general, ___ losses can only offset ___ income.

deductions, lose, active, portfolio, and passive, passive, passive

The taxpayer performs services with payment due from the customer within 30 days. All customers pay within the time limit. What would be the benefit to the taxpayer using the cash method of accounting rather than the accrual method? The taxpayer can ___ by using the cash method of accounting when the customers or clients pay subsequent to the taxpayer delivering the goods or performing the services.

defer income

A corporation distributes a truck it has owned for three years to its sole shareholder. The shareholder will use the truck for personal use activity. The truck's fair market value at the time of the distribution is greater than its adjusted basis but less than its original cost. Does the corporation recognize a gain? If so, what is the character of the gain? The distribution of the truck is a ___ transaction for the corporation to the extent of the excess of the fair market value of the truck over the adjusted basis. ___ is § 1245 gain.

taxable, All of the gain

Identify the three factors reflected in the MACRS tables when the amount of cost recovery is determined. The three factors which the MACRS tables take into account are:

1) recovery period 2) method 3) convention

Employees of the Valley Country Club are allowed to use the golf course without charge before and after working hours on Mondays, when the number of players on the course is at its lowest. Tom, an employee of the country club played 40 rounds of golf during the year at no charge when the non-employee charge was $20 per round.

Tom is not required to include anything in gross income because this is a "no-additional-cost service" fringe benefit.

An employee with outside income may be able to avoid the penalty for underpayment of estimated tax by having his employer increase income tax withholdings.

True

Property which is classified as personalty may be depreciated.

True

Gary, who is single, sells his principal residence (owned and occupied by him for seven years) in November 2018 for a realized gain of $148,000. He had purchased a more expensive new residence eight months prior to the sale. He anticipates that he will occupy this new house as his principal residence for only about 18 additional months. He expects it to appreciate substantially while he owns it. Gary would like to recognize the realized gain on the 2018 sale to offset a large investment loss from the sale of stock. Select from the dropdown list a response to the question, "Can Gary recognize the realized gain of $148,000 on the sale of his principal residence in 2018?"

Yes, Gary can elect to forgo the § 121 exclusion.

Answer the following regarding the earned income credit. Can the earned income credit be characterized as a form of negative income tax? ___, because the earned income credit is a ____ credit.

Yes, refundable

Answer the following questions regarding excise and sales taxes. a. What is the difference between an excise tax and a general sales tax? __ ______ ___ is limited to a particular transaction, while _ _______ _____ ___ covers a multitude of transactions. b. Do all states impose a general sales tax? c. Does the Federal government impose a general sales tax?

a. An excise tax, a general sales tax b. No c. No

Aubry, a cash basis and calendar year taxpayer, decides to reduce his taxable income for 2018 by buying $65,000 worth of supplies for his business on December 27, 2018. The supplies will be used up in 2019. a. Can Aubry deduct the expenditure for 2018? b. If Aubry had bought the supplies because the seller was going out of business and offered a large discount on the price, can he deduct the expenditure for 2018?

a. No, because he was motivated by tax considerations. b. Yes, because there is a business reason.

Indicate whether the following assets are capital assets. a. Inventory b. Real estate used in a business c. Personal residence d. Stock held for investment

a. Not a capital asset b. Not a capital asset c. Capital asset d. Capital asset

Cheryl incurred $8,700 of medical expenses in November 2018. On December 5, the clinic where she was treated mailed her the insurance claim form it had prepared for her with a suggestion that she sign and return the form immediately to receive her reimbursement from the insurance company by December 31. Indicate which of following are tax issues that Cheryl should consider in deciding whether to sign and return the form in December 2018 or January 2019. a. If she receives reimbursement in 2019, she is not required to reduce her 2018 medical expense deduction by the amount of the anticipated reimbursement. b. If she receives reimbursement in 2018, she must increase her medical expense deduction by the amount of the reimbursement. c. She should consider her expected marginal tax rates for 2018 and 2019 and determine whether it is better to receive the reimbursement in 2018 or 2019. d. If she receives the reimbursement in 2019 and deducted the expenses in 2018, she must include the reimbursement in gross income to the extent she received a tax benefit from the medical expense deduction in 2018.

a. Yes, a tax issue b. Not a tax issue c. Yes, a tax issue d. Yes, a tax issue

Complete the following regarding the treatment of a business bad debt when the business also has long-term capital gains. The business bad debt is treated as ___, hence, the long-term capital gains are ___.

an ordinary loss, not relevant in determining the net loss

In which, if any, of the following situations will the kiddie tax not apply? a. The child has unearned income that exceeds more than half of his (or her) support. b. The child has unearned income of $2,100 or less. c. The child is married but does not file a joint return. d. The child is under age 24 and a full-time student. e. None of these choices are correct.

b. The child has unearned income of $2,100 or less.

The Sage Company has the opportunity to purchase a building located next to its office. Sage would use the building as a day care center for the children of its employees and an exercise facility for the employees. Occasionally, portions of the building could be used for employees' family events such as reunions, birthday parties, and anniversaries. The company would like to know if the planned uses of the building would fit into a beneficially taxed employee compensation plan. Complete the statement below regarding this potential employee fringe benefit. The employee does not have to include in gross income the value of ___ paid for by the employer. The exclusion cannot exceed $___ per ___. For a married couple, the annual exclusion cannot exceed ___of the spouse who has the lesser amount of ___. For an unmarried taxpayer, the exclusion cannot exceed the taxpayer's ___. The value of ___by employees, their spouses, and their dependent children ___ be excluded from an employee's ___. The facility ___ on the employer's premises, and ___ of the facility must be by employees and their family members.

child and dependent care services, $5,000, year, earned income, earned income, earned income, the use of a gymnasium or other athletic facility, may, gross income, must be, substantially all of the use

Which of the following is a required test for the deduction of a business expense? a. Reasonable b. Necessary c. Ordinary d. All of these choices are correct

d. All of these choices are correct

Kyle, whose wife died in December 2015, filed a joint tax return for 2015. He did not remarry, but has continued to maintain his home in which his two dependent children live. What is Kyle's filing status in 2018? a. Surviving spouse b. Single c. Married filing separately d. Head of household e. None of these choices are correct.

d. Head of household

Mason Gregg's car was destroyed by a flood that occurred in a Federally declared disaster area. Unfortunately, his insurance had lapsed two days before he incurred the loss. Mason uses his car for both business and personal use. Mason, who is self-employed, does not have adequate savings to replace the car and must borrow money to purchase a new car. He is considering using his credit card, at a 12% interest rate, to obtain funds for the purchase. Margaret, his wife, would prefer to sell some of their stock in Bluebird, Inc., to raise funds to purchase the new car. Mason does not want to sell the stock because it has declined in value since they purchased it and he is convinced that its price will increase in the next two years. Mason has suggested that they obtain conventional bank financing for the purchase, which charges 7% interest on car loans. Note: Although there are many nontax issues (financial, investment, and personal) involved in the Greggs' decision, this discussion is limited to the tax issues. Complete the statements below regarding the identified tax issues. Interest: Interest on the loans will be ___ Sale of Stock: If the Greggs have capital gains in excess of the loss, ___ loss can be offset against the gains. If they have no capital gains, they will be limited to a capital loss deduction of $___ in the year of sale, with excess capital losses being ___. Other: There will also be tax issues related to ___ on the car, but those issues are independent of the source of funding for the purchase.

deductible to the extent used for business purposes, the entire, $3,000, carried forward, depreciation

Thomas receives tangible personal property as an inheritance from a decedent who died in 2018. The property was depreciated by the deceased, and Thomas will also depreciate it. At the date of the deceased's death, the property was worth more than the deceased's adjusted basis. What is the impact of these facts on Thomas when he sells the property at a gain several years after he acquired it? The § 1245 depreciation recapture ___ carry over from a decedent. New § 1245 depreciation recapture potential ___ created as Thomas depreciates the asset.

does not, is

Complete the following statements regarding the formula for Federal gift tax. Federal gift tax usually ____ ___ apply to the full amount of the gift. Prior taxable gifts ____ __ _____ in arriving at the tax base to which the unified transfer tax rate is applied. Because the gift tax __ cumulative, a credit __ allowed against the gift taxes paid (or deemed paid) on prior taxable gifts included in the tax base. Because Congress did not intend for the gift tax to apply to smaller transfers, it provided for an annual exclusion which is $__,___for 2018.

does not, must be added, is, is, $15,000

Robin inherits 1,000 shares of Wal-Mart stock from her aunt in 2018. According to the information received from the executor of her aunt's estate, Robin's adjusted basis for the stock is $55,000. Albert, Robin's fiancé, receives 1,000 shares of Wal-Mart stock from his uncle as a gift in 2018. His uncle tells Albert that his adjusted basis for the Wal-Mart stock is $7,000. What could cause the substantial difference in the adjusted basis for Robin's and Albert's respective 1,000 shares of Wal-Mart stock? For inherited property, the basis for the heir is the ___. For property received by gift, the basis for the donee is the ___.

fair market value on the date of the decedent's death, donor's basis plus any gift tax paid

Michael, who is single, age 20 and claimed by his parents as a dependent, earned $20,000 at the K-M Resort Golf Club during the summer prior to his senior year in college. He wants to make a contribution to a traditional IRA for the tax year 2018, but the amount is dependent on whether it reduces his taxable income. If Michael is going to claim the standard deduction, will a contribution to a traditional IRA reduce his taxable income? Because the traditional IRA contribution is a deduction ___ AGI, it ___ reduce his taxable income and ___ impact on his itemized deductions.

for, does, has no

During the year, Tucker had the following personal casualty gains and losses (after deducting the $100 floor): Asset Holding Period Gain/(Loss) Asset 1 18 months ($1,200) Asset 2 2 months 750 Asset 3 3 years 1,500 As a result of these transactions, Tucker has net personal casualty ___ of ___. Therefore, he treats all of the gains and losses as ____ gains and losses.

gain, 1,050, capital

Auralia owns stock in Orange Corporation and Blue Corporation. She receives a $10,000 distribution from both corporations. The instructions from Orange state that the $10,000 is a dividend. The instructions from Blue state that the $10,000 is not a dividend. Complete the following statements to explain the difference in the treatment of the distributions. Orange Corporation apparently ___ resulting in the distribution being treated as ___. Blue Corporation ___ to cover the amount of the distribution resulting in the distribution being treated as ___.

has adequate earnings and profits, a dividend, does not have adequate earnings and profits, a return of capital

Pebble Securities is a corporation that buys and sells financial assets. It purchases notes receivable from manufacturers that need cash immediately and cannot wait to collect the notes. Pebble pays about 88% of the face value of the receivables and then collects them. Because of the quality of the notes, Pebble collected less than it paid for some of the notes. Does Pebble have a capital loss when it collects the receivables for less than it paid for them? Pebble is in the business of buying and selling financial assets. Consequently, the notes receivable it acquires are ___ and are considered ___. Therefore, the loss from the disposition of the notes receivable is ___.

held as inventory, ordinary assets, an ordinary loss

Noelle's diamond ring was stolen in 2017. She originally paid $8,000 for the ring, but it was worth considerably more at the time of the theft. Noelle filed an insurance claim for the stolen ring, but the claim was denied. Because the insurance claim was denied, Noelle took a casualty loss for the stolen ring on her 2017 tax return. In 2017, Noelle had AGI of $40,000. In 2018, the insurance company had a "change of heart" and sent Noelle a check for $5,000 for the stolen ring. The per event floor is $100. What is the proper tax treatment of the $5,000 Noelle received from the insurance company in 2018? Noelle should ___ the amount of $___.

include as income in 2018, $3900 [8,000 - 100 - (10% x 40,000 = 4,000)]

Thelma inherited land from Sadie on June 7, 2018. The land appreciated in value by 100% during the six months it was owned by Sadie. The value has remained stable during the three months Thelma has owned it, and she expects it to continue to do so in the near future. Although she would like to sell the land now, Thelma has decided to postpone the sale for another three months. The delay is to enable the recognized gain to qualify for long-term capital gain treatment. For inherited property, Thelma's holding period ___. Therefore, if she sells the property now, it will be treated as a ___ capital gain or loss.

is automatically long-term, long-term

In May 2018, Hernando, a resident of California, has his 2016 Federal income tax return audited by the IRS. An assessment of additional tax is made because he had inadvertently omitted some rental income. In October 2018, California audits his state return for the same year. Complete the following statement regarding the coincidence. What happened here likely __ ___ a coincidence. ___ ___ probably notified the state of California regarding Hernando's omission of income.

is not, The IRS

A depreciable business dump truck has been owned for four years and is no longer useful to the taxpayer. What would have to be true for the disposition of the dump truck to generate at least some § 1231 loss? The dump truck would have to be sold for ___ than its adjusted basis.

less

Determine the net effect on Tamara's adjusted gross income with regard to these capital asset transactions that occurred this year. • Sold ABCCo stock, acquired 2 years ago, for a $1,500 loss. • Sold collectible coins, held for 17 months, for a $2,000 gain. • Sold XYZCo shares, acquired 6 months ago, for a $4,100 loss. • Sold LMNCo stock, acquired 3 years ago, for a $500 gain. As a result of these transactions, Tamara has a ___ of ___ and a ___ of ___. Overall, Tamara has a net ___ of ___.

net long-term capital gain, $1,000, short-term capital loss, $4,100, short-term capital loss, $3,100

Joan is a self-employed consultant. What is her exposure to the Federal self-employment tax? What tax rates that apply to Joan's profits, and the income base amounts for the year? Individuals with ___ earnings of $___ or more from self-employment are subject to the self-employment tax. For 2018, the self-employment tax is ___ percent of self-employment earnings up to $___ for the Social Security portion plus ___ percent of the total amount of self-employment earnings for the Medicare portion. Self-employed taxpayers deduct a portion of the self-employment tax in determining ___ as well as a deduction ___ computing AGI.

net, $400, 12.4, $128,400, 2.9, the self-employment tax, for

Ron, a cash basis taxpayer, sells his business accounts receivable of $100,000 to Felicia for $70,000 (70% of the actual accounts receivable). What is the amount and classification of Ron's bad debt deduction? Ron has ___debt deduction because he is a cash basis taxpayer. Ron has $___ of ___.

no bad, 70,000, income

Complete the following statements regarding business and nonbusiness bad debts. A ___ bad debt is a debt unrelated to the taxpayer's trade or business either when it was created or when it became worthless. The use to which the borrowed funds are put by the debtor ___. A ___ bad debt is deductible as an ordinary loss in the year incurred, whereas a ___ bad debt is always treated as a short-term capital loss. Although no deduction is allowed when a ___ bad debt is partially worthless, the taxpayer is entitled to deduct the net amount of the loss upon final settlement. The nonbusiness bad debt provisions ___ applicable to corporations.

nonbusiness, is of no consequence, business, nonbusiness, nonbusiness, are not

Edith exchanges a building and land used in her business for land and stock of Teal, Inc. If Edith had sold her building and land, she would have had a realized gain. Why would the new land have a different holding period than the stock? The exchange ___ as a like-kind exchange ___ involved. As a result, the land acquired has a ___ and the stock has a ___.

qualifies, with boot, carryover basis and holding period, fair market value basis and new holding period starting on the date of the exchange

Dino contributes to charity some tangible personal property that he had used in his business and depreciated. At the date of the donation, the property has a fair market value greater than its adjusted basis, but less than the original cost. What is the impact of these facts on Dino's charitable contribution? The charitable contribution deduction is ___ to the extent there is a potential § 1245 depreciation recapture on the donated property. In effect, the charitable contribution is equal to the ___ of such appreciated property.

reduced, adjusted basis

Complete the statement below in response to the question: "How does the § 179 immediate expensing deduction affect the computation of MACRS cost recovery?" The basis of the property for cost recovery purposes is ___ by the § 179 amount. The business income limitation ___ affect basis.

reduced, does not

A corporation distributes a truck it has owned for three years to its sole shareholder. The shareholder will use the truck for business activity. The truck's fair market value at the time of the distribution is greater than its adjusted basis but less than its original cost. Does the corporation recognize a gain? If so, what is the character of the gain? The distribution of the truck is a ___ transaction for the corporation to the extent of the excess of the fair market value of the truck over the adjusted basis. Because the shareholder is a related taxpayer and will use the truck in a business ___ is ordinary income.

taxable, all of the gain

Ben lost his job when his employer moved its plant. During the year, he collected unemployment benefits for three months, a total of $1,800. While he was waiting to hear from prospective employers, he painted his house. If Ben had paid someone else to paint his house, the cost would have been $3,000. The cost of the paint Ben used was $800. What is Ben's gross income for tax purposes from the above events? Ben ___ recognize income from the unemployment benefits. His savings from painting his house ___ included in gross income because it ___ income ___. This is because the savings ___ an amount received from another. Therefore, Ben will report $___ as income.

will, are not, was not, realized, were not, $1800

Amy lives and works in St. Louis. In the morning she flies to Boston, has a three-hour business meeting, and returns to St. Louis that evening. For tax purposes, Amy was away from home.

False

Joyce owns an activity (not real estate) in which she participates for 100 hours a year; her spouse participates for 450 hours. Joyce qualifies as a material participant.

True

Joe was in an accident and required cosmetic surgery for injuries to his nose. He also had the doctor do additional surgery to reshape his chin, which had not been injured. a. Will the cosmetic surgery to Joe's nose qualify as a medical expense? b. Will the cosmetic surgery to Joe's chin qualify as a medical expense?

a. Yes, because the surgery was necessary due to injury or abnormality. b. No, because unnecessary cosmetic surgery is nondeductible.

The effects of a below-market loan for $100,000 made by a corporation to its chief executive officer as an enticement to get him to remain with the company are: a. The employee has no income unless the funds are invested and produce investment income for the year. b. The corporation has imputed interest income and the employee is deemed to have received a gift. c. The corporation has imputed interest income and dividends paid. d. The employee has imputed compensation income and the corporation has imputed interest income. e. None of these choices are correct.

d. The employee has imputed compensation income and the corporation has imputed interest income.

Joe, a cash basis taxpayer, took out a 12-month business loan on December 1, 2018. He prepaid all $3,600 of the interest on the loan on December 1, 2018. Joe can deduct only $300 of the prepaid interest in 2018.

True

Carlos owns an interest in an activity that produces a $100,000 loss during the year. Complete the statements below regarding whether Carlos would prefer to have the activity classified as active or passive. Generally, if an activity is classified as passive, a loss from that activity can offset ___ income. An active loss can offset income from ___ . Therefore, Carlos would prefer to have the activity classified as ___.

only passive activity active, passive, and portfolio active

Taxes levied by all states include: a. Individual income tax. b. Inheritance tax. c. Tobacco excise tax. d. General sales tax. e. None of these choices are correct.

c. Tobacco excise tax.

Olaf was injured in an automobile accident and received $25,000 for his physical injury, $50,000 for his loss of income, and $10,000 punitive damages. As a result of the award, the amount Olaf must include in gross income is:

$10,000

On October 1, 2018, Priscilla purchased a business. Of the purchase price, $60,000 is allocated to a patent and $375,000 to goodwill. If required, round your answer to the nearest dollar. The 2018 § 197 amortization deduction is $___

$7,250 [(60,000/15 * 3/12) + (375,000/15 * 3/12)]

In 2017, Fred invested $127,000 in a general partnership. Fred's interest is not considered to be a passive activity. If his share of the partnership losses is $88,900 in 2017 and $62,230 in 2018, how much can he deduct in each year? Fred can deduct $___ in 2017 and $___ in 2018.

$88,900 $38,100 (127,000 - 88,900)

Indicate whether the following statements are "True" or "False" regarding the rules designed to limit the tax benefits a taxpayer may obtain from a tax shelter investment. 1. The tax benefits associated with tax shelter investments are reduced by the at-risk provisions and the passive activity loss rules. 2. The passive activity loss rules only apply to individuals and personal service corporations. 3. A taxpayer's deductible loss from an activity for any taxable year is limited to the amount the taxpayer has at risk at the end of the taxable year. 4. Losses from passive activities that are disallowed in the current year may be carried forward to future years where they may provide a tax benefit. 5. In general, passive activity losses can only offset passive activity income.

1. True 2. False 3. True 4. True 5. True

The income of a sole proprietorship is reported on Schedule C (Profit or Loss from Business).

True

Trade or business expenses of a self-employed taxpayer should be treated as:

Deductible for AGI on Schedule C.

Mason, a physically handicapped individual, pays $10,000 this year for the installation of wheelchair ramps, support bars, and railings in his personal residence. These improvements increase the value of his personal residence by $2,000. Only $8,000 of the expenditure qualifies as a medical expense for tax purposes.

False

Holly was injured while working in a factory and received $12,000 as workers' compensation as she was unable to work because of the injury. Jill, who was self-employed, was also injured and unable to work. Jill collected $12,000 on an insurance policy she had purchased to replace her loss of income while she was unable to work. How much are Holly and Jill each required to include in their gross income? If an amount is zero, enter "0".

Holly: $0 Jill: $0

Answer the following question regarding personal use property. Is property that is classified as personal use property subject to cost recovery?

No

An individual taxpayer had a net § 1231 loss in 2018. Could any of this loss be treated as a long-term capital loss? ___, because if the disposition of depreciable property and real property used in business results in a net loss, § 1231 treats the loss as ___ rather than as ___. This type of loss is ___ for adjusted gross income (AGI).

No, an ordinary loss, a capital loss, fully deductible

Kathy owns and operates a grocery store as a sole proprietor. She pays wages to her husband Joe and their 17-year-old daughter Marla, both of whom work at the store. Should Kathy withhold FICA taxes from the wages paid to Joe and Marla?

Only her husband is subject to FICA

The cost recovery basis for property converted from personal use to business use may be the fair market value of the property at the time of the conversion.

True

The base amount for the Social Security portion (old age, survivors, and disability insurance) is different from that for the Medicare portion of FICA.

True

A rationale for the installment method tax rule is: a. Equity and fairness. b. Ability to pay. c. Revenue neutrality. d. Simplicity.

b. Ability to pay.

For a person who receives Social Security benefits, what effect, if any, can an increase in other income have on that person's taxable income? Other income is ___in the computation of modified adjusted gross income (used to determine taxable Social Security benefits). Therefore any increase in other income ___ the Social Security benefits to be taxable, thus ___ taxable income.

included, may cause more of, increasing

Andrew, who operates a laundry business, incurred the following expenses during the year. * Parking ticket of $250 for one of his delivery vans that parked illegally. * Parking ticket of $75 when he parked illegally while attending a rock concert in Tulsa. * DUI ticket of $500 while returning from the rock concert. * Attorney's fee of $600 associated with the DUI ticket. What amount can Andrew deduct for these expenses?

$0

Janet works at Green Company's call center. If Janet's compensation is based on the number of calls she handles, she is an independent contractor.

False

Deductions are allowed unless a specific provision in the tax law provides otherwise.

False

In 2018, a taxpayer takes six clients to an NBA playoff game. If all of the tickets (list price of $120 each) are purchased on the Internet for $1,800 ($300 each), only $60 ($120 × 50% overall limitation) per ticket is deductible.

False

In January 2019, Pam, a calendar year cash basis taxpayer, made an estimated state income tax payment for 2018. The payment is deductible in 2018.

False

In determining whether a debt is a business or nonbusiness bad debt, the debtor's use of the borrowed funds is important.

False

In the current year, Don has a $55,000 loss from a business he owns. His at-risk amount at the end of the year, prior to considering the current year loss, is $36,000. He will be allowed to deduct the $55,000 loss this year if he is a material participant in the business.

False

Juan refuses to give the bank where he maintains a savings account his Social Security number. Juan is subject to backup withholding for the interest earned on the savings account.

True

Letha incurred a $1,600 prepayment penalty to a lending institution because she paid off the mortgage on her home early. The $1,600 is deductible as interest expense.

True

Alfred owned a term life insurance policy at the time he was diagnosed as having a terminal illness. After paying $64,550 in premiums, he sold the policy to a company that is authorized by the state of South Carolina to purchase such policies. The company paid Alfred $451,850. When Alfred died 18 months later, the company collected the face amount of the policy, $542,220. Alfred is required to include $__ in his gross income as a result of the sale of the policy.

$0

Derek, a cash basis, unmarried taxpayer, had $2,900 of state income tax withheld during 2018. Also in 2018, Derek paid $290 that was due when he filed his 2017 state income tax return and made estimated payments of $5,800 towards his 2018 state income tax liability. When Derek files his 2018 Federal income tax return in April 2019, he elects to take the standard deduction, which reduced his taxable income. As a result of overpaying his 2018 state income tax, Derek receives a refund of $2,030 in 2019. How much of the $2,030 will Derek include in his 2019 gross income?

$0

Early in the year, Marion was in an automobile accident during the course of his employment. As a result of the physical injuries he sustained, he received the following payments during the year: Reimbursement of medical expenses Marion paid by a medical insurance policy he purchased $10,000 Damage settlement to replace his lost salary 15,000 What is the amount that Marion must include in gross income for the current year?

$0

Ellie purchases an insurance policy on her life and names her brother, Jason, as the beneficiary. Ellie pays $32,000 in premiums for the policy during her life. When she dies, Jason collects the insurance proceeds of $500,000. As a result, Jason reports gross income of $___.

$0

In 2018, Robert entertains four key clients and their spouses at a nightclub. Business discussions occurred over dinner and prior to the entertainment beginning. Expenses were $465 (limo charge), $125 (cover charge), $1,015 (drinks and dinner), and $203 (tips to servers). If Robert is self-employed, how much can he deduct as an entertainment expense for this event?

$0

In 2018, the CEO of Crimson, Inc., entertains 15 clients at a skybox in Memorial Stadium for a single athletic event during the year. Substantive business discussions occurred at various times during the event. The box cost $2,850 per event and seats 17 people. (The cost of a regular, nonluxury box seat at Memorial ranges from $40 to $80.) Refreshments served during the event cost $1,445. How much of these costs may Crimson deduct as an entertainment expense?

$0

This year, Nadia donates $17,125 to Eastern University's athletic department. The payment guarantees that Nadia will have preferred seating at football games near the 50-yard line. Assume that Nadia subsequently buys six $120 game tickets. How much can she deduct as a charitable contribution to the university's athletic department? $___

$0 A charitable contribution is defined as a gift made to a qualified organization. The major elements needed to qualify a contribution as a gift are a donative intent, the absence of consideration, and acceptance by the donee. Generally when a donor derives a tangible benefit from a contribution, he or she cannot deduct the value of the benefit. That is the case here. Nadia cannot deduct any portion of the $4,000 donation since it relates to getting preferred seating at an athletic event (here, football games).

On June 2, 2017, Fred's TV Sales sold Mark a large HD TV, on account, for $12,000. Fred's TV Sales uses the accrual method. In 2018, when the balance on the account was $8,000, Mark filed for bankruptcy. Fred was notified that he could not expect to receive any of the amount owed to him. In 2019, final settlement was made and Fred received $1,000. How much bad debt loss can Fred deduct in 2019?

$0 This debt is a business debt. Therefore, partial worthlessness can be recognized in 2018. The loss in 2018 would be $8,000. In 2019, the account has been written down to zero and hence, the collection of $1,000 would produce a $1,000 ($1,000 - $0) gain rather than a loss.

Sandstorm Corporation decides to develop a new line of paints. The project begins in 2018. Sandstorm incurs the following expenses in 2018 in connection with the project: Salaries $85,000 Materials 30,000 Depreciation on equipment 12,500 The benefits from the project will be realized starting in July 2019. If an amount is zero, enter "0". Do not round intermediate calculations. If Sandstorm Corporation elects a 60-month deferral and amortization period, there is a ___ for 2018 and a ___ deduction for 2019.

$0, $12,750 Since the benefits from the project will be realized starting in July 2019, Sandstorm Corporation has no deduction prior to July 2019, the month benefits from the project begin to be realized. The deduction for 2019 is $12,750, computed as follows: Research and Experimental Expenditures: $85,000 + $30,000 + $12,500 = $127,500. 2019 Deduction: $127,500 x 6 months /60 months = $12,750.

Myrna and Geoffrey filed a joint tax return in 2017. Their AGI was $85,000, and itemized deductions were $13,700, which included $4,000 in state income tax. In 2018, they received a $1,800 refund of the state income taxes that they paid in 2017. The standard deduction for married filing jointly in 2017 was $12,700. Under the tax benefit rule, $___ of the state income tax refund is included in gross income in 2018.

$1,000 ($13,700 itemized deductions - $12,700 standard deduction)

Data, Inc., purchased and placed in service a $5,000 computer on August 24, year 3. This is the only asset purchase during the year. Section 179 expensing was not elected. Using the excerpt of the MACRS half-year convention table below, what is the MACRS depreciation in year 3 for the computer?

$1,000 (20% x $5,000)

Complete the following statements regarding estimated taxes. Enter percents as whole numbers. Any individual who has estimated tax for the year of $___ or more and whose withholding ___ equal or exceed the required annual payment must make quarterly payments. The required annual payment is the ___ of 1) ___% of the tax shown on the current year's return or 2) ___% of the tax shown on the preceding year's return. If the AGI on the preceding year's return exceeds $150,000, the requirement is increased to ___%.

$1,000, does not, smaller, 90%, 100%, 110%

On March 25, 2018, Parscale Company purchases the rights to a mineral interest for $8,000,000. At that time, the remaining recoverable units in the mineral interest are estimated to be 500,000 tons. If required, round any division to two decimal places and use in subsequent computations. Round your final answer to the nearest dollar. If 80,000 tons are mined and 75,000 tons are sold this year, the cost depletion is $___

$1,200,000

On January 1, 2018, Kunto, a cash basis taxpayer, pays $46,228 for a 24-month certificate. The certificate is priced to yield 4% (the effective interest rate) with interest compounded annually. No interest is paid until maturity, when Kunto receives $50,000. In your computations, round any amounts to the nearest dollar. Kunto has $___ income for 2018 and $___for 2019.

$1,849 ($46,228 x 4%), $1,923 ($46,228 + $1,849) x 4%)

Mio was transferred from New York to Germany. He lived and worked in Germany for 356 days in 2018. Mio's salary for 2018 is $248,000. In your computation, round any division to four decimal places before converting to a percentage. For example, .473938 would be rounded to .4739. If required, round your final answer to the nearest dollar. Mio's foreign earned income exclusion is $

$101,334 $103,900 (2018 limit) x (356 days in Germany/365 in the year) = $101,334.

Euclid acquires a 7-year class asset on May 9, 2018, for $80,000. Euclid does not elect immediate expensing under § 179. He does not claim any available additional first-year depreciation. Click here to access the depreciation table to use for this problem. If required, round your answers to the nearest dollar. Euclid's cost recovery deduction is $___ for 2018 and $___ for 2019.

$11,432, $19,592 2018: $80,000 x .1429 (Exhibit 8.3) = $11,432 2019: $80,000 x .2449 (Exhibit 8.3) = $19,592

Wilbur has been offered a job at a salary that would put him in the 24% marginal tax bracket. In addition to his salary, he would receive health insurance coverage. Another potential employer does not offer health insurance but has agreed to match the first offer on an after-tax and insurance basis. The cost of health insurance comparable to that provided by the other potential employer is $9,000 per year. How much more in salary must the second potential employer pay so that Wilbur's financial status will be the same under both offers?

$11,842 [$9,000/(1 - .24)]

Jordan Johnson is single and has adjusted gross income of $50,000 in the current year. Additional information is as follows: State income taxes paid $2,000 Mortgage interest on her personal residence 9,000 Points paid on purchase of her personal residence 1,000 Deductible contributions to her IRA 3,000 Uninsured realized casualty loss (in a Federal disaster area) 6,000 Tax preparation fees for her prior year income tax return 400 What amount may Jordan claim as itemized deductions on her current-year income tax return?

$12,900 (????)

Maud, a calendar year taxpayer, is the owner of a sole proprietorship that uses the cash method. On February 1, 2018, she leases an office building to use in her business for $120,000 for an 18-month period. To obtain this favorable lease rate, she pays the $120,000 at the inception of the lease. How much rent expense may Maud deduct on her 2018 tax return?

$120,000

Rex became a partner with a 30% interest in the partnership profits when he invested $200,000. In 2018, the partnership generated $400,000 of taxable income, and Rex withdrew $100,000. In 2019, the partnership had $600,000 of taxable income, and Rex withdrew $200,000. What is Rex's gross income from the partnership in 2018 and 2019? 2018: 2019:

$120,000 ($400,000 x 30%), $180,000 ($600,000 x 30%)

Luciana, a nonshareholder, purchases a condominium from her employer for $85,000. The fair market value of the condominium is $120,000. What is Luciana's basis in the condominium and the amount of any income as a result of this purchase? Luciana's basis in the condominium is $___ and she reports ___ as ___.

$120,000, $35,000, additional compensation

Renata Corporation purchased equipment in 2016 for $180,000 and has taken $83,000 of regular MACRS depreciation. Renata Corporation sells the equipment in 2018 for $110,000. What is the amount and character of Renata's gain or loss? Renata Corporation has a gain of $___ of which $___ is treated as ordinary income due to ___.

$13,000 (180,000 - 83,000 - 110,000), $13,000, § 1245 recapture

Bernadette suffers from multiple sclerosis and has great difficulty climbing stairs. Upon the recommendation of her physician, she has an elevator chair-system installed in her personal residence. In connection with the system, Bernadette incurs and pays the following amounts during the current year: Doctor and hospital bills $6,700 Elevator chair-system 19,500 Installation charge for elevator chair-system 1,500 Cost of certified appraisal 500 In addition, Bernadette pays $1,350 for prescribed medicines. The system has an estimated useful life of 25 years. The appraisal was to determine the value of Bernadette's residence with and without the system. The appraisal states that her residence was worth $224,000 before the system was installed and $240,000 after the installation. Bernadette's AGI for the year was $83,000. What is the total of her qualifying medical expenses for 2018? What is her medical expense deduction for the current year?

$13,050 = [$6,700 + 1,500 + 1,350 + (19,500 - (240,000 - 224,000))] $6,825 (7.5% * $83,000) = 6,225 13,050 - 6,225 = $6,825

Jarrod receives a scholarship of $18,500 from Riggers University to be used to pursue a bachelor's degree. He spends $12,000 on tuition, $1,500 on books and supplies, $4,000 for room and board, and $1,000 for personal expenses. Jarrod may exclude $___ from his gross income.

$13,500 (12,000 + 1,500)

Seojun acquired an activity several years ago, and in the current year, it generates a loss of $50,000. Seojun has AGI of $140,000 before considering the loss from the activity. If the activity is a bakery and Seojun is not a material participant, what is his AGI? $___

$140,000

Logan and Johnathan exchange land, and the exchange qualifies as like kind under § 1031. Because Logan's land (adjusted basis of $85,000) is worth $100,000 and Johnathan's land has a fair market value of $80,000, Johnathan also gives Logan cash of $20,000. a. Logan's recognized gain is $___ b. Assume that Johnathan's land is worth $90,000 and he gives Logan $10,000 cash. Logan's recognized gain is $___.

$15,000 (100,000 - 85,000) $10,000 (Lesser of gain or boot = 15,000 or 10,000)

Three years ago, Sharon loaned her sister $30,000 to buy a car. A note was issued for the loan with the provision for monthly payments of principal and interest. Last year, Sharon purchased a car from the same dealer, Hank's Auto. As partial payment for the car, the dealer accepted the note from Sharon's sister. At the time Sharon purchased the car, the note had a balance of $18,000. During the current year, Sharon's sister died. Hank's Auto was notified that no further payments on the note would be received. At the time of the notification, the note had a balance due of $15,500. What is the amount of loss, with respect to the note, that Hank's Auto may claim on the current year tax return?

$15,500

Leland pays premiums of $5,000 for an insurance policy in the face amount of $25,000 upon the life of Caleb and subsequently transfers the policy to Tyler for $7,500. Over the years, Tyler pays subsequent premiums of $1,500 on the policy. Upon Caleb's death, Tyler receives the proceeds of $25,000. As a result, Tyler is required to include $___ in her gross income.

$16,000 [$25,000 proceeds - ($7,500 paid for policy + $1,500 in subsequent premiums)]

Miller owns a personal residence with a fair market value of $200,000 and an outstanding first mortgage of $160,000, which was used entirely to acquire the residence. This year, Miller gets a home equity loan of $10,000 to purchase new jet skis. Interest on the $___ of this mortgage debt is treated as qualified residence indebtedness.

$160,000

Arturo, a calendar year taxpayer, paid $17,000 in medical expenses and sustained a $20,400 casualty loss in 2018 (the loss occurred in a Federally declared disaster area). He expects $11,900 of the medical expenses and $14,280 of the casualty loss to be reimbursed by insurance companies in 2019. How much can Arturo include in determining his itemized deductions for 2018? Disregard %-of-AGI limitations or casualty loss floor in determining your answers. Before considering any limitations (or reductions) on deductions, Arturo can include $___ of the medical expenses and ___ of the casualty loss when determining his itemized deductions in 2018.

$17,000 $6,120 (20,400 - 14,280)

Tobias has a brokerage account and buys on the margin, which resulted in an interest expense of $20,000 during the year. Income generated through the brokerage account was as follows: Municipal interest $50,000 Taxable dividends and interest 350,000 In your computations, do not round any division. How much investment interest can Tobias deduct?

$17,500 (350,000/400,000 = .875 * 20,000 = 17,500)

Gertrude, who is single, has the following items for the current year: Salary . $230,000 Gain on sale of § 1244 stock acquired five years ago $40,000 Loss on sale of § 1244 stock acquired nine months ago ($60,000) Nonbusiness bad debt ($15,000) Business bad debt ($20,000)

$175,000

On January 15, 2018, Dillon purchased the rights to a mineral interest for $3,500,000. At that time it was estimated that the recoverable units would be 500,000. During the year, 40,000 units were mined and 25,000 units were sold for $800,000. Dillon incurred expenses during 2018 of $500,000. The percentage depletion rate is 22%. Determine Dillon's depletion deduction for 2018.

$175,000 Cost depletion: $3,500,000 ÷ 500,000 = $7 per unit 25,000 units sold × $7 per unit = $175,000 Percentage depletion: $800,000 × 22% =$176,000 Limit: ($800,000 - $500,000) × 50% =$150,000 Greater of cost or percentage depletion = $175,000

Roberto invested $18,000 in a chicken production operation. Using nonrecourse notes, the business purchases $120,000 worth of grain to feed the chickens. Roberto's share of the expense is $26,000. Assuming that the passive activity loss rules do not apply, he can deduct $___

$18,000

The Wilmoths plan to purchase a house but want to determine the after-tax cost of financing its purchase. Given their projected taxable income, the Wilmoths are in the 24% Federal income tax bracket and the 8% state income tax bracket (i.e., an aggregate marginal tax bracket of 32%). Assume that the Wilmoths will benefit from itemizing their deductions for both Federal and state purposes. The total cash outlay during the first year of ownership will be $26,600 ($2,660 principal payments, $23,940 qualified residence interest payments). If required, round your interim calculation to nearest dollar. As a result, the annual after-tax cost of financing the purchase of the home will be $____.

$18,939 Nondeductible principal payments: $2,660 Deductible qualified residence interest taxes: $23,940 x (1 - .32) = $16,279 Total: $2,660 + $16,279 = $18,939.

Shen purchased corporate stock for $20,000 on April 10, 2016. On July 14, 2018, when the stock was worth $12,000, Shen died and his son, Mijo, inherited the stock. Mijo sold the stock for $14,200 on November 12, 2018. What is the amount and character of Mijo's gain or loss?

$2,200 (14,200 - 12,000), long-term capital gain

In 2018, Ava, an employee, who files single, has AGI of $28,800 and incurred the following miscellaneous itemized deductions this year: Union dues and work uniforms: $630 Home office expenses: $1,890 Unreimbursed employee expenses: $882 Gambling losses to the extent of gambling winnings: $2,325. What is Ava's total itemized deduction related to these items?

$2,325

Tabitha sells real estate on March 2 of the current year for $299,200. The buyer, Ramona, pays the real estate taxes of $14,960 for the calendar year, which is the real estate property tax year. Round any division to four decimal places and use in subsequent calculations. Round your final answers to the nearest dollar. Assume a 365-day year. $___ of the real estate taxes is apportioned to and is deductible by the seller, Tabitha, and $___ of the taxes is deductible by Ramona. Ramona's basis is in the property is $___ and the amount realized by Tabitha from the sale is $___.

$2,459 (60/365) * 14,960 $12,501 (14,960 - 2,459) $301,659 (2,459 + 299,200) $301,659

Peyton sells an office building and the associated land on May 1 of the current year. Under the terms of the sales contract, Peyton is to receive $1,600,000 in cash. The purchaser is to assume Peyton's mortgage of $950,000 on the property. To enable the purchaser to obtain adequate financing, Peyton is to pay the $9,000 in points charged by the lender. The broker's commission on the sale is $75,000. What is Peyton's amount realized? The amount realized by Peyton is $___

$2,466,000 (1,600,000 + 950,000 - 9,000 - 75,000)

Leonard's home was damaged by a fire. He also had to be absent from work for several days to make his home habitable. Leonard's employer paid Leonard his regular salary, $2,500, while he was absent from work. In Leonard's pay envelope was the following note from the employer "To help you in your time of need" and $900, a collection from Leonard's fellow employees. Leonard spent over $4,000 repairing the fire damage. Based on the above information, how much, if any, is Leonard required to include in his gross income?

$2,500

Three years ago, Marshall purchased an automobile for personal purposes for $38,000. In 2018, he contributes the automobile to his business. At the time of the contribution the automobile was appraised for $22,000. The basis for cost recovery of the automobile is $___.

$22,000

In the current year, Ed invests $30,000 in an oil partnership. He has taxable income for the current year of $2,000 from the oil partnership and withdraws $10,000. What is Ed's at-risk amount at the end of the year?

$22,000 (30,000 + 2,000 - 10,000)

Tan Company acquires a new machine (ten-year property) on January 15, 2018, at a cost of $200,000. Tan also acquires another new machine (seven-year property) on November 5, 2018, at a cost of $40,000. No election is made to use the straight-line method. The company does not make the § 179 election and elects to not take additional first-year depreciation. Determine the total deductions in calculating taxable income related to the machines for 2018.

$25,716 10-year property MACRS cost recovery ($200,000 × .10) $20,000 7-year property MACRS cost recovery ($40,000 × .1429) 5,716 Total cost recovery $25,716

Constanza, who is single, sells her current personal residence (adjusted basis of $165,000) for $450,000. She has owned and lived in the house for 30 years. Her selling expenses are $22,500. Constanza's realized gain is $___ and her recognized gain would be $___.

$262,500 (450,000 - 165,000 - 22,500), $12,500 (262,500 - 250,000)

Bruce, who is single, had the following items for the current year: * Salary of $80,000. * Gain of $20,000 on the sale of § 1244 stock acquired two years earlier. * Loss of $75,000 on the sale of § 1244 stock acquired three years earlier. * Worthless stock of $15,000. The stock was acquired on February 1 of the prior year and became worthless on January 15 of the current year. Determine Bruce's AGI for the current year.

$27,000 Salary $80,000 § 1244 ordinary loss (50,000) Long-term capital gain $20,000 Long-term capital loss Excess § 1244 loss ($75,000 - $50,000) $25,000 Worthless security 15,000 (40,000) Net long-term capital loss (limited) (3,000) Adjusted gross income $27,000

Indigo Company acquires a new machine (5-year MACRS property) on February 2, 2018 at a cost of $100,000. On November 18, 2018, Indigo also acquires office equipment (7-year MACRS property) at a cost of $50,000. Indigo does not make a § 179 expense election and chooses not to take additional first-year depreciation. What is Indigo's total MACRS deduction for 2018?

$27,145 5-year MACRS $100,000 × .2000 = $20,000 7-year MACRS $50,000 × .1429 = 7,145 Total = $27,145

Vella owns and operates an illegal gambling establishment. In connection with this activity, he has the following expenses during the year: Rent $24,000 Bribes 40,000 Travel expenses 4,000 Utilities 18,000 Wages 230,000 Payroll taxes 13,800 Property insurance 1,600 Illegal kickbacks 22,000 Vella's total deductible expenses for tax purposes are

$291,400 (everything but bribes and kickbacks)

Jena is a full-time undergraduate student at State University and qualifies as a dependent of her parents. Her only source of income is a $10,000 athletic scholarship ($1,000 for books, $5,500 tuition, $500 student activity fee, and $3,000 room and board). Jena's gross income for the year is:

$3,000 (for room and board)

George, an unmarried cash basis taxpayer, received the following amounts during 2018: Interest on savings accounts $2,000 Interest on a State tax refund 600 Interest on City of Salem school bonds 350 Interest portion of proceeds of a 5% bank certificate of deposit purchased on July 1, 2017, and matured on June 30, 2018 250 Dividends on USG common stock 300 What amount should George report as gross income from dividends and interest for 2018?

$3,150 (2,000 + 600 + 250 + 300)

Hamlet acquires a 7-year class asset on November 23, 2018, for $100,000. Hamlet does not elect immediate expensing under § 179. He does not claim any available additional first-year depreciation. Click here to access the depreciation table to use for this problem. If required, round your answers to the nearest dollar. Hamlet's cost recovery deduction is $___ for 2018 and $___ for 2019.

$3,570, $27,550 2018: $100,000 x .0357 (Exhibit 8.4) = $3,570 2019: $100,000 x .2755 (Exhibit 8.4) = $27,550

Reba, who is single, does a lot of business entertaining at home. Lawrence, Reba's 84-year-old dependent grandfather, lived with Reba until this year, when he moved to Lakeside Nursing Home because he needs medical and nursing care. During the year, Reba made the following payments on behalf of Lawrence: Room at Lakeside $22,275 Meals for Lawrence 4,455 Doctor & nurse fees 3,341 Cable TV service 446 Total $30,517 Lakeside has medical staff in residence. Disregarding the AGI floor, how much, if any, of these expenses qualifies for a medical expense deduction by Reba? The amount that qualifies for the medical expense deduction is $

$30,071

Ramirez's adjusted basis in a passive activity is $45,000 at the beginning of the year. His loss from the activity for the current year is $14,200. Ramirez has wages of $65,000 and dividend income of $300. At year-end, Ramirez has the following: Adjusted basis in the passive activity: $ At-risk amount in the passive activity: $ A suspended passive loss: $

$30,800 $30,800 $14,200

Sydney, a single taxpayer, had $80,000 in adjusted gross income in year 2. During the year, she contributed $15,000 to her church. She also had a $17,000 contribution carryover from her year 1 church contributions. What is the maximum amount of charitable deduction that Sydney may claim on her year 2 income tax return?

$32,000

An apartment building was acquired in 2009. The depreciation taken on the building was $123,000, and the building was sold for a $34,000 gain. What is the maximum amount of 25% gain? The maximum amount of unrecaptured § 1250 gain is $___

$34,000

Enzo is a single taxpayer with the following gains and losses for 2018: • $2,100 short-term capital loss. • $24,000 long-term capital gain from sale of stock. • $14,000 § 1231 gain that is all unrecaptured § 1250 gain. What is the amount and character of Enzo's gain or loss? Enzo's net capital gain is $___ and his potential unrecaptured § 1250 gain that is subject to the 25% tax rate is $___.

$35,900 (24,000 + 14,000 - 2,100), $11,900 (14,000 - 2,100)

Andrew sends Godiva chocolates to 14 of his key clients at Christmas. The chocolates cost $40 a box not including $2 for gift wrapping and shipping. How much can Andrew deduct?

$378 = [($25 x 14) + ($2 x 14)]

Troy's financial records for the year reflect the following: Interest income from bank savings account $2,000 Taxable annuity receipts $4,000 City ad valorem property tax on investment $300 Investment interest expense $7,000 Calculate Troy's net investment income and his current investment interest deduction. How is any potential excess investment interest deduction treated? Troy's net investment income is $___ and his investment interest deduction is $___. Any potential excess investment interest expense not deducted this year is ___.

$5,700 (2,000 + 4,000 - 300), $5,700, carried forward

Stanford owns and operates two dry cleaning businesses. He travels to Boston to discuss acquiring a restaurant. Later in the month, he travels to New York to discuss acquiring a bakery. Stanford does not acquire the restaurant but does purchase the bakery on November 1, 2018. Stanford incurred the following expenses: Total investigation costs related to the restaurant $28,000 Total investigation costs related to the bakery 51,000 If required, round any division to two decimal places and use in subsequent computation. Round your final answer to the nearest dollar. What is the maximum amount Stanford can deduct in 2018 for investigation expenses?

$4,522 Regarding the bakery, Stanford may immediately deduct $4,000 [$5,000 - ($51,000 - $50,000)] and amortize the balance of $47,000 ($51,000 - $4,000) over a period of 180 months. $47,000/180 months = $261.11 x 2 months = $522.22. For calendar year 2018, Stanford can deduct $4,522.22 ($4,000 + $522.22), rounded to $4,522.

At the beginning of the year, Amber has a carryover of unused losses in the amount of $1,250 under the at-risk rules and $2,000 under the passive loss rules. In the current year, Amber has no gain or loss from her passive activity, however she contributes $6,000 to the passive activity. She has no other passive income for the year. If an amount is zero, enter "0". At year-end, Amber has the following: Adjusted basis and an at-risk amount in the passive activity: $ Suspended losses under the at-risk rules: $ Suspended passive losses: $

$4,750 $0 $3,250

Sebastian purchases two pieces of equipment for $100,000. Appraisals of the equipment indicate that the fair market value of the first piece of equipment is $72,000 and that of the second piece of equipment is $108,000. What is Sebastian's basis in these two assets? Do not round your interim calculations. If required, round your final answers to the nearest dollar. Sebastian's basis for the first piece of equipment is $___ and $___ for the second piece of equipment.

$40,000 = (72,000/180,000) * 100,000 $60,000 = (108,000/180,000) * 100,000

Shelby works for Dynasty Motors, an automobile dealership. All employees can buy a car at the company's cost plus 5%. The company does not charge employees the $400 dealer preparation fee that nonemployees must pay. Shelby purchased an automobile for $22,785 ($21,700 + $1,085). The company's cost was $21,700. The price for a nonemployee would have been $23,100 ($22,700 + $400 preparation fee). How much must Shelby include in his gross income related to the purchase of the car?

$405. The discount on the price of the automobile of $1,085 ($22,785 - $21,700) is a qualified employee discount. However, the discount is taxable to the extent it exceeds the normal gross profit. The gross profit component of the price to customers is $1,000 ($22,700 - $21,700). Therefore, $85 ($1,085 - $1,000) of the discount is included in Shelby's gross income. In addition, the maximum qualified employee discount that can be excluded for a service is 20%. Shelby must include in gross income $320, or 80% of the preparation fee ($400 x 80%). The total taxable amount is $405 ($85 + $320).

Jebali Company reports gross income of $340,000 and other property-related expenses of $229,000 and uses a depletion rate of 14%. Jebali's depletion allowance is $___

$47,600 Gross income $340,000 Less: other expenses (229,000) Taxable income before depletion $111,000 Depletion allowance = $47,600* *[The lesser of $47,600 (14% x $340,000) or $55,500 (50% x $111,000)]

Dexter owns a large tract of land and subdivides it for sale. Assume that Dexter meets all of the requirements of § 1237 and during the tax year sells the first eight lots to eight different buyers for $22,000 each. Dexter's basis in each lot sold is $15,000, and he incurs total selling expenses of $900 on each sale. If an amount is zero, enter "0". Dexter has a realized and recognized gain of $___, of which $___ is classified as ordinary income and $___ as a capital gain.

$48,800 [(22,000 * 8) - (900 * 8) - (15,000 * 8)] $1,600 [5% (22,000 * 8) - (900 * 8)] $47,200 (48,800 - 1,600)

Faye, Gary, and Heidi each have a one-third interest in the capital and profits of the FGH Partnership. Each partner had a capital account of $77,500 at the beginning of the tax year. The partnership profits for the tax year were $1,458,600. Changes in their capital accounts during the tax year were as follows: Faye Gary Heidi Total Beginning $77,500 $77,500 $77,500 $232,500 Withdrawals (20,000) (35,000) (10,000) (65,000) Add. contrib. -0- -0- 5,000 5,000 Profit allocate 486,200 486,200 486,200 1,458,600 End balance 543,700 528,700 558,700 1,631,100 In arriving at the $1,458,600 of partnership profits, the partnership deducted $3,300 ($1,100 for each partner) in premiums paid for group term life insurance on the partners. Faye and Gary are 39 years old, and Heidi is 35 years old. Other employees are also eligible for group term life insurance equal to their annual salary. These premiums of $10,500 have been deducted in calculating the partnership profits of $1,458,600. Compute each partner's gross income from the partnership for the tax year. Each partner's gross income from the partnership for the tax year is $___

$487,300 [($1,458,600 + $3,300)/3]

Marsha is single, had gross income of $50,000, and incurred the following expenses: Charitable contribution $2,000 Taxes and interest on home 7,000 Legal fees incurred in a tax dispute 1,000 Medical expenses 3,000 Penalty on early withdrawal of savings 250 Her AGI is:

$49,750 (50,000 - 250)

Jacob purchased business equipment for $56,000 in 2015 and has taken $35,000 of regular MACRS depreciation. Jacob sells the equipment in 2018 for $26,000. What is the amount and character of Jacob's gain or loss? If an amount is zero, enter "0". Jacob has § 1245 gain of $___ and § 1231 gain of $___.

$5,000 (56,000 - 35,000 - 26,000), $0 (because none of the gain is greater than the depreciation taken).

On April 5, 2018, Kinsey places in service a new automobile that cost $36,000. He does not elect § 179 expensing, and he elects not to take any available additional first-year depreciation. The car is used 70% for business and 30% for personal use in each tax year. Kinsey chooses the MACRS 200% declining-balance method of cost recovery (the auto is a 5-year asset). Click here to access the depreciation table to use for this problem. Assume the following luxury automobile limitations: year 1: $10,000; year 2: $16,000. Compute the total depreciation allowed for: 2018: 2019:

$5,040 = the lesser of ($36,000 x 20% x 70% = 5,040) or (10,000 x 70% = 7,000) $8,064 = the lesser of ($36,000 x 32% x 70% = 8,064) or (16,000 x 70%)

Compute the following regarding business and nonbusiness bad debts. On May 9, 2016, Calvin acquired 250 shares of stock in Aero Corporation, a new startup company, for $68,750. Calvin acquired the stock directly from Aero, and it is classified as § 1244 stock (at the time Calvin acquired his stock, the corporation had $900,000 of paid-in capital). On January 15, 2018, Calvin sold all of his Aero stock for $7,000. Assume that Calvin is single. If an amount is zero, enter "0". As a result of the sale, Calvin has: Ordinary loss: Short-term capital loss: Long-term capital loss:

$50,000 $0 $11,750 Calvin's total loss of $61,750 ($68,750 - $7,000) is treated as follows: $50,000 is ordinary loss and the remaining $11,750 is long-term capital loss.

An employer provides all of his employees with life insurance protection equal to twice the employee's annual salary. Melba, age 42, has an annual salary of $70,000. Is Melba required to recognize income even though she is still alive at the end of the year and thus nothing has been collected on the life insurance policy? If the coverage exceeds $___, the employee must include in gross income the premiums paid on ___. Therefore, Melba includes ___.

$50,000, the protection above this amount, $108 (For Melba's age, 42 years, the premium amount is $.10 per month per $1,000 of insurance coverage for her annual insurance coverage in excess of $50,000 which is $108 [($.10)(12)($90,000/$1,000)].)

Fred, who is self-employed, travels from Denver to Miami primarily on business. He spends five days conducting business and two days sightseeing. His expenses are $555 (airfare), $140 per day (meals), and $190 per night (lodging). If required, round your answers to the nearest dollar. Fred's deductible expenses are: Airfaire: Lodging: Meals:

$555 $950 ($190 x 5 days) $350 ($140 x 5 days x 50%)

Tim, a single taxpayer, operates a business as a single-member LLC. In 2018, his LLC reports business income of $410,500 and business deductions of $718,375, resulting in a loss of $307,875. What are the implications of this business loss? Tim has an excess business loss of $___ Can this business loss be used to offset other income that Tim reports? If so, how much? If not, what happens to the loss? Tim may use $___ of the $307,875 LLC business loss, to offset ___. The excess business loss is ___.

$57,875 $250,000, nonbusiness income, treated as part of Tim's NOL carryforward

Zhou owns a nonrental business with two separate departments. Department A generates net income of $70,000, and Department B generates a net loss of $58,000. Zhou participates 800 hours in the operations of Department A and 300 hours in the operations of Department B. If Zhou is allowed to treat the departments as components of a single activity, calculate the amount of the Department B loss that can be offset against the income from Department A in the current year. $___.

$58,000

Diana acquires, for $65,000, and places in service a 5-year class asset on December 19, 2018. It is the only asset that Diana acquires during 2018. Diana does not elect immediate expensing under § 179. She elects additional first-year deprecation. Click here to access the depreciation table to use for this problem. Diana's total cost recovery deduction for the asset is $___ for 2018.

$65,000

Compute the total Medicare tax for the following taxpayer. If required, round your answer to the nearest dollar. Mario, who is single, earns wages of $384,600 in 2018. His total Medicare tax is $___

$7,238 For tax years beginning after December 31, 2015, an additional .9 percent Medicare tax is imposed on wages received in excess of $250,000 for married taxpayers filing a joint return ($125,000 if married filing separately) and $200,000 for all other taxpayers. Unlike the general 1.45 percent Medicare tax on wages, the additional tax on a joint return is on the combined wages of the employee and the employee's spouse. As a result, the Medicare tax rate will be: 1.45 percent on the first $200,000 of wages ($125,000 on a married filing separate return; $250,000 of combined wages on a married filing joint return), and 2.35 percent (1.45% + .9%) on wages in excess of $200,000 ($125,000 on a married filing separate return; $250,000 of combined wages on a married filing joint return). Employers must withhold the additional .9% Mario will pay $2,900 of Medicare taxes on the first $200,000 ($200,000 x 1.45%) of his wages and $4,338 of Medicare taxes on his wages in excess of $200,000 [($384,600 - $200,000) x 2.35%]. In total, his Medicare tax will be $7,238; his additional Medicare tax is $1,661 [($384,600 - $200,000) x .9%].

On September 30, 2018, Priscilla purchased a business. Of the purchase price, $60,000 is allocated to a patent and $375,000 to goodwill. If required, round your intermediate values to nearest dollar and use in subsequent computations. The 2018 § 197 amortization deduction is $___

$7,250 Patent: $60,000 /15 years = $4,000 x 3/12* = $1,000 Goodwill: $375,000 /15 years = $25,000 x 3/12* = $6,250.

In 2018, Bianca earned a salary of $172,700 from her employer CallMart. How much in FICA and Medicare taxes will CallMart withhold from Bianca's salary? Round your answer to whole dollar amount, if necessary. The FICA taxes withheld from her salary is $___ and the Medicare portion withheld is $___

$7,961 (.062 * $128,400), $2,504 (.0145 * $172,700)

Data, Inc., purchased and placed in service $5,000 of office furniture on August 24, year 3. This is the only asset purchase during the year. Section 179 expensing was not elected. Using the excerpt of the MACRS half-year convention table below, what is the MACRS depreciation in year 3 for the office furniture?

$715 (14.29% x $5,000)

On February 20, 2017, Bill purchased stock in Pink Corporation (the stock is not small business stock) for $1,000. On May 1, 2018, the stock became worthless. During 2018, Bill also had an $8,000 loss on § 1244 small business stock purchased two years ago, a $9,000 loss on a nonbusiness bad debt, and a $5,000 long-term capital gain. How should Bill treat these items on his 2018 tax return?

$8,000 ordinary loss and $3,000 short-term capital loss Ordinary loss (small business stock) ($8,000) Long-term capital gain $5,000 - long-term capital loss (worthless securities) (1,000) Net long-term capital gain $4,000 - short-term capital loss (nonbusiness bad debt) (9,000) Net short-term capital loss ($5,000) Short-term capital loss limited to ($3,000)

Valentino is a patient in a nursing home for 45 days of 2018. While in the nursing home, he incurs total costs of $13,500. Medicare pays $8,000 of the costs. Valentino receives $15,000 from his long-term care insurance policy, which pays while he is in the facility. Assume that the daily federal statutory amount for Valentino is $360. Valentino may exclude $___ of the $15,000 from his gross income.

$8,200 Greater of: Daily statutory amount in 2018 ($360 x 45 days) = $16,200 Actual cost of the care =$13,500 $16,200 Less: Amount received from Medicare (8,000) Equals: Amount of exclusion $8,200

Britt's financial records for the year reflect the following: Interest income from bank savings account: $2,100 Taxable annuity receipts: $8,000 Investment counsel fee: $1,500 Investment interest expense: $7,000. Britt's net investment income is $___ and her investment interest deduction is $___.

$8,600 (2,100 + 8,000 - 1,500) $7,000

For calendar year 2018, Stuart and Pamela Gibson file a joint return reflecting AGI of $320,000. Their itemized deductions are as follows. Note: All expenses are before any applicable limitations, unless otherwise noted. Casualty loss in a Federally declared disaster area after $100 floor (not covered by insurance) = $56,800 Home mortgage interest (loan qualifies as acquisition indebtedness) = 22,720 Credit card interest = 1,136 Property taxes on home = 17,040 Charitable contributions = 31,240 State income tax = 19,880 Tax return preparation fees = 1,704 Round your final answers to the nearest whole dollar. The amount of itemized deductions the Gibsons may claim for the year is $___

$88,760 Casualty loss = [$56,800 - (10% × $320,000)] =$24,800 Home mortgage int = 22,720 State taxes ($19,880 income and $17,040 property; limited to $10,000) = 10,000 Charitable contributions = 31,240 = 88,760

Dolly is a college student who works as a part-time server in a restaurant. Her usual tip is 20% of the price of the meal. A customer ordered a piece of pie and said that he would appreciate prompt service. Dolly abided with the customer's request. The customer's bill was $8, but the customer left a $100 bill on the table and did not ask for a receipt. Dolly gave the cashier $8 and pocketed the $100 bill. Dolly concludes that the customer thought that he had left a $10 bill, although the customer did not return to correct the apparent mistake. The customer had commented about how much he appreciated Dolly's prompt service. Dolly thinks that a $2 tip would be sufficient and that the other $98 is like "found money". How much, if any, should Dolly include in her gross income?

$92

Several of Egret Company's employees have asked the company to create a hiking trail that employees could use during their lunch hours. The company owns vacant land that is being held for future expansion but would have to spend approximately $50,000 if it were to make a trail. Nonemployees would be allowed to use the facility as part of the company's effort to build strong community support. Regarding the following items, help Egret employees by identifying each as either "Yes, a tax issue" or "No, not a tax issue". 1. Is the benefit a no-additional-cost service? 2. Will the benefit impact the employee's flexible spending plan with the employer? 3. Does the walking trail qualify as an excludable "athletic facility"? 4. Does the employee experience an economic benefit from using the facility? 5. Is the benefit considered to be a de minimis fringe benefit?

1. Yes, a tax issue 2. No, not a tax issue 3. Yes, a tax issue 4. Yes, a tax issue 5. Yes, a tax issue

For calendar year 2018, Jean was a self-employed consultant with no employees. She had $80,000 of net profit from consulting and paid $7,000 in medical insurance premiums on her policy covering 2018. How much of these premiums may Jean deduct as a deduction for AGI? As an itemized deduction? If an amount is zero, enter "0". Self-employed persons can deduct ___% of their medical insurance premiums as a deduction for AGI in 2018. Thus, Jean may deduct $___ as a deduction ___ AGI and she may deduct $___ as an itemized deduction (subject to the AGI floor).

100%, $7,000, for, $0

Complete the statement below regarding the amortization period of a § 197 intangible the actual useful life is less than 15 years. The amortization period for a § 197 intangible is ___, beginning ___.

15 years regardless of the actual useful life, in the month acquired

On December 29, 2018, an employee received a $5,000 check from her employer's client. The check was payable to the employer. The employee did not remit the funds to the employer until December 30, 2018. The employer deposited the check on December 31, 2018, but the bank did not credit the employer's bank account until January 2, 2019. Complete the statement below regarding when this company is required to include the $5,000 in gross income. In ___ because a check received __ considered a cash equivalent and a cash basis taxpayer must recognize the income when the check is ___.

2018, is, received

Belinda was involved in a boating accident in 2018. Her speedboat, which was used only for personal use and had a fair market value of $28,000 and an adjusted basis of $14,000, was completely destroyed. She received $10,000 from her insurance company. Her AGI for 2018 is $37,000. Belinda's tentative casualty loss before any limitations is $___. Is her loss currently deductible?

4,000, Only to the extent the loss can offset other personal casualty gains.

A taxpayer, age 64, purchases an annuity from an insurance company for $60,000. She is to receive $500 per month for life. Her life expectancy 20.8 years from the annuity starting date. Assuming that she receives $6,000 this year, what is the exclusion percentage and how much is included in her gross income? Round the exclusion percentage to two decimal places. Round the final answer for the income to the nearest dollar. Exclusion percentage: Included in income:

48.08%, $3,115 (The taxpayer's expected return is $500 x 12 x 20.8 = $124,800.00. The exclusion percentage is 0.4808, rounded to 48.08%, ($60,000 investment/$124,800.00). The expected return is $2,884.80 (48.08% x $6,000 annual payment). The $2,884.80 is a nontaxable return of capital, and $3,115.20 ($6,000 - $2,884.80), rounded to $3,115 is included in gross income.)

Determine the treatment of a loss on rental property under the following facts: Basis $650,000 FMV before the loss 800,000 FMV after the loss 200,000 The amount of the loss is $

600,000 (800,000-200,000) which is less than $650,000

A taxpayer, age 64, purchases an annuity from an insurance company for $50,000. She is to receive $300 per month for life. Her life expectancy 20.8 years from the annuity starting date. Assuming that she receives $3,600 this year, what is the exclusion percentage and how much is included in her gross income? Round the exclusion percentage to two decimal places. Round the final answer for the income to the nearest dollar. Exclusion percentage: Included in income:

66.77% (300 x 12 x 20.8) = 74,880 50,000/74,880 = 66.77% $1,196 (1 - .6677) x (3,600) = $1,196

Complete the statements below regarding why some rental activities may not be treated as such under the passive activity loss rules. One exception is when property is rented for___ days or less. This exception is based on the presumption that a person who rents property for this period is generally ___ to provide significant services to the customer. The fact that a business is not treated as a rental activity due to the exception ___ mean that it is classified as a nonpassive activity. Therefore, if the taxpayer is a material participant, the business is treated as ___; if he is not a material participant, it is treated as a ___ activity.

7, required, does not necessarily, active, passive

Albert had a terminal illness which required almost constant nursing care for the remaining two years of his estimated life, according to his doctor. Albert had a life insurance policy with a face amount of $100,000. Albert had paid $25,000 of premiums on the policy. The insurance company has offered to pay him $80,000 to cancel the policy, although its cash surrender value was only $55,000. Albert accepted the $80,000. Albert used $15,000 to pay his medical expenses. Albert made a miraculous recovery and lived another 20 years. As a result of cashing in the policy:

Albert is not required to recognize any gross income because of his terminal illness.

Donald was killed in an accident while he was on the job in 2018. Darlene, Donald's wife, received several payments as a result of Donald's death. Review the payments below and then enter the amount to be included in Darlene's gross income in the table provided. a) Donald's employer paid Darlene an amount equal to Donald's three months' salary ($42,200), which is what the employer does for all widows and widowers of deceased employees. b) Donald had $6,800 in accrued salary that was paid to Darlene. c) Donald's employer had provided Donald with group term life insurance of $120,000, which was payable to his widow in a lump sum. Premiums on this policy totaling $23,600 had been included in Donald's gross income under § 79. d) Donald had purchased a life insurance policy (premiums totaled $158,000) that paid $313,000 in the event of accidental death. The proceeds were payable to Darlene, who elected to receive installment payments as an annuity of $29,500 each year for a 20-year period. She received her first installment this year.

Amount payable: a) $42,200 b) $6,800 c) $0 d) $13,865* Total = $62,865 *Life insurance proceeds, annuity: ($313,000 a/$590,000 b) × $29,500 = $15,635 $29,500 - $15,635 = $13,865 a= Investment in contract; nontaxable $313,000 insurance proceeds. b= Expected return: $29,500 x 20 years.

An advance payment received in June 2018 by an accrual basis and calendar year taxpayer for services to be provided over a 36-month period can be spread over four tax years.

False

Aram owns a 20% interest in a partnership (not real estate) in which her at-risk amount was $35,000 at the beginning of the year. The partnership borrowed $50,000 on a recourse note and made a $40,000 profit during the year. Her at-risk amount at the end of the year is $43,000.

False

For all of the current year, Randy (a calendar year taxpayer) allowed the Salvation Army to use a building he owns rent-free. The building normally rents for $24,000 a year. Randy will be allowed a charitable contribution deduction this year of $24,000.

False

Tax advantages of being self-employed (rather than being an employee) include:

Both "An office in the home deduction (for AGI) is available" and "Job-related expenses are deductions for AGI" are advantages.

From January through November, Vern participated for 420 hours as a salesman in a partnership in which he owns a 50% interest. The partnership has four full-time employees. During December, Vern spends 110 hours cleaning the store and painting the walls in order to meet the material participation standards. Vern qualifies as a material participant.

False

If insurance proceeds are received for property used in a trade or business, a casualty transaction can result in recognized gain, but cannot result in a recognized loss.

False

In order to dissuade his pastor from resigning and taking a position with a larger church, Michael, an ardent leader of the congregation, gives the pastor a new car. The cost of the car is deductible by Michael as a charitable contribution.

False

In terms of income tax consequences, abandoned spouses are treated the same way as married persons filing separate returns.

False

Calculator Mauve Company permits employees to occasionally use the copying machine for personal purposes. The copying machine is located in the office where the higher paid executives work, so they occasionally use the machine. However, the machine is not convenient for use by the lower paid warehouse employees and, thus, they never use the copier. The use of the copy machine may not be excluded from gross income because the benefit is discriminatory.

False

Certain high-income individuals are subject to three additional Medicare taxes—on wages, unearned income, and tax credits claimed.

False

During the year, Victor spent $300 on bingo games sponsored by his church. If all profits went to the church, Victor has a charitable contribution deduction of $300.

False

For a person who is in the 35% marginal tax bracket, $1,000 of tax-exempt income is equivalent to $1,350 of income that is subject to tax.

False

Adrienne sustained serious facial injuries in a motorcycle accident. To restore her physical appearance, Adrienne had cosmetic surgery. She cannot deduct the cost of this procedure as a medical expense.

False

David, a sole proprietor of a bookstore, pays a $7,500 premium for medical insurance for himself and his family. Joan, an employee of a small firm that doesn't provide her with medical insurance, pays medical insurance premiums of $8,000 for herself. How does the tax treatment differ for David and Joan? ___deducts 100% of the premium as a deduction for AGI, whereas ___ include(s) the premiums in computing the itemized medical expense deduction.

David, Joan

After the automatic mileage rate has been set by the IRS for a year, it cannot later be changed by the IRS.

False

Al, who is single, has a gain of $40,000 on the sale of § 1244 stock (small business stock) and a loss of $80,000 on the sale of § 1244 stock. As a result, Al has a $40,000 ordinary loss.

False

Samuel, who is single, files head of household and has the following items for the current year: Salary $98,000 Loss on sale of § 1244 stock acquired three years ago 36,000 Gain on sale of § 1244 stock acquired one month ago 10,000 Nonbusiness bad debt 30,000 Business bad debt 5,000

Determine his AGI for the current year. $54,000

Swan Finance Company, an accrual method taxpayer, requires all of its customers to carry credit life insurance. If a customer dies, the company receives from the insurance company the balance due on the customer's loan. Ali, a customer, died owing Swan $1,500. The balance due included $200 accrued interest that Swan has included in income. When Swan collects $1,500 from the insurance company, Swan:

Does not recognize income from the life insurance because the entire amount is a recovery of capital.

The employees of Mauve Accounting Services are permitted to use the copy machine for personal purposes, provided the privilege is not abused. Ed is the president of a civic organization and uses the copier to make several copies of the organization's agenda for its meetings. The copies made during the year would have cost $150 at a local office supply.

Ed may exclude the cost of the copies as a de minimis fringe benefit.

A corporation which makes a loan to a shareholder can have a nonbusiness bad debt deduction.

False

A nonbusiness bad debt deduction can be taken any year after the debt becomes totally worthless.

False

A physician recommends a private school for Ellen's dependent child. Because of the physician's recommendation, the cost of the private school will qualify as a medical expense deduction (subject to percentage limitations).

False

A taxpayer may qualify for the credit for child and dependent care expenses if the taxpayer's dependent is under age 17.

False

All exclusions from gross income are reported on Form 1040.

False

Meg's employer carries insurance on its employees that will pay an employee his or her regular salary while the employee is away from work due to illness. The premiums for Meg's coverage were $1,800. Meg was absent from work for two months as a result of a kidney infection. Meg's employer's insurance company paid Meg's regular salary of $8,000 while she was away from work. Meg also collected $2,000 on a wage continuation policy she had purchased. Meg must include $11,800 in her gross income.

False

Mindy paid an appraiser to determine how much a capital improvement made for medical reasons increased the value of her personal residence. The appraisal fee qualifies as a deductible medical expense.

False

One indicator of independent contractor (rather than employee) status is when the individual performing the services is paid based on time spent (rather than on tasks performed).

False

Sherri owns an interest in a business that is not a passive activity and in which she has $20,000 at risk. If the business incurs a loss from operations during the year and her share of the loss is $32,000, this loss will be fully deductible.

False

Susan is a sales representative for a U.S. weapons manufacturer. She makes a $100,000 "grease" payment to a U.S. government official associated with a weapons purchase by the U.S. Army. She makes a similar payment to a Saudi Arabian government official associated with a similar sale. Neither of these payments is deductible by Susan's employer.

False

The amount of partial worthlessness on a nonbusiness bad debt is deducted in the year partial worthlessness is determined.

False

The cost of repairs to damaged property is not an acceptable measure of the loss in value of the property.

False

The credit for child and dependent care expenses is an example of a refundable credit.

False

The education tax credits (i.e., the American Opportunity credit and the lifetime learning credit) are available to help defray the cost of higher education regardless of the income level of the taxpayer.

False

The election to itemize is appropriate when total itemized deductions are less than the standard deduction based on the taxpayer's filing status.

False

The excess business loss rule applies to partnerships and S corporations (rather than partners and shareholders).

False

The value added tax (VAT) has not had wide acceptance in the international community.

False

This year Allison drove 800 miles to volunteer in a project sponsored by a qualified charitable organization in Utah. In addition, she spent $250 for meals while away from home. In total, Allison may take a charitable contribution deduction of $112 (800 miles × $.14) relating to her volunteer work.

False

Under the Federal income tax formula for individuals, a choice must be made between claiming deductions for AGI and itemized deductions.

False

Walt wants to give his daughter $1,800 for Christmas. As an alternative, she suggests that he pay the property taxes on her residence. If Ralph pays the property taxes, he can deduct them.

False

When separate income tax returns are filed by married taxpayers, one spouse cannot claim the other spouse as an exemption.

False

Zork Corporation was very profitable and had accumulated excess cash. The company decided to repurchase some of its bonds that had been issued for $1,000,000. Because of an increase in market interest rates, Zork was able to retire the bonds for $900,000. The company is not required to recognize $100,000 of income from the discharge of its indebtedness but must reduce the basis in its assets.

False

Beginning in 2018, the moving expense deduction has been eliminated for all taxpayers.

False - members of the Armed Forces (or their spouse or dependents) on active duty who move because of a military order that relates to a permanent change of station get the deduction

Under MACRS, if the mid-quarter convention is applicable, all property sold is treated as being sold at the mid-point of the quarter in which it is placed in service.

False, all property sold is treated as being sold at the mid-point of the quarter in which it is sold.

Land improvements are generally not eligible for cost recovery.

False, land improvements are 15-year class property.

Dan, a self-employed individual taxpayer, prepared his own income tax return for the past year and has asked you to check it for accuracy. Your review indicates that Dan failed to claim certain business meals expenses. Assume Dan elects to itemize deductions. How are the business meals expenses reported on a tax return? ___ of the business meals expenses in this situation are deductible ___. Will the correction of this omission affect the amount of medical expenses Dan can deduct?

Fifty percent, for AGI, Yes

In 2018 Todd purchased an annuity for $150,000. The annuity is to pay him $2,500 per month for the rest of his life. His life expectancy is 100 months. Which of the following is correct?

For each $2,500 payment received in the first year, Todd must include $1,000 in gross income. Each payment is in part a recovery of capital and in part income. The recovery of capital portion is $1,500 [($150,000 cost/$250,000 expected return) × $2,500 payment]. The balance of the amount received of $1,000 ($2,500 - $1,500) is income.

For each of the following independent transactions, calculate the recognized gain or loss to the seller and the adjusted basis to the buyer. If an amount is zero, enter "0." a. Kiera sells Parchment, Inc. stock (adjusted basis $18,500) to Phillip, her brother, for its fair market value of $14,400. b. Amos sells land (adjusted basis $77,000) to his nephew, Boyd, for its fair market value of $41,000. c. Susan sells a tax-exempt bond (adjusted basis $30,700) to her wholly owned corporation for its fair market value of $16,100. d. Sinbad sells a business truck (adjusted basis $17,400) that he uses in his sole proprietorship to his cousin, Agnes, for its fair market value of $7,400. e. Martha sells her partnership interest (adjusted basis $158,800) in Pearl Partnership to her adult daughter, Kim, for $245,800.

Gain/Loss, Amount, Buyer's Adjusted Basis a. Recognized loss, $0, $14,400 b. Recognized loss, $36,000, $41,000 c. Recognized loss, $0, $16,100 d. Recognized loss, $10,000, $7,400 e. Recognized gain, $87,000, $245,000

Benita incurred a business expense on December 10, 2018, which she charged on her bank credit card. She paid the credit card statement which included the charge on January 5, 2019. Which of the following is correct?

If Benita is an accrual method taxpayer, she can deduct the expense in 2018

All employees of United Company are covered by a group hospitalization insurance plan, but the employees must pay the premiums ($8,000 for each employee). None of the employees has sufficient medical expenses to deduct the premiums. Instead of giving raises next year, United is considering paying the employee's hospitalization insurance premiums. If the change is made, the employee's after-tax and insurance pay will:

Increase more for the higher income (35% marginal tax bracket) employees.

Which of the following assets would be subject to cost recovery?

Landscaping around the doctor's office.

Bigham Corporation, an accrual basis calendar year taxpayer, sells its services under 12-month and 24-month contracts. The corporation provides services to each customer every month. On July 1, 2018, Bigham sold the following customer contracts: Length of contract Total Proceeds 12 months $28,000 24 months $56,000 Determine the income to be recognized in taxable income in 2018 and 2019.

Length: 12 months 2018 Income: $14,000 = ($28,000 x 6/12) 2019 Income: $14,000 = ($28,000 x 6/12) Length: 24 months 2018 Income: $14,000 = ($56,000 x 6/24) 2019 Income: $42,000 = ($56,000 x 18/24)

Which of the following are deductions for AGI?

Mortgage interest on a building used in a business.

Gordon anticipates that being positively perceived by the individual who is elected mayor will be beneficial for his business. Therefore, he contributes to the campaigns of both the Democratic and the Republican candidates. The Republican candidate is elected mayor. Can Gordon deduct any of the political contributions he made?

No

Clear, Inc., is a bottled water distributor. Clear's delivery trucks frequently are required to park in no-parking zones to make their deliveries. If the trucks are occasionally ticketed, can Clear deduct the fines that it pays?

No, because all fines are not allowed as a deduction.

Stuart, an insurance salesperson, is arrested for allegedly robbing a convenience store. He hires an attorney who is successful in getting the charges dropped. Can Stuart deduct the attorney's fee?

No, because it is personal in nature.

Fred specified in his will that his nephew John should serve as executor of Fred's estate. John received $10,000 for serving as executor. Select a response to the question "Can John exclude the $10,000 from his gross income?"

No, because the $10,000 is treated as compensation income.

If depreciable equipment used in a business is sold at a recognized gain on July 10, 2018, and it was purchased on August 21, 2017, does § 1245 depreciation recapture apply to the asset?

No, because the equipment was held for less than one year.

The value of Terrah's personal residence has declined significantly because of a recent forest fire in the area where she lives. The fire was a Federally declared disaster. Terrah's house suffered no actual damage during the fire, but because much of the surrounding area was destroyed, the value of all of the homes in the area declined substantially. Can Terrah take a casualty loss for the decline in value of her residence caused by the fire? __ because the decline in value of her residence ___ meet the "sudden, unexpected, and unusual, progressive deterioration" requirement for a casualty loss.

No, does not

Andrew owns land (adjusted basis of $40,000) that he uses in his business. He exchanges the land and $20,000 in cash for a different parcel of land worth $50,000. May Andrew avoid like-kind exchange treatment to recognize his realized loss of $10,000? ___, because the §1031 like-kind exchange provision ___.

No, is mandatory

Karla exchanges personal use property for property to be held for productive use in a trade or business. Can this transaction qualify for like-kind exchange treatment? ___. To qualify for like-kind exchange treatment, the property must be held for ___.

No, productive use in a trade or business or be held for investment

Jim discovers that his residence has extensive termite damage. May he take a deduction for the damage to his residence? __, because the current position of ___ is that termite damage ___ deductible as a casualty loss. Generally, ___ qualify as casualties. In addition, under the TCJA of 2017, for 2018 through 2025, personal casualty losses are ___.

No, the IRS, is not, nonsudden events do not, only allowed if the loss occurs in a Federally declared disaster area

The City of Lawrence recently was hit by a tornado, leaving many families in need of food, clothing, shelter, and other necessities. Betty contributed $500 to a family whose home was completely destroyed by the tornado. Jack contributed $700 to the family's church, which gave the money to the family. Assume both taxpayers itemize their deductions. Are the contributions deductible or not deductible? • Betty's contribution • Jack's contribution

Not deductible Deductible

On November 12, 2018, Woody received land and a building from Ruby as a gift. Ruby's adjusted basis and the fair market value at the date of the gift are as follows: Asset Adjusted Basis FMV Land $10,000 $28,000 Building 90,000 110,000 Ruby paid gift tax of $15,000 on the transfer. Round any division to four decimal places. Round your final answers to the nearest dollar. a. Determine Woody's adjusted basis for the land and building. Woody's total basis for the assets received from Ruby is $___ Woody's adjusted basis for the land is $___. Woody's adjusted basis for the building is $___. b. Assume instead that the fair market value of the land was $8,000 and that of the building was $88,000. Determine Woody's adjusted basis for the land and building. Woody's basis for gain: Woody's adjusted basis for the land is $___ Woody's adjusted basis for the building is $___. Woody's basis for loss: Woody's adjusted basis for the land is $___ Woody's adjusted basis for the building is $___.

The formula is: Donor's basis + (unrealized appreciation/taxable gift x gift taxes paid). Note: The taxable gift is the fair market value of the gift less the per donee annual exclusion. a. $104,598 = 10,000 + 90,000 + [(138,000 - 100,000)/(138,000 - 14,000) x $15,000] $21,223 = (28,000/138,000) x 104,598 $83,375 = (110,000/138,000) x 104,598 b. $10,000, $90,000, $8,000, $8,000

Sharon had some insider information about a corporate takeover. She unintentionally informed a friend, who immediately bought the stock in the target corporation. The takeover occurred and the friend made a substantial profit from buying and selling the stock. The friend told Sharon about his stock dealings, and gave her a pearl necklace because she "made it all possible". The necklace was worth $10,000, but she already owned more jewelry than she desired.

The value of the necklace must be included in Sharon's gross income for the tax year it was received by her.

"Other casualty" means casualties similar to those associated with fires, storms, or shipwrecks.

True

A U.S. citizen who works in France from February 1, 2018 until January 31, 2019 is eligible for the foreign earned income exclusion in 2018 and 2019.

True

A bond held by an investor that is uncollectible will be treated as a worthless security and hence, produce a capital loss.

True

A decrease in a taxpayer's AGI could increase the amount of medical expenses that can be deducted.

True

A loss is not allowed for a security that declines in value.

True

A statutory employee is not a common law employee but is subject to Social Security tax.

True

A taxpayer is considered to be a material participant if he or she spends more than 500 hours in the activity.

True

Brooke works part-time as a waitress in a restaurant. For groups of 7 or more customers, the customer is charged 15% of the bill for Brooke's services. For parties of less than 7, the tips are voluntary. Brooke received $11,000 from the groups of 7 or more and $7,000 in voluntary tips from all other customers. Using the customary 15% rate, her voluntary tips would have been only $6,000. Brooke must include $18,000 ($11,000 + $7,000) in gross income.

True

Chad pays the medical expenses of his son, James. James would qualify as Chad's dependent except that he earns $7,500 during the year. Chad may claim James' medical expenses even if he is not a dependent.

True

Dan and Donna are husband and wife and file separate returns for the year. If Dan itemizes his deductions from AGI, Donna cannot claim the standard deduction.

True

Derek, age 46, is a surviving spouse. If he has itemized deductions of $26,250 for 2018, Derek should not claim the standard deduction.

True

Dick participates in an activity for 90 hours during the year. He has no employees and there are no other participants. Dick is a material participant.

True

Ed died while employed by Violet Company. His wife collected $40,000 on a group term life insurance policy that Violet provided its employees, and $6,000 of accrued salary Ed had earned prior to his death. All of the premiums on the group term life insurance policy were excluded from the Ed's gross income. Ed's wife is required to recognize as gross income only the $6,000 she received for the accrued salary.

True

Expenditures made for ordinary repairs and maintenance of property are not added to the original basis in the determination of the property's adjusted basis whereas capital expenditures are added to the original basis.

True

Fees for automobile inspections, automobile titles and registration, bridge and highway tolls, parking meter deposits, and postage are not deductible if incurred for personal reasons, but they are deductible as deductions for AGI if incurred as a business expense by a self-employed taxpayer.

True

For no-additional cost services, qualified employee discounts, and qualified retirement planning services, if the plan is discriminatory in favor of highly compensated employees, exclusion treatment is denied.

True

For personal property placed in service in 2018, the § 179 maximum deduction is limited to $1,000,000.

True

Georgia contributed $2,000 to a qualifying Health Savings Account in the current year. The entire amount qualifies as an expense deductible for AGI.

True

Gray Company, a closely held C corporation, incurs a $50,000 loss on a passive activity during the year. The company has active income of $34,000 and portfolio income of $24,000. If Gray is not a personal service corporation, it may deduct $34,000 of the passive activity loss.

True

If a business debt previously deducted as partially worthless becomes totally worthless this year, only the amount not previously deducted can be deducted this year.

True

If an automobile is placed in service in 2018, the limitation for cost recovery in 2020 will be based on the cost recovery limits for the year 2018.

True

If an individual is subject to the direction or control of another only to the extent of the end result but not as to the means of accomplishment, an employer-employee relationship does not exist.

True

If an owner participates for more than 500 hours in a bicycle rental activity located at a beach resort, any loss from that activity is treated as an active loss that can offset active income.

True

If more than 40% of the value of property, other than real property, is placed in service during the last quarter, all of the property placed in service in the second quarter will be allowed 7.5 months of cost recovery.

True

In 2008, Terry purchased land for $150,000. In 2018, Terry received $10,000 from a local cable television company in exchange for Terry allowing the company to run an underground cable across Terry's property. Terry is not required to recognize income from receiving the $10,000 because it was a return of his capital invested in the land.

True

In 2018, Brandon, age 72, paid $5,000 for long-term care insurance premiums. He may include the $5,000 in computing his medical expense deduction for the year.

True

In 2018, Theresa was in an automobile accident and suffered physical injuries. The accident was caused by Ramon's negligence. In 2019, Theresa collected from his insurance company. She received $15,000 for loss of income, $10,000 for pain and suffering, $50,000 for punitive damages, and $6,000 for medical expenses which she had deducted on her 2018 tax return (the amount in excess of 7.5% of adjusted gross income). As a result of the above, Theresa's 2019 gross income is increased by $56,000.

True

In 2018, personal casualty gains are allowed to offset personal casualty losses. If an excess casualty loss results, it is not deductible (unless attributable to a Federally-declared disaster).

True

In 2019, Rhonda received an insurance reimbursement for medical expenses incurred in 2018. She is not required to include the reimbursement in gross income in 2019 if she claimed the standard deduction in 2018.

True

In a deductible casualty or theft, the basis of property involved is reduced by the amount of insurance proceeds received and by any resulting recognized loss.

True

In choosing between the actual expense method and the automatic mileage method, a taxpayer should consider the cost of insurance on the automobile.

True

In the case of an office in the home deduction, the exclusive business use test does not apply when the home is used as a daycare center.

True

Individual deductions must be properly classified as either for adjusted gross income (AGI) or from AGI (itemized deductions). Business expenses and losses, along with expenses and losses related to rent and royalty activities (entered into for profit), are deducted for AGI. Most other investment expenses are deducted from AGI. Deductible losses on personal use property are deductions from AGI (as itemized deductions).

True

Individuals can deduct from active or portfolio income losses of up to $25,000 from real estate rental activities in which they actively participate.

True

Jessica is a cash basis taxpayer. When Jessica failed to repay a loan, the bank garnished her salary. Each week $60 was withheld from Jessica's salary and paid to the bank. Jessica is required to include the $60 each week in her gross income even though it is the creditor that benefits from the income.

True

Jim's employer pays half of the premiums on a group medical insurance plan covering all employees, and employees pay the other half. Jim can exclude the half of the premium paid by his employer from his gross income and may include the half he pays in determining his medical expense deduction.

True

Judy is a cash basis attorney. This year, she performed services in connection with the formation of a corporation and received stock with a value of $4,000 for her services. By the end of the year, the value of the stock had decreased to $2,000. She continued to hold the stock. Judy must recognize $4,000 of gross income from the stock for the current year.

True

Life Insurance proceeds paid to the beneficiary because of the death of the insured are exempt from income tax.

True

Linda owns investments that produce portfolio income and Activity A that produces losses. From a tax perspective, Linda will be better off if Activity A is not passive.

True

Losses on rental property are classified as deductions for AGI.

True

Nathan owns Activity A, which produces income, and Activity B, which produces passive activity losses. From a tax planning perspective, Nathan will be better off if Activity A is passive.

True

Nonrefundable credits are those that reduce the taxpayer's tax liability but are not paid when the amount of the credit (or credits) exceeds the taxpayer's tax liability.

True

On December 31, Lynette used her credit card to make a $500 contribution to the United Way, a qualified charitable organization. She will pay her credit card balance in January of the following year. If Lynette itemizes, she can deduct the $500 in the year she used the card.

True

Points paid by the owner of a personal residence to refinance an existing mortgage must be capitalized and amortized over the life of the new mortgage.

True

Property used for the production of income is not eligible for § 179 expensing.

True

Qualifying tuition expenses paid from the proceeds of a tax-exempt scholarship do not give rise to an education tax credit.

True

Ralph purchased his first Series EE bond during the year. He paid $709 for a 10-year bond with a $1,000 maturity value. The yield to maturity on the bonds was 3.5%. Ralph is not required to recognize the $291 ($1,000 - $709) original issue discount until the bond matures. However, Ralph can elect to amortize the discount over the ten-year period.

True

Realized gain or loss is measured by the difference between the amount realized from the sale or other disposition of property and the property's adjusted basis at the date of disposition.

True

Sadie mailed a check for $2,200 to a qualified charitable organization on December 31, 2018. The $2,200 contribution is deductible on Sadie's 2018 tax return if she itemizes her deductions.

True

Sam, a single individual, took an itemized deduction of $5,500 for state income tax paid in 2018. His total itemized deductions in 2018 were $18,000. In 2019, he received a $900 refund of his 2018 state income tax. Sam must include the $900 refund in his 2019 Federal gross income in accordance with the tax benefit rule.

True

Sergio was required by the city to pay $2,000 for the cost of new curbing installed by the city in front of his personal residence. The new curbing was installed throughout Sergio's neighborhood as part of a street upgrade project. Sergio may not deduct $2,000 as a tax, but he may add the $2,000 to the basis of his property.

True

Shirley pays FICA (employer's share) on the wages she pays her housekeeper to clean and maintain Shirley's personal residence. The FICA payment is not deductible as an itemized deduction.

True

Taxable income for purposes of § 179 limited expensing is computed by including the MACRS deduction.

True

The additional Medicare taxes assessed on high-income individuals carry differing tax rates depending on the tax base.

True

The basic and additional standard deductions both are subject to an annual adjustment for inflation.

True

The basis of cost recovery property must be reduced by at least the cost recovery allowable.

True

The constructive receipt doctrine requires that income be recognized when it is made available to the cash basis taxpayer, although it has not been actually received. The constructive receipt doctrine does not apply to accrual basis taxpayers.

True

Regarding the computation of cost recovery in the year of sale of an asset using the mid-quarter convention, indicate whether the following statements are "True" or "False". • If the mid-quarter convention applies, the property is treated as though it were disposed of at the midpoint of the quarter. • If the mid-quarter convention applies, no cost recovery deduction is allowed in the year of sale. • If the mid-quarter convention applies, one-quarter of the annual cost recovery deduction is allowed in the year of sale.

True, False, False

Lime Finance Company requires its customers to purchase a credit life insurance policy associated with the loans it makes. Lime is the beneficiary of the policy to the extent of the remaining balance on the loan at the time of the customer's death. In 2017, Lime wrote off as uncollectible a $5,000 account receivable from Wally, which included $1,500 of accrued interest. When Wally died in 2018, the life insurance policy was still in force and Lime received $3,500. Is the $3,500 of life insurance proceeds received by Lime included in its gross income?

Yes, Lime would include $3,500 in gross income in 2018 under the tax benefit rule.

In June 2018, Sue exchanges a sport-utility vehicle (adjusted basis of $16,000; fair market value of $19,500) for cash of $2,000 and a pickup truck (fair market value of $17,500). Both vehicles are for business use. Sue believes that her basis for the truck is $17,500. Is Sue correct? Why or why not? ___. She must treat the transaction as ___. Therefore the basis of the new property is ___ and Sue has a gain recognized of $___.

Yes, a regular sale or exchange, $17,500, $3,500 Amount realized ($17,500 + $2,000) $19,500 Adjusted basis (16,000) Realized and recognized gain $3,500

A retailer's store is destroyed by a tornado but is insured for its replacement cost. Consequently, the retailer has a $40,000 gain after receiving the insurance proceeds. The store is not replaced because the retailer spends the insurance proceeds on additional inventory. What is the nature of the gain if the building originally cost $100,000 three years ago and had an adjusted basis of $82,000 at the time of its destruction? Complete the below e-mail to your supervisor, Sharon Regan, summarizing your answer. Research Question: What is the nature of the gain if the building originally cost $100,000 three years ago and had an adjusted basis of $82,000 at the time of its destruction? Answer: The building, which is used as a retail store, is destroyed, so the gain is ___. Because this is the only property transaction, the net gain becomes ___.

a casualty gain, a § 1231 gain

Lucy sells her partnership interest, a passive activity, with an adjusted basis of $305,000 for $330,000. In addition, she has current and suspended losses of $28,000 associated with the partnership and has no other passive activities. a. Calculate Lucy's total gain and her current deductible loss. Her total gain is $___ and her deductible loss is $___. b. What type of income can the deductible loss offset? Lucy's deductible loss is offset against ___.

a. $25,000 (330,000 - 305,000), $3,000 (28,000 - 25,000) b. any income

a) Dividends are taxable to extent paid out of either current or accumulated earnings and profits (E&P). b) Stock dividends are not taxable - if shareholder has the option to receive stock or cash, the dividend is taxable whether the shareholder receives cash or stock

a) True b) True

Martha was considering starting a new business. During her preliminary investigations related to the new venture, she incurred the following expenditures: Salaries $22,000 Travel 18,000 Professional fees 13,000 Interest on a short-term note 4,000 Martha begins the business on July 1 of the current year. a. Classify for Martha the following expenditures as "Qualifies as a startup costs" or "Does not qualify as a startup cost". 1) Salaries 2) Travel 3) Professional fees 4) Interest on a short-term note b. If Martha elects § 195 treatment, enter the startup expenditure deduction for the current year. In your calculations, round any division to 2 decimal places. Round your final answer to the nearest dollar. Current year startup expenditure equals $___

a. 1) Qualifies as a startup cost 2) Qualifies as a startup cost 3) Qualifies as a startup cost 4) Does not qualify as a startup cost b. $3,700 [5,000 - (53,000-50,000)] + {[(53,000 - 2,000)/180] x 6 months}

a. Classify the following events as either an "Increase" or a "Decrease" in an investor's at-risk amount. 1. The amount of cash and the adjusted basis of property contributed to the activity. 2. Amounts borrowed for use in the activity for which the taxpayer is personally liable or has pledged as security property not used in the activity. 3. The amount of cash and the adjusted basis of property withdrawn from the activity (i.e., withdrawals). 4. Taxpayer's share of amounts borrowed for use in an activity involving the holding of real property that is qualified nonrecourse financing that is secured by real property used in the activity. 5. The taxpayer's share of deductible losses from the activity. 6. The taxpayer's share of any reductions of debt for which recourse against the taxpayer exists, or reductions of qualified nonrecourse debt. 7. Taxpayer's share of the activity's income. b. Assume a taxpayer is facing disallowances because of at-risk limitations. Indicate whether the taxpayer can employ the any of the strategies listed below to increase the at-risk amount in order to claim a higher deduction for losses. Select either "Yes, can be used to increase amount" or "No, it can't be used to increase amount". 1. Invest additional funds or property. 2. Increase recourse or qualified nonrecourse business debt. 3. Decrease recourse or qualified nonrecourse business debt. 4. Decrease his or her share of the activity's income.

a. 1. Increase 2. Increase 3. Decrease 4. Increase 5. Decrease 6. Decrease 7. Increase b. 1. Yes, can use to increase amount 2. Yes, can use to increase amount 3. No, it can't be used to increase amount 4. No, it can't be used to increase amount

Compute 2018 taxable income in each of the following independent situations. Click here to access the standard deduction table to use if required. a. Drew and Meg, ages 40 and 41, respectively, are married and file a joint return. In addition to four dependent children, they have AGI of $125,000 and itemized deductions of $27,000. b. Sybil, age 40, is single and supports her dependent parents, who live with her. Sybil also supports her grandfather, who lives in a nursing home. She has AGI of $80,000 and itemized deductions of $8,000. c. Scott, age 49, is a surviving spouse. His household includes two unmarried stepsons who qualify as his dependents. He has AGI of $75,000 and itemized deductions of $10,100. d. Amelia, age 33, is an abandoned spouse and maintains a household for her three dependent children. She has AGI of $58,000 and itemized deductions of $10,650. e. Dale, age 42, is divorced but maintains the home in which he and his daughter, Jill, live. Jill is single and qualifies as Dale's dependent. Dale has AGI of $64,000 and itemized deductions of $9,900.

a. AGI $125,000 Less: itemized deductions 27,000 Taxable income 98,000 b. AGI $80,000 Less: standard deduction 18,000 Taxable income 62,000 c. AGI $75,000 Less: standard deduction 24,000 Taxable income 51,000 d. AGI $58,000 Less: standard deduction 18,000 Taxable income 40,000 e. AGI $64,000 Less: standard deduction 18,000 Taxable income 46,000

Daniel, age 38, is single and has the following income and expenses in 2018: Salary income $153,000 Net rent income 15,500 Dividend income 900 Payment of alimony in accordance w/separation agreement signed in 2018 16,000 Mortgage interest on residence 7,000 Property tax on residence 1,800 Contribution to traditional IRA (assume the amount is fully deductible) 2,400 Contribution to United Church 1,900 Loss on the sale of real estate (held for investment) 2,950 Medical expenses 2,500 State income tax 2,100 Federal income tax 5,300 Daniel's standard deduction for 2018 is 12,000. a. Classify the following expenses as either "Deductible for AGI", "Deductible from AGI", or "Not deductible". b. Daniel's gross income is $___ and his AGI income is $___. c. Calculate Daniel's total itemized deductions (after any limitations).

a. Payment of alimony Deductible for AGI Mortgage interest Deductible from AGI Property tax on Deductible from AGI Contribution to IRA Deductible for AGI Contribution to Church Deductible from AGI Loss on sale of estate Deductible for AGI Medical expenses Deductible from AGI State income tax Deductible from AGI Federal income tax Not deductible b. $169,400, $148,050 c. 15,300

A warehouse owned by Martha and used in her business (i.e., to store inventory) is being condemned by the city to provide a right-of-way for a highway. The warehouse has appreciated by $180,000 based on Martha's estimate of its fair market value. In the negotiations, the city is offering $35,000 less than what Martha believes the property is worth. Alan, a real estate broker, has offered to purchase Martha's property for $20,000 more than the city's offer. Martha plans to invest the proceeds she will receive in an office building she will lease to various tenants. a. Help Martha by classifying each of the following as a "Relevant tax issue" or "Not a tax issue". • Can Martha defer the realized gain or loss on the disposition of the warehouse? • Can the transaction be structured so it will qualify as an involuntary conversion? • Will replacing the warehouse with the office building qualify as like-kind property under § 1031? • Has Martha had other property condemned within the last 12 months? b. Would the answer above change if Martha's warehouse was property held for investment rather than being used in her business?

a. Relevant tax issue Relevant tax issue Relevant tax issue Not a tax issue b. No

The following information applies to Emily's 2018 taxable income. Her filing status is single. Salary: $85,000 Interest income from bonds issued by Xerox: 1,100 Alimony payments received (divorce finalized in 2014): 6,000 Contribution to traditional IRA: 5,500 Gift from parents: 25,000 Capital gain from stock investment, held for 7 months: 2,000 Amount lost in football office betting pool: 500 Age 40 Emily has no gambling winnings this year. a. Indicate whether the following items are taxable or nontaxable to Emily. b. Identify whether the items are deductible (fully or partially) by Emily. c. Emily's taxable income in 2018 is

a. Salary: Taxable Gift from parents: Not taxable Alimony payments received: Taxable Capital gain from stock investment, held for 7 months: Taxable Interest income from bonds issued by Xerox: Taxable b. Amount lost in football office betting pool: Not deductible Contribution to a traditional IRA: Deductible c. $76,600 ($85,000 + 1,100 + 6,000 + 2,000) - 5,500 - 12,000 (standard deduction)

On June 5, 2018, Brown, Inc., a calendar year taxpayer, receives cash of $750,000 from the county upon condemnation of its warehouse building (adjusted basis of $500,000 and fair market value of $750,000). a. Select either "True" or "False" for each of the following regarding what must Brown do to qualify for § 1033 postponement of gain treatment. • Elect § 1033 postponement of gain treatment • Acquire qualifying replacement property costing at least $750,000 • Acquire the qualifying property no later than December 31, 2020 • Acquire qualifying replacement property costing at least $500,000 b. Complete the statements below regarding what must Brown do to qualify for § 1033 postponement if the adjusted basis was instead $795,000. Because Brown has a realized ___ on the condemnation of business property, ___. In this case, §1033 ___ the normal rules of recognition. Therefore, Brown has a realized ___ of $___ of which ___ is recognized.

a. True True False False b. loss, it is automatically recognized, does not modify, loss, $45,000, $45,000 (795,000 - 750,000)

a. Indicate how passive activity is defined in the Code by selecting either "Yes" or "No" for each statement below regarding the definition of passive activity. • Any trade or business or income-producing activity in which the taxpayer does not materially participate. • Subject to certain exceptions, all rental activities, whether the taxpayer materially participates or not. • Any trade, business, or income-producing activity in which the taxpayer materially participates. • Without exception, all rental activities. b. What aspects of the definition have been clarified by final or Temporary Regulations?

a. Yes Yes No No b. Definition of rental activities and passive activity loss activities; Guidelines for material participation

Melaney has had a bad year with her investments. She lent a friend $8,000; the friend did not repay the loan when it was due and then declared bankruptcy. The loan is totally uncollectible. Melaney also was notified by her broker that the Oak corporate bonds she owned became worthless on December 31, 2018. She had purchased the bonds for $22,000 on November 10, 2017. Melaney also had a $60,000 loss on the disposition of § 1244 corporate stock that she purchased several years ago. Melaney is single. a. What are the nature and amount of Melaney's losses? a. Uncollectible loan to friend b. Worthless Oak corporate bonds c. Loss on § 1244 corporate stock b. What is Melaney's AGI for 2018 assuming that she has $65,000 of ordinary gross income from sources other than those discussed? c. What are the nature and amount of Melaney's loss carryforwards? She has $___ in short-term capital loss carryforward and $___ in long-term capital loss carryforward.

a. a. Short-term capital loss, $8,000 b. Long-term capital loss, $22,000 c. Part ordinary loss and part capital loss, $60,000 b. $12,000 (65,000 - (3,000 short-term capital loss dedution + $50,000 stock deduction)) c. $5,000 (8,000 - 3,000), $32,000 (82,000 - 50,000)

During the year, Eugene had the four property transactions summarized below. Eugene is a collector of antique glassware and occasionally sells a piece to get funds to buy another. Antique vase 06/18/07-05/23/18 $37,000-$42,000 Blue Growth Fund 12/23/09-11/22/18 $22,000-38,000 (100 shares) Orange bonds 02/12/10-04/11/18 $34,000-42,000* Green stock 02/14/18-11/23/18 $11,000-13,000 * The sales price included $750 of accrued interest. a. What are the amount and nature of the gain or loss from each of these transactions? b. How is the accrued interest on the Orange bonds treated by Eugene for tax purposes? The amount is reported as ___.

a. Antique vase - long term capital gain of $5,000 Blue Growth Fund - long-term capital gain of $16,000 Orange bonds - Long-term capital gain of $7,250 Green stock - short-term capital gain of $2,000 b. interest income

Compute Aiden's 2018 taxable income on the basis of the following information. Aiden is married but has not seen or heard from his wife for over three years. Salary $80,000 Interest on bonds issued by City of Boston 3,000 Interest on CD issued by Wells Fargo Bank 2,000 Cash dividend received on Chevron common stock 2,200 Life insurance proceeds paid due to the death of Aunt Margie (Aiden was the designated beneficiary of the policy) 200,000 Inheritance received on death of Aunt Margie 100,000 Jackson (a cousin) repaid a loan Aiden made to him in 2010 (no interest was provided for) 5,000 Itemized deductions (state income tax, property taxes on residence, interest on home mortgage, charitable contributions) 9,700 Number of dependents (children, ages 17 and 18; mother-in-law, age 60) 3 Age 43 a. Indicate whether the items are taxable or not taxable to Aiden. b. What is Aiden's filing status? c. Should Aiden itemize his deductions or take the standard deduction?

a. Cash dividend received on Chevron common stock: Taxable Salary: Taxable Interest on bonds issued by City of Boston: Not taxable Life insurance proceeds: Not taxable Interest on CD issued by Wells Fargo Bank: Taxable Inheritance received on death of Aunt Margie: Not taxable Proceeds from repayment of a loan: Not taxable b. Head of household c. He should take the standard deduction of $18,000 (because it is greater than the itemized deductions of $9,700)

During the year, Shanta, a single individual, recorded the following transactions involving capital assets. a. Indicate the tax treatment for each item. Overall, Shanta has ___ of __. However, ___ of this gain is from collectibles, which are taxed at a maximum rate of __. b. If Shanta is in the 32% bracket, how much income tax results? If Shantan is in the 12% bracket, how much income tax results?

a. Gain on the sale of a coin collection (held as an investment for 10 years) --> $8,000 --> Long-term capital gain Gain on the sale of corporate stock (purchased two years ago as an investment) --> 1,000 --> Long-term capital gain Loss on the sale of sail boat held as an investment (acquired eight months ago) --> (5,000) --> Short-term capital loss a long-term capital gain, $4,000, $3,000, 28% b. $990, $360 (If Shanta is in the 32% bracket, her tax liability is $990 [($3,000 × 28%) + ($1,000 × 15%)]. If Shanta is in the 12% bracket, her tax liability is $360 [($3,000 × 12%) + ($1,000 × 0%)]. Note that the gain from collectibles is taxed at 12% (an amount equal to her tax bracket) and the gain from the stock is taxed at the preferential rate of 0%. The 28% is only used when the regular tax rate exceeds it.)

On July 16, 2018, Logan acquires land and a building for $500,000 to use in his sole proprietorship. Of the purchase price, $400,000 is allocated to the building, and $100,000 is allocated to the land. Cost recovery of $4,708 is deducted in 2018 for the building (nonresidential real estate). a. What is the adjusted basis for the land and the building at the acquisition date? b. What is the adjusted basis for the land and the building at the end of 2018?

a. Land: $100,000 Building: $400,000 b. Land: $100,000 Building: $395,292 (400,000 - 4,708)

Jarret owns City of Charleston bonds with an adjusted basis of $190,000. During the year, he receives interest payments of $3,800. Jarret partially financed the purchase of the bonds by borrowing $100,000 at 5% interest. Jarret's interest payments on the loan this year are $4,900, and his principal payments are $1,100. a. How much reportable interest income does Jarret have this year? b. How much can Jarret deduct as interest expense this year?

a. $0 b. $0

Samantha, who is single and has MAGI of $28,000, was recently employed by an accounting firm. During the year, she spends $2,500 for a CPA exam review course and begins working on a law degree in night school. Her law school expenses were $4,200 for tuition and $450 for books (which are not a requirement for enrollment in the course). Click here to access Exhibit 9.1. If an amount is zero, enter "0". Assuming no reimbursement, how much can Samantha deduct for the: a. CPA exam review course? b. Law school expenses?

a. $0 b. $4,000

Rhonda has an adjusted basis and an at-risk amount of $7,500 in a passive activity at the beginning of the year. She also has a suspended passive activity loss of $1,500 carried over from the prior year. During the current year, she has a loss of $12,000 from the passive activity. Rhonda has no passive activity income from other sources this year. Determine the following items relating to Rhonda's passive activity as of the end of the year. If an amount is zero, enter "0". At year-end, Rhonda has the following: a. Adjusted basis in the passive activity: $___ b. Loss suspended under the at-risk rules: $___ c. Suspended passive activity loss: $___

a. $0 b. $4,500 (12,000 - 7,500) c. $9,000 (7,500 + 1,500)

Casper and Cecile divorced in 2018. As part of the divorce settlement, Casper transferred stock to Cecile. Casper purchased the stock for $182,500, and it had a market value of $292,000 on the date of the transfer. Cecile sold the stock for $255,500 a month after receiving it. In addition Casper is required to pay Cecile $9,125 a month in alimony. He made five payments to her during the year. What are the tax consequences for Casper and Cecile regarding these transactions? If an amount is zero, enter "$0". a. How much gain or loss does Casper recognize on the transfer of the stock? b. Does Casper receive a deduction for the $45,625 alimony paid? c. How much income does Cecile have from the $45,625 alimony received? d. When Cecile sells the stock, how much does she report?

a. $0 b. Yes c. $45,625 d. Cecile will report a gain of $73,000 (255,500 - 182,500)

In a § 1031 like-kind exchange, Rafael exchanges a business building that originally cost $200,000. On the date of the exchange, the building given up has an adjusted basis of $85,000 and a fair market value of $110,000. Rafael pays $15,000 and receives a building with a fair market value of $125,000. Compute the following. If an amount is zero, enter "0". a. Rafael's realized gain on the exchange is $___. b. Rafael's recognized § 1031 gain is $___. c. Rafael's unrecaptured § 1250 gain of $___ is ___.

a. $25,000 (110,000 - 85,000) b. $0 (because no boot received) c. $115,000, carried over to the replacement property (200,000 - 85,000)

Casper and Cecile divorced in 2018. As part of the divorce settlement, Casper transferred stock to Cecile. Casper purchased the stock for $25,000, and it had a market value of $43,000 on the date of the transfer. Cecile sold the stock for $40,000 a month after receiving it. In addition Casper is required to pay Cecile $1,500 a month in alimony. He made five payments to her during the year. What are the tax consequences for Casper and Cecile regarding these transactions? If an amount is zero, enter "$0". a. How much gain or loss does Casper recognize on the transfer of the stock? b. Does Casper receive a deduction for the $7,500 alimony paid? c. How much income does Cecile have from the $7,500 alimony received? d. When Cecile sells the stock, how much does she report? Cecile will report a ___ of $___.

a. $0 b. Yes c. $7,500 d. gain, $15,000

Brenda, who is self-employed, travels from Chicago to Barcelona (Spain) on business. She is gone 10 days (including 2 days of travel) during which time she spends 5 days conducting business and 3 days sightseeing. Her expenses are $1,500 (airfare), $200 per day (meals), and $400 per night (lodging). Because Brenda stayed with relatives while sightseeing, she only paid for 5 nights of lodging. What is Brenda's deduction for: If required, round your answers to the nearest dollar. a. Airfare? b. Meals? c. Lodging?

a. $1,050 ($1500 * 7/10) b. $700 ($200 x 7 x 50%) c. $2,000 ($400 x 5)

Cindy maintains an office in her home that comprises 8% (200 square feet) of total floor space. Gross income for her business is $42,000, and her residence expenses are as follows: Real property taxes $2,400 Interest on mortgage 4,000 Operating expenses 2,200 Depreciation (based on 8% business use) 450 What is Cindy's office in the home deduction based on: a. The Regular Method? b. The Simplified Method?

a. $1,138 [(8% x $2,400) + (8% x $4,000) + (8% x $2,200) + 450] b. $1,000 ($5 per foot x 200 square feet)

On January 1, 2018, Kunto, a cash basis taxpayer, pays $46,228 for a 24-month certificate. The certificate is priced to yield 4% (the effective interest rate) with interest compounded annually. No interest is paid until maturity, when Kunto receives $50,000. In your computations, round any amounts to the nearest dollar. a. Compute Kunto's gross income from the certificate for 2018. b. Compute Kunto's gross income from the certificate for 2019.

a. $1,849 b. $1,923

Printer Company pays a $25,000 annual membership fee to a trade association for paper wholesalers. The trade association estimates that 60% of its dues are allocated to lobbying activities. If amount is zero, enter, "0". a. Printer's total deductible expenses for tax purposes is ___ b. Assume the same facts as above, except that the $25,000 was incurred for in-house lobbying expenses. Printer's total deductible expense for tax purposes is ___

a. $10,000 ($25,000 x 40%) b. $0

Determine the taxable amount of social security benefits for the following situations. If an amount is zero, enter "0". a. Erwin and Eleanor are married and file a joint tax return. They have adjusted gross income of $46,000, no tax-exempt interest, and $12,400 of Social Security benefits. As a result, $___ of the Social Security benefits are taxable. b. Assume Erwin and Eleanor have adjusted gross income of $12,000, no tax-exempt interest, and $16,000 of Social Security benefits. As a result, $___ of the Social Security benefits are taxable. c. Assume Erwin and Eleanor have adjusted gross income of $85,000, no tax-exempt interest, and $15,000 of Social Security benefits. As a result, $___ of the Social Security benefits are taxable.

a. $10,540 b. $0 c. $12,750 (Chapter 4, Pages 4-28-29)

Determine the taxable amount of social security benefits for the following situations. If required, round your answers to the nearest dollar. If an amount is zero, enter "0". a. Erwin and Eleanor are married and file a joint tax return. They have adjusted gross income of $42,600, no tax-exempt interest, and $14,910 of Social Security benefits. As a result, $___ of the Social Security benefits are taxable. b. Assume Erwin and Eleanor have adjusted gross income of $18,200, no tax-exempt interest, and $20,020 of Social Security benefits. As a result, $___ of the Social Security benefits are taxable. c. Assume Erwin and Eleanor have adjusted gross income of $94,000, no tax-exempt interest, and $14,100 of Social Security benefits. As a result, $___ of the Social Security benefits are taxable.

a. $11,147 b. $0 c. $11,985

Sarah has investments in four passive activity partnerships purchased several years ago. Last year the income and losses were as follows: Activity Income (Loss) A $30,000 B (30,000) C (15,000) D (5,000) In the current year, she sold her interest in Activity D for a $10,000 gain. Activity D, which had been profitable until last year, had a current loss of $1,500. Answer the following questions to determine how the sale of Activity D affects Sarah's taxable income in the current year. a. The amount of suspended losses carried forward to the year of the sale is $___ b. What amount of the suspended losses is allocated to Activity D? $___ c. How much, if any, of this net gain may be used to absorb passive activity losses from other activities? $___

a. $20,000 (30,000 - 30,000 - 15,000 - 5,000) b. $2,000 [20,000 x (5,000/50,000)] c. $6,500 (10,000 - 2,000 - 1,500)

Under the terms of a divorce agreement entered into in 2017, Lanny was to pay his wife Joyce $2,000 per month in alimony and $500 per month in child support. For a twelve-month period, Lanny can deduct from gross income (and Joyce must include in gross income): a. $24,000. b. $30,000. c. $0. d. $6,000. e. None of these choices are correct.

a. $24,000. ($2,000 * 12) The $500 per month for child support is not deductible by Lanny.

Camilo's property, with an adjusted basis of $155,000, is condemned by the state. Camilo receives property with a fair market value of $180,000 as compensation for the property taken. If an amount is zero, enter "0". a. Camilo's realized gain is $___ and his recognized gain is $___. b. The basis of the replacement property is $___.

a. $25,000 (180,000 - 155,000), $0 b. $155,000

Lopez acquired a building on June 1, 2013, for $1,000,000. Compute the depreciation deduction assuming the building is classified as (a) residential and (b) non residential. Click here to access the depreciation table to use for this problem. If required, round your answers to the nearest dollar. a. If the building is classified as residential rental real estate, Lopez's cost recovery deduction is $___ for 2018. b. If the building is classified as nonresidential real estate, Lopez's cost recovery deduction is $___ for 2018.

a. $36,360 b. $25,640 a. Residential rental real estate: $1,000,000 x .03636 (Exhibit 8.8) = $36,360 b. Nonresidential rental real estate: $1,000,000 x .02564 (Exhibit 8.8) = $25,640

Al is a medical doctor who conducts his practice as a sole proprietor. During 2018, he received cash of $398,800 for medical services. Of the amount collected, $32,400 was for services provided in 2017. At the end of 2018, Al had accounts receivable of $94,800, all for services rendered in 2018. In addition, at the end of the year, Al received $9,500 as an advance payment from a health maintenance organization (HMO) for services to be rendered in 2019. Compute Al's gross income for 2018: a. Using the cash basis of accounting. b. Using the accrual basis of accounting. c. Advise Al on which method of accounting he should use. Al should use the ___ of accounting so that he will not have to pay income taxes on the ___.

a. $408,300 (398,800 + 9,500) b. $461,200 [(398,800 + 9,500) - 9,500 - 32,400 + 94,800] c. cash method, uncollected accounts receivable

Determine the taxable amount of social security benefits for the following situations. If an amount is zero, enter "$0". a. Erwin and Eleanor are married and file a joint tax return. They have adjusted gross income of $36,000, no tax-exempt interest, and $12,400 of Social Security benefits. As a result, $___ of the Social Security benefits are taxable. b. Assume Erwin and Eleanor have adjusted gross income of $12,000, no tax-exempt interest, and $16,000 of Social Security benefits. As a result, $___ of the Social Security benefits are taxable. c. Assume Erwin and Eleanor have adjusted gross income of $85,000, no tax-exempt interest, and $15,000 of Social Security benefits. As a result, $___ of the Social Security benefits are taxable.

a. $5,100 b. $0 c. $12,750 Chapter 4 page 28-29

McKenzie purchased qualifying equipment for his business that cost $212,000 in 2018. The taxable income of the business for the year is $5,600 before consideration of any § 179 deduction. If an amount is zero, enter "0". a. McKenzie's § 179 expense deduction is $___ for 2018. His § 179 carryover to 2019 is $___. b. How would your answer change if McKenzie decided to use additional first-year (bonus) depreciation on the equipment? Hint: See Concept Summary 8.5. McKenzie's § 179 expense deduction is $___ for 2018. His § 179 carryover to 2019 is $___.

a. $5,600, $206,400 b. $212,000, $0

On January 1, 2018, Kunto, a cash basis taxpayer, pays $142,382 for a 24-month certificate. The certificate is priced to yield 4% (the effective interest rate) with interest compounded annually. No interest is paid until maturity, when Kunto receives $154,000. In your computations, round any amounts to the nearest dollar. a. Compute Kunto's gross income from the certificate for 2018. b. Compute Kunto's gross income from the certificate for 2019.

a. $5,695 b. $5,923

Juliana purchased land three years ago for $50,000. She gave the land to Tom, her brother, in the current year, when the fair market value was $70,000. No gift tax is paid on the transfer. Tom subsequently sells the property for $63,000. a. Tom's basis in the land is ___ and he has a realized ___ of $___. b. Assume instead that the land has a fair market value of $45,000 and that Tom sold the land for $43,000. Tom's basis in the land is $___ and he has a realized ___ of $___.

a. $50,000, gain, $13,000 b. $45,000, loss, $2,000

Falcon, Inc., paid salaries of $500,000 to its employees during its first year of operations. At the end of the year, Falcon had additional unpaid salaries of $45,000. a. Calculate the salary deduction if Falcon is a cash basis taxpayer. b. Calculate the salary deduction if Falcon is an accrual basis taxpayer

a. $500,000 b. 545,000

Xavier Corporation begins business on March 1, 2018. The corporation incurs start-up expenditures of $38,000. Round your final answers to the nearest dollar. a. If Xavier elects amortization under § 195, the total start-up expenditures that Xavier may deduct in 2018 is $___ b. Assume the same facts except the start-up costs totaled $52,000. The total start-up expenditures that Xavier may deduct in 2018 is $___

a. $6,833 = 5,000 + {[(38,000-5,000)/180] x 10 months} b. $5,722 = [5,000 - (52,000 - 50,000)] + {[(52,000 - 3,000)/180] x 10 months}

Henrietta, the owner of a very successful hotel chain in the Southeast, is exploring the possibility of expanding the chain into a city in the Northeast. She incurs $63,500 of expenses associated with this investigation. Based on the regulatory environment for hotels in the city, she decides not to expand. During the year, she also investigates opening a restaurant that will be part of a national restaurant chain. Her expenses for this are $54,800. The restaurant begins operations on May 1. Determine the amount that Henrietta can deduct in the current year for investigating these two businesses. In your computations, round the per-month amount to the nearest dollar and use rounded amount in subsequent computations. If required, round your final answers to the nearest dollar. a. The investigation expenses related to expansion of her hotel chain into another city: $___ b. The investigation expenses related to opening a restaurant: $___

a. $63,500 b. $2,627 [5,000 - (54,800-50,000)] + {[(54,800 - 200)/180] * 8 months}

During 2018, Enrique had the following transactions: Salary $70,000 Interest income on Xerox bonds 2,000 Inheritance from uncle 40,000 Contribution to traditional IRA 5,500 Capital losses 2,500 Enrique's AGI is: a. $64,000. b. $62,000. c. $104,000. d. $67,000. e. $102,000.

a. $64,000. $70,000 (salary) + $2,000 (interest) - $5,500 (IRA contribution) - $2,500 (capital losses) = $64,000. The inheritance is a nontaxable exclusion. The capital losses are deductible.

Wes acquired a mineral interest during the year for $10,000,000. A geological survey estimated that 250,000 tons of the mineral remained in the deposit. During the year, 80,000 tons were mined, and 45,000 tons were sold for $12,000,000. Other related expenses amounted to $5,000,000. Assume the mineral depletion rate is 22%. a. What is the taxable income before the deduction for depletion? b. Under cost depletion, what is the amount of the deduction? c. Under percentage depletion, what is the amount of the deduction? d. Wes's lowest taxable income after the depletion deduction is ___

a. $7,000,000 b. $1,800,000 [(10,000,000/250,000)*45,000] c. 2,640,000 = lesser of (7,000,000 * 50%) or (12,000,000 * 22%) d. 4,360,000

Jackson, who is self-employed, uses his automobile 90% for business and during 2018 drove a total of 14,000 business miles. Information regarding his car expenses is listed below. Business parking $ 140 Auto insurance 1,300 Auto club dues (includes towing service) 180 Toll road charges (business-related) 200 Oil changes & engine tune-ups 210 Repairs 160 Depreciation allowable 2,850 Fines for traffic violations (incurred during business use) 320 Gasoline purchases 2,800 What is Jackson's deduction in 2018 for the use of his car if he uses: a. The actual cost method? b. The automatic mileage method? c. What records must Jackson maintain? Keeping a written or electronic log of miles driven, the dates the automobile was used, the location of travel, and the business purpose is enough evidence for the ___. If the ___is used, keeping copies of receipts, canceled checks, and bills in addition to a mileage log is sufficient. Records and logs should be kept contemporaneously (e.g., updated weekly or daily).

a. $7,090 [140 + 200 + 90%(1,300 + 180 + 210 + 160 + 2,850 + 2800)] b. $7,970 [(54.5 cents x 14,000 miles) + (140 + 200)] c. standard rate method, actual expense method

Lara uses the standard mileage method for determining auto expenses. During 2018, she used her car as follows: 13,800 miles for business, 2,760 miles for personal use, 4,140 miles for a move to a new job, 1,380 miles for charitable purposes, and 690 miles for medical visits. Presuming that all the mileage expenses are allowable (i.e., not subject to percentage limitations), what is Lara's deduction for: If required, round your answers to the nearest dollar. a. Business? b. Charitable? c. Medical?

a. $7,521 (54.5 cents for business miles in 2018 x 13,800 miles) b. $193 (14 cents for charitable miles x 1,380 miles) c. $124 (18 cents for medical miles x 690 miles)

Bianca, who is self-employed, uses her automobile 70% for business and during 2018 drove a total of 23,000 business miles. Information regarding her car expenses is listed below. Business parking $165 Auto insurance 2,900 Parking tickets 50 New tires 200 Oil changes 355 Minor Repairs 185 Depreciation allowable 3,250 Gasoline purchases 3,900 What is Bianca's deduction in 2018 for the use of her car if she uses: a. The actual cost method? b. The automatic mileage method?

a. $7,718 [165 + 70%(2,900 + 200 + 355 + 185 + 3,250 + 3,900)] b. $12,700 [(23,000 * 54.5 cents) + 165]

Noah Yobs, who has $98,200 of AGI (solely from wages) before considering rental activities, has $88,380 of losses from a real estate rental activity in which he actively participates. He also actively participates in another real estate rental activity from which he has $49,100 of income. He has other passive activity income of $31,424. a. What amount of rental loss can Noah use to offset active or portfolio income in the current year?

a. $7,856 [(49,100 - 88,380) = 39,280 - 31,424] = 7,856

In the current year, Roger pays a $4,300 premium for high-deductible medical insurance for him and his family. In addition, he contributed $3,655 to a Health Savings Account. a. How much may Roger deduct if he is self-employed and earned $86,800? Is the deduction for AGI or from AGI? If Roger is self-employed, he may deduct $___ ___AGI. b. How much may Roger deduct if he is an employee (disregarding 7.5% AGI floor implications)? If Roger is an employee, he may deduct $___ for AGI and $___ from AGI if he ___.

a. $7,955 (4,300 + 3,655), for b. $3,655, $4,300, itemizes his deductions

For the current year, David has wages of $80,000 and the following property transactions: Stock investment sales— Long-term capital gain $ 9,000 Short-term capital loss (12,000) Loss on sale of camper (purchased 4 years ago and used for family vacations) (2,000) What is David's AGI for the current year? a. $77,000. b. $89,000. c. $76,000. d. $78,000. e. None of these choices are correct.

a. $77,000. (9,000 - 12,000) = -3,000 + 80,000 = $77,000

Pierre, a cash basis, unmarried taxpayer, had $4,190 of state income tax withheld during 2018. Also in 2018, Pierre paid $1,048 that was due when he filed his 2017 state income tax return and made estimated payments of $2,933 towards his 2018 state income tax liability. When Pierre files his 2018 Federal income tax return in April 2019, he elects to itemize deductions, which amount to $14,750, including the state income tax payments and withholdings, all of which reduce his taxable income. As a result of overpaying his 2018 state income tax, Pierre receives a refund of $1,676 in 2019. The standard deduction for single taxpayers for 2018 was $12,000. a. What is Pierre's 2018 state income tax deduction? b. How much of the $1,676 will Pierre include in his 2019 gross income?

a. $8,171 (4,190 + 1,048 + 2,933) b. $1,676

Maria meets all of the requirements of § 1237 (subdivided realty). In 2018, she begins selling lots and sells four separate lots to four different purchasers. She also sells two contiguous lots to another purchaser. The sales price of each lot is $30,000. Maria's basis for each lot is $15,000. Selling expenses are $500 per lot. a. What are the realized and recognized gain? The realized gain is $___ and recognized gain is $___. b. What is the nature of the gain (i.e., ordinary income or capital gain)? The gain is considered ___ gain. c. Assume instead that the lots sold to the fifth purchaser were not contiguous. If the lots sold to the fifth purchaser were not contiguous, Maria would have $___ ordinary income, and the $___ gain would be long-term capital gain.

a. $87,000, $87,000 (6*30,000) - [(6*500) + (6*15,000)] b. a long-term capital c. $6,000 [5% *(30,000 * 6) - (500 * 6)], $81,000 (87,000 - 6,000)

A number of years ago, Kay acquired an interest in a partnership in which she is not a material participant. Kay's basis in her partnership interest at the beginning of 2017 is $40,000. Kay's share of the partnership loss is $35,000 in 2017, while her share of the partnership income is $15,000 in 2018. How much may Kay deduct in 2017 and 2018, assuming that she owns no other passive activities? If an amount is zero, enter "0". a. In 2017, Kay can deduct $___ of the passive activity losses. b. In 2018, Kay can deduct $___ of the passive activity losses. c. How much, if any, of the 2017 passive activity loss remains suspended at the end of 2018? $___.

a. 0 b. $15,000 c. $20,000

For the following assets, indicate the cost recovery periods under MACRS. a. Land improvements. b. Water utilities. c. Single-purpose agricultural or horticultural structures. d. Office furniture, fixtures, and equipment. e. Computers and peripheral equipment. f. Any horse that is not a racehorse and is more than 12 years old at the time it is placed in service.

a. 15 b. 20 c. 10 d. 7 e. 5 f. 3

Shanna, a calendar year and cash basis taxpayer, rents property from Janice. As part of the rental agreement, Shanna pays $8,400 rent on April 1, 2018 for the 12 months ending March 31, 2019. If required, round any division to two decimal places. Round your final answer to the nearest dollar. a. Shanna's deduction for rent expense in 2018 is $___ b. Assume the same facts, except that the $8,400 is for 24 months' rent ending March 31, 2020. Shanna's deduction for rent expense in 2018 is $___

a. 8,400 - does not extend beyond the end of 2019. b. 3,150 [($8,400/24 months) x 9 months].

Which of the following statements is true regarding the taxation of Social Security benefits? a. 85% is the maximum amount of taxable Social Security benefits. b. 50% is the maximum amount of taxable Social Security benefits. c. If a taxpayer's only source of income is $10,000 of Social Security benefits, then 50% of the benefits are taxable. d. If a taxpayer's only source of income is $10,000 of Social Security benefits, then 85% of the benefits are taxable.

a. 85% is the maximum amount of taxable Social Security benefits.

Regarding the tax formula and its relationship to Form 1040, which, if any, of the following statements is correct? a. A "page 1 deduction" refers to a deduction for AGI. b. The taxable income (TI) amount appears both at the bottom of page 1 and at the top of page 2 of Form 1040. c. An "above the line deduction" refers to a deduction from AGI. d. Most exclusions from gross income are reported on page 2 of Form 1040. e. None of these choices are correct.

a. A "page 1 deduction" refers to a deduction for AGI.

Casey is in the 12% marginal tax bracket, and Jean is in the 35% marginal tax bracket. Their employer is experiencing financial difficulties and cannot continue to pay for the company's health insurance plan. The annual premiums are approximately $8,000 per employee. The employer has proposed to either (1) require the employee to pay the premiums or (2) reduce each employee's pay by $10,000 per year with the employer paying the premium. a. ___ will experience a decrease in income net of health insurance premiums. b. Which option is less objectionable to Casey, and which is less objectionable to Jean? Casey would find the ___ less objectionable. Jean would find the ___ less objectionable.

a. Both Casey and Jean b. first option, second option Under option (1), Casey would be required to pay $8,000 in premiums each year. Assuming he cannot deduct the insurance as a medical expense because of the adjusted gross income floor, his cash flow after-tax and health insurance premiums will decrease by $8,000. Under option (2), Casey's cash flow after-tax and health insurance premiums would decrease by $8,800 [(1 - .12) × $10,000]. Therefore, Casey would be better off with option (1). Jean would find the second option less objectionable. As in Casey's case, with option (1) she is $8,000 poorer than without any change. But under option (2), her after-tax cash flow would decrease by $6,500 [(1 - .35) × $10,000]. Therefore, Jean would be better off with option (2).

Indicate whether the following statements are applicable to the "Cash receipts method" or the "Accrual method" of accounting. a. Property or services received are included in the taxpayer's gross income in the year of actual or constructive receipt by the taxpayer or agent, regardless of whether the income was earned in that year. b. An item is generally included in gross income for the year in which it is earned, regardless of when it is collected. c. All that is necessary for income recognition is that property or services received have a fair market value—a cash equivalent. d. The income is earned when (1) all events have occurred that fix the right to receive such income and (2) the amount to be received can be determined with reasonable accuracy.

a. Cash receipts method b. Accrual method c. Cash receipts method d. Accrual method

On February 24, 2018, Allison's building, with an adjusted basis of $1,300,000 (and used in her trade or business), is destroyed by fire. On March 31, 2018, she receives an insurance reimbursement of $1,650,000 for the loss. Allison invests $1,550,000 in a new building and buys stock with the balance of insurance proceeds. Allison is a calendar year taxpayer. a. Allison has until ___ to make the new investment and qualify for the nonrecognition election. b. Assuming that the replacement property qualifies as similar or related in service or use, Allison's realized gain is $___. Allison's basis in the new building is $___.

a. December 31, 2020 b. $350,000 (1,650,000 - 1,300,000) $100,000 (1,650,000 - 1,550,000) $1,300,000 [1,550,000 - (350,000 - 100,000)

Indicate whether the following are "Deductible" or "Nondeductible" as medical expenses for tax purposes. a. Transportation for medical care b. Medical and hospital insurance premiums c. Nonprescription drugs d. Diaper service, maternity clothes e. Eyeglasses (including contact lenses) f. Hearing aids

a. Deductible b. Deductible c. Nondeductible d. Nondeductible e. Deductible f. Deductible

Indicate whether the following personal expenditures are "Deductible" or "Not deductible" for tax purposes. a. Medical expenses b. Mortgage insurance c. Life insurance premiums d. Child support payments e. Charitable contributions f. Funeral expenses g. Credit card expense

a. Deductible b. Deductible c. Not deductible d. Not deductible e. Deductible f. Not Deductible g. Not Deductible

Indicate whether the following personal expenditures are "Deductible" or "Not deductible" for tax purposes. Ignore any limitations that may affect the deduction. a. Traditional IRA Contributions b. Alimony (pre-2019 divorce agreement) c. Fines and penalties d. Political contributions e. State income taxes f. Hobby losses

a. Deductible b. Deductible c. Not deductible d. Not deductible e. Deductible f. Not deductible

Nanette is a first-grade teacher. Potential deductions are charitable contributions of $800, personal property taxes on her car of $240, and various supplies purchased for use in her classroom of $225 (none reimbursed by her school). How will these items affect Nanette's Federal income tax return for 2018? a. Charitable contributions of $800 b. Personal property taxes on her car of $240 c. Supplies purchased for use in her classroom of $225

a. Deductions from AGI b. Deductions from AGI c. Deduction for AGI

Complete the following requirements regarding the statement "On a taxable disposition of a passive activity, the taxpayer can use any suspended losses and credits related to that activity." (Note: Assume that this is the only passive activity investment.) a. Do you agree or disagree with the statement above? b. Explain. When the only passive activity owned is disposed of, a final determination can be made. As a result, under the passive activity loss rules, upon a fully taxable disposition, any overall ___ realized from the activity is recognized and can be offset against ___. Suspended credits can be offset against the tax attributable to ___. . Unused credits are ___.

a. Disagree b. loss, all income, other passive activity income, lost

Mason performs services for Isabella. Based on the factors listed below, select "Employee" or "Independent contractor" to classify Mason's likely employment status. a. Mason performs services only for Isabella and does not work for anyone else. b. Mason sets his own work schedule. c. Mason reports his job-related expenses on a Schedule C. d. Mason obtained his job skills from Isabella's training program. e. Mason performs the services at Isabella's business location. f. Mason is paid based on time worked rather than on task performed.

a. Employee b. Independent contractor c. Independent contractor d. Employee e. Employee f. Employee

For the following statements, indicate either "True" or "False" regarding what is the "actually paid" requirement for the deduction of an expense by a cash basis taxpayer. a. The expenses of cash basis taxpayers are deductible only when they are actually paid with cash but not property. b. Promising to pay or issuing a note does not satisfy the actually paid requirement. c. At the time taxpayers charge expenses on their credit cards, they are allowed to claim the deduction. d. For a cash basis taxpayer, actual payment does not ensure a current deduction.

a. False b. True c. True d. True

Ed, an employee of the Natural Color Company, suffered from a rare disease that was very expensive to treat. The local media ran several stories about Ed's problems, and the family created a website that generated more than $10,000 in gifts from individuals to help pay the medical bills. Ed's employer provided hospital and medical insurance for its employees, but the policy did not cover Ed's illness. When it became apparent that Ed could not pay all of his medical expenses, the hospital canceled the $25,000 Ed owed at the time of his death. After Ed's death, his former employer, in accordance with company policy, paid Ed's widow $12,000 in "her time of need". Ed's widow also collected $50,000 on a group term life insurance policy paid for by Ed's employer. Classify the following amounts as "Included in" or "Excluded from" the gross income of Ed and his widow. a. $10,000 in gifts from individuals to help pay the medical bills. b. The $25,000 debt canceled by the hospital. c. The employer's $12,000 payment to Ed's widow. d. The employer-paid $50,000 group term life insurance policy. What are Ed's and his widow's gross income from these items? ___

a. Excluded from b. Excluded from c. Excluded from d. Excluded from $0

Thomas purchased a personal residence from Rachel. To sell the residence, Rachel agreed to pay $5,500 in points related to Thomas's mortgage. Indicate whether the following are "True" or "False" about points paid in this real estate transaction. a. The seller can deduct the points if certain conditions are met. b. To be deductible, the points paid represent a service charge or payment for specific services. c. Points paid by the seller are treated as an adjustment to the selling price of the residence. d. The buyer may deduct seller-paid points if several conditions are met

a. False b. False c. True d. True

Mike purchased four $100 tickets to a fund-raising dinner and dance sponsored by the public library, a qualified charitable organization. In its advertising for the event, the library indicated that the cost of the tickets would be deductible for Federal income tax purposes. Indicate whether the following statements are "True" or "False" regarding the contribution. a. Because it is a public library, Mike will receive a charitable contribution deduction for the total cost of the ticket. b. Mike will be required to reduce his deduction by the value of the dinner and the dance. c. Mike not be allowed to deduct any portion of the cost of ticket since his intent was to attend the dinner and dance and not make a contribution.

a. False b. True c. False

How does the earned income credit fit into the Federal government's plan to fight poverty? Indicate whether the following statements are "True or False" regarding how the earned income credit fits into the Federal government's plan to fight poverty. a. It is meant to provide tax equity to high-income taxpayers. b. It is meant to encourage low-income taxpayers to enter and remain in the workforce. c. The credit is designed to reimburse the taxpayer for many Federal taxes.

a. False b. True c. True

Indicate whether the following statements are "True" or "False" regarding investment losses. a. Securities held as an investment that become worthless produce ordinary losses. b. Sales of securities held as investments for less than basis result in capital losses. c. When an activity is classified as a hobby, any losses resulting are limited to the income from the activity.

a. False b. True c. True

Indicate whether the following statements are "True" or "False" regarding the doctrine of constructive receipt. a. The rationale for the constructive receipt doctrine is that if the income is available, the taxpayer should be allowed to postpone the income recognition. b. The constructive receipt doctrine does not apply to income the taxpayer is not yet entitled to receive even though the taxpayer could have contracted to receive the income at an earlier date. c. Income set apart or made available is not constructively received if its actual receipt is subject to substantial restrictions.

a. False b. True c. True

Allen visits Reno, Nevada, once a year to gamble. This year his gambling loss was $25,000. He commented to you, "At least I didn't have to pay for my airfare and hotel room. The casino paid that because I am such a good customer. That was worth at least $3,000." Select "True" or "False" to classify the following statements regarding Allen's relevant tax issues. a. Allen did not receive any cash, therefore no income. b. Under the broad concept, the airfare and hotel accommodations would be considered income. c. Taking into account the losses on this trip, if Allen combines his income and losses, he has an economic loss. d. Allen can report a $25,000 loss on this tax return.

a. False b. True c. True d. False

Indicate whether the following statements are "True" or "False" regarding accounting methods available to a partnership. a. Generally, the cash method may be adopted by a partnership that has one or more C corporation partner. b. A newly formed partnership may adopt either the cash or accrual method of accounting or a hybrid of these two methods. c. The cash method may not be adopted by a partnership that is a tax shelter. d. If a partnership uses the accrual method of accounting, its income must be reported no later than the date that income would be reported on the partnership's "applicable financial statement".

a. False b. True c. True d. True

Indicate whether the following statements are "True" or "False" regarding income from partnerships, S corporations, trusts, and estates. a. A partnership is a separate taxable entity. b. A small business corporation may elect to be taxed similarly to a partnership. c. Each partner report his or her distributive share of the partnership's income and deductions for the partnership's tax year ending in or with the partner's tax year. d. The beneficiaries of estates and trusts generally are taxed on the income earned by the estates or trusts that is actually distributed or required to be distributed to them.

a. False b. True c. True d. True

Jacob, a self-employed taxpayer, is married and has two children. With respect to Health Savings Accounts (HSAs), indicate whether of the following statements are "True" or "False". a. The taxpayer must itemize deductions in order to take the deduction. b. The income earned on the HSA is not included in gross income if it is used to pay medical expenses not covered by the high-deductible plan. c. Contributions to an HSA are deductible for AGI. d. The annual deduction for contributions to an HSA is limited to an amount that depends on whether the taxpayer has self-only coverage or family coverage.

a. False b. True c. True d. True

Patrick and Eva are planning to divorce in 2018. Patrick has offered to pay Eva $12,000 each year until their 11-year-old daughter reaches age 21. Alternatively, Patrick will transfer to Eva common stock that he owns with a fair market value of $100,000. a. Select either "True" or "False" to classify the following regarding the transfer of the stock from Patrick to Eva. • The stock cannot be transferred without recognition of a gain by Patrick or Eva. • Eva's cost basis in the stock will be the same as Patrick's. • Eva's cost basis in the stock will be fair market value on the day Patrick transfers it to her. • The transfer of stock will not qualify as alimony. b. If Patrick's cash payments to Eva are considered alimony, indicate whether the following are true or false? • The payments are considered as income to Eva and are deductible by Patrick. • In the event Eva dies, the payments can continue to be made to her estate. • Eva and Patrick can live in the same household when the payments are made. • The agreement must not specify that the payments are not alimony.

a. False, True, False, True b. True, False, False, True

Classify each of the following expenditures paid in 2018 as a deduction "For AGI", "From AGI", or "Not deductible". Note: Ignore any possible limitations. a. Roberto gives cash to his father as a birthday gift. b. Sandra gives cash to her church. c. Albert pays Dr. Dafashy for medical services rendered. d. Mia pays alimony to Bill in accordance with their separation agreement which was signed in 2018. e. Rex, who is self-employed, contributes to his pension plan. f. Bonita pays expenses associated with her rental property. g. Lu, who operates a sole proprietorship, takes a client to the theater after dinner, where they discussed new business.

a. Not deductible b. From AGI c. From Agi d. For AGI e. For AGI f. For AGI g. Not deductible

Wade paid $7,000 for an automobile that needed substantial repairs. He worked nights and weekends to restore the car and spent $2,400 on parts for it. He knows that he can sell the car for $13,000, but he is very wealthy and does not need the money. On the other hand, his daughter, who has very little income, needs money to make the down payment on a house. a. Would it matter, after taxes, whether Wade sells the car and gives the money to his daughter or whether he gives the car to his daughter and she sells it for $13,000? Select "True" or "False" regarding the tax consequences for each scenario. • Wade should give the car to his daughter to sell. She will not have to pay taxes on the subsequent sale of the car, since it was acquired as a gift. • If Wade sells the car, he would be taxed on the gain. However, if he gives the car to his daughter and she sells, the daughter would be taxed on any gain. • Wade should sell the car. If Wade gives his daughter the car, he will have to pay gift tax as well as income tax, since his daughter is considered a related party. • Wade should sell the car, since his taxable gain will be less due to the time he spent restoring the car than if his daughter sells the car. b. Assume Wade gave the car to his daughter after he had arranged for another person to buy it from his daughter. The daughter then transferred the car to the buyer and received $13,000. Who is taxed on the gain? Wade sold the car and ___should be taxed on the gain. If his daughter received the property without ___, ___ would be the taxpayer on the sale.

a. False, true, false, false b. Wade, a binding commitment, his daughter

Classify each of the following expenditures paid in 2018 as a deduction "For AGI", a deduction "From AGI", or "Not deductible". Ignore any possible limitations. a. Samuel has deductions attributable to the rental of his office building. b. Katie paid for a safe deposit box rental in which she stores stock certificates. c. Troy paid interest on qualified student loans. d. Roberto has a personal casualty loss which occurred in a federally declared disaster. e. Harriet paid a fine for speeding. f. Gabby, who teaches elementary school, paid for teacher supplies.

a. For AGI b. Not deductible c. For AGI d. From AGI e. Not deductible f. For AGI

Classify each of the following expenditures paid in 2018 as a deduction for AGI, partially deductible for AGI, a deduction from AGI, partially deductible from AGI, or not deductible. Ignore any percentage or dollar amount limitations. a. Barak contributes to his H.R. 10 plan (i.e., a retirement plan for a self-employed individual). b. Keith pays child support to his former wife, Renee, for the support of their son, Chris. c. Judy pays professional dues that are not reimbursed by her employer. d. Ted pays $500 as the monthly mortgage payment on his personal residence. Of this amount, $100 represents a payment on principal, and $400 represents an interest payment. e. Oni, a private citizen, pays a moving company for moving her household goods (a qualified moving expense) to Detroit, where she is starting a new job. She is not reimbursed by her employer. f. Ralph pays $6,000 of property taxes on his personal residence and $5,000 of state income taxes.

a. For AGI b. Not deductible c. Not deductible d. From AGI e. Not deductible f. Partially deductible from AGI

Indicate whether the following personal expenditures are deductible "For AGI" or "From AGI" (as itemized deductions). a. Sales/use taxes paid in lieu of state and local income taxes b. Cost of uniforms or other clothing that cannot be used for normal wear c. Job-hunting costs d. Fees incurred for the preparation of one's tax return e. Alimony payments f. Unreimbursed moving expenses

a. From AGI b. From AGI c. From AGI d. From AGI e. For AGI f. For AGI

Indicate whether the following items are "Included" in or "Excluded" from gross income for tax purposes. a. An employee embezzled $30,000 from his employer. b. A dentist received accounting services in exchange for dental work provided to a patient. c. Mary received $15,000 from the bank for a personal loan. d. A shareholder of Green Company received a $20,000 forgiveness of debt from the company.

a. Included b. Included c. Excluded d. Included

Indicate whether the following items "Increase" or "Decrease" a taxpayer's at-risk amount. a. Cash and the adjusted basis of property contributed to the activity. b. Withdrawals from the activity. c. Taxpayer's share of the activity's deductible loss. d. Amounts borrowed for use in the activity for which the taxpayer is personally liable or has pledged as security property not used in the activity. e. Taxpayer's share of amounts borrowed for use in an activity, involving the holding of real property, that are qualified nonrecourse financing.

a. Increase b. Decrease c. Decrease d. Increase e. Increase

Answer the following questions regarding the tax implications of various retirement plans. a. What is the difference between Keogh (H.R. 10) and traditional deductible IRA plans? b. What is the difference between traditional IRA and Roth IRA plans?

a. Keogh (H.R. 10) plans are limited to self-employed participants. b. Contributions to Roth IRAs are made with after-tax dollars.

Classify the following activities by selecting either "Passive" or "Not passive". a. Any trade, business, or income-producing activity in which the taxpayer does not materially participate. b. All rental activities, subject to certain exceptions. c. Income from intangible property if the taxpayer's personal efforts contributed significantly to the creation of the property. d. Gain on the sale or other disposition of assets used in an active trade or business. e. Interest, dividends, annuities, and royalties not derived in the ordinary course of a trade or business.

a. Passive b. Passive c. Not passive d. Not passive e. Not passive

Barbara incurred the following expenses during the year: $1,770 dues at a health club she joined at the suggestion of her physician to improve her general physical condition, $460 for multiple vitamins and antioxidant vitamins, $750 for a smoking cessation program, $545 for nonprescription nicotine gum, $815 for insulin, and $10,525 for funeral expenses for her mother who passed away in June. Which of these expenses may be included in computing the medical expense deduction? Select "May be included" or "May not be included", whichever is applicable. a. $1,770 dues at a health club she joined at the suggestion of her physician to improve her general physical condition b. $460 for multiple vitamins and antioxidant vitamins c. $750 for a smoking cessation program d. $545 for nonprescription nicotine gum e. $815 for insulin f. $10,525 for funeral expenses

a. May not be included b. May not be included c. May be included d. May not be included e. May be included f. May not be included

As sole heir, Cheval receives all of Emeraude's property (adjusted basis of $13,500,200 and fair market value of $14,375,000). Six months after Emeraude's death in 2018, the fair market value is $14,660,000. a. Can the executor of Emeraude's estate elect the alternate valuation date and amount? b. Cheval's basis for the property is $___ c. Assume, instead, that the fair market value six months after Emeraude's death is $14,300,000. Can the executor of Emeraude's estate elect the alternate valuation date and amount? Cheval's basis for the property is $___.

a. No b. $14,375,000 c. Yes, $14,300,000

Wes was a major league baseball pitcher who earned $10 million for his 20 wins this year. Sam was also a major league baseball pitcher before a career-ending injury caused by a negligent driver. Sam sued the driver and collected $6 million as compensation for lost estimated future income as a pitcher and $4 million as punitive damages. a. Do the amounts that Wes and Sam receive have the same effect on their gross income? b. Enter the amount, if any, to be included in the gross income for Wes and Sam. Note: Enter your answer in millions.

a. No b. Wes: $10 million Sam: $4 million

In each of the following situations, indicate whether the 50% reduction for meals applies. a. Each year, the employer awards its top salesperson an all-expense-paid trip to Jamaica. b. The employer has a cafeteria for its employees, where meals are furnished at cost. The value of the meals are not included in the compensation of employees. c. The employer sponsors an annual Labor Day picnic for its employees. d. Every Christmas, the employer gives each employee a fruitcake. e. The taxpayer gives business gifts to her clients at Christmas.

a. No b. Yes c. No d. No. e. No

fter netting all of her short-term and long-term capital gains and losses, Misty has a net short-term capital loss and a net long-term capital loss. a. Should these be netted against each other? b. All taxpayers net their capital gains and losses. First, ___ gains and losses are netted against one another. Second, ___ gains and losses are netted against one another. The result is ___. If these positions are ___, they are ___.

a. No b. short-term, long-term, any ONE of the above, oppositve, netted against each other

Paul and Sonja, who are married, reported 2018 itemized deductions of $13,200 and $400, respectively. Paul suggests that they file their Federal income tax returns separately—he will itemize his deductions from AGI, and she will claim the standard deduction. a. Is Paul's suggestion correct? b. What should they do to ensure the maximum tax benefit?

a. No. Sonja is ineligible to use the standard deduction and must itemize. b. Both should take the standard deduction.

Many years ago, Jack purchased 400 shares of Canary stock. During the current year, the stock became worthless. It was determined that the company "went under" because several corporate officers embezzled a large amount of company funds. Identify which of the following are relevant tax issues for Jack regarding the loss associated with his stock. a. Has Jack taken a loss related to worthless securities within the last three years? b. Should this be treated as a worthless security? c. Is it § 1244 stock? d. Should this be treated as a theft loss? e. If an NOL is created, what are the NOL carryforward implications?

a. Not an issue b. Yes, an issue c. Yes, an issue d. Yes, an issue e. Yes, an issue

Linda operates an illegal gambling operation. Indicate which of the following expenses that she incurs can reduce taxable income. Select "Deductible", "Partially Deductible", or "Not deductible", whichever is applicable. a. Bribes paid to city employees. b. Salaries to employees. c. Security cameras. d. Kickbacks to police. e. Rent on an office. f. Depreciation on office furniture and equipment. g. Tenant's casualty insurance. h. Utilities.

a. Not deductible b. Deductible c. Deductible d. Not Deductible e. Deductible f. Deductible g. Deductible h. Deductible

Explain the meaning of the terms active income, portfolio income, and passive activity income by classifying each of the following activities as "Active Income", "Portfolio Income", or "Passive Activity Income". a. Any trade or business or income-producing activity in which the taxpayer does not materially participate. b. Wages, salary, commissions, bonuses, and other payments for services rendered by the taxpayer. c. Gain on the sale or other disposition of assets used in an active trade or business. d. Interest, dividends, annuities, and royalties not derived in the ordinary course of a trade or business. e. Gain or loss from the disposition of property that produces portfolio income or is held for investment purposes. f. Income from intangible property if the taxpayer's personal efforts significantly contributed to the creation of the property. g. Profit from a trade or business in which the taxpayer is a material participant. h. Subject to certain exceptions, all rental activities, whether the taxpayer materially participates or not.

a. Passive Activity Income b. Active Income c. Active Income d. Portfolio Income e. Portfolio Income f. Active Income g. Active Income h. Passive Activity Income

Indicate whether the following items are classified as "Active", "Portfolio", or "Passive" income for the purpose of the passive loss rules. a. Interest, dividends, annuities, and royalties not derived in the ordinary course of a trade or business. b. Any trade or business or income-producing activity in which the taxpayer does not materially participate. c. Gain on the sale or other disposition of assets used in an active trade or business. d. Wages, salary, commissions, bonuses, and other payments for services rendered by the taxpayer.

a. Portfolio b. Passive c. Active d. Active

Diane owns a principal residence in Georgia, a townhouse in San Francisco, and a yacht in Cape Cod. All of the properties have mortgages on which Diane pays interest. a. For which residences can Diane deduct mortgage interest? b. What are the limitations on Diane's mortgage interest deduction? What strategy should Diane consider to maximize her mortgage interest deduction? Diane's deduction is limited to interest on acquisition indebtedness up to $___ if the debt existed at December 15, 2017 or ___ if incurred after that date. Therefore, she should choose the second residence that will result in the ___ interest deduction.

a. Principal residence plus one of the two other residences b. $1,000,000, $750,000, highest

Classify the following exchanges as "Qualifying" or "Not Qualifying" as a like-kind exchange under § 1031. a. Improved for unimproved real estate b. Vending machine (used in business) for inventory c. Rental house for personal residence d. Business equipment for securities e. Warehouse for office building (both used for business) f. Truck for computer (both used in business) g. Rental house for land (both held for investment) h. Ten shares of stock in Blue Corporation for 10 shares of stock in Red Corporation i. Office furniture for office equipment (both used in business) j. Unimproved land in Jackson, Mississippi, for unimproved land in Toledo, Spain k. General partnership interest for a general partnership interest

a. Qualifying b. Not Qualifying c. Not Qualifying d. Not Qualifying e. Qualifying f. Not Qualifying g. Qualifying h. Not Qualifying i.Not Qualifying j. Not Qualifying k. Not Qualifying

Elizabeth made the following interest-free loans during the year. Assume that tax avoidance is not a principal purpose of any of the loans. Assume that the relevant Federal rate is 5% and that the loans were outstanding for the last six months of the year. Borrower Amount Net Investment Richard $5,000 $800 Woody $8,000 $600 Irene $105,000 $0 What are the effects of the imputed interest rules on these transactions? Compute Elizabeth's gross income from each loan.

a. Richard is not subject to the imputed interest rules because the $10,000 exception does apply. Elizabeth's gross income from the loan is $0. b. The $10,000 exception does not apply to the loan to Woody because the proceeds were used to purchase income producing assets. Because the $1,000 exception applies to this loan, no interest is imputed. Elizabeth's gross income from the loan is $0. c. None of the exceptions apply to the loan to Irene because the loan was for more than $100,000. Elizabeth's gross income from the loan is $2,625 (105,000 * 5% * 6/12).

During the year, Hernando recorded the following transactions: a. Gain on the sale of stock held as an investment for 10 months. b. Gain on the sale of land held as an investment for 4 years. c. Gain on the sale of a houseboat owned for 2 years and used for family vacations. d. Loss on the sale of a reconditioned motorcycle owned for 3 years and used for recreational purposes. Tax Rates: Short-term capital gains are taxed at: Long-term capital gains are taxed at:

a. Short-term capital gain b. Long-term capital gain c. Long-term capital gain d. Nondeductible personal-use asset Tax Rates: Ordinary tax rates, Preferential tax rates

Which, if any, of the statements regarding the standard deduction is correct? a. Some taxpayers may qualify for two types of standard deductions. b. The standard deduction may be taken as a for AGI deduction. c. The basic standard deduction is indexed for inflation but the additional standard deduction is not. d. The standard deduction is not available to taxpayers who are dependents. e. None of these choices are correct.

a. Some taxpayers may qualify for two types of standard deductions.

Would an individual taxpayer receive greater benefit from deducting an expenditure or from taking a credit equal to 25% of the expenditure? a. Read each scenario then respond by selecting either "Take the credit" or "Take the deduction" from the dropdown lists. If the taxpayer's marginal tax rate is less than 25%? If the taxpayer's marginal tax rate is more than 25%? b. How would your response change if the item would only be deductible from AGI? If the item were deductible from AGI, a benefit would result only if the taxpayer ____.

a. Take the credit, take the deduction b. itemized his or her deductions

Ray loses his job as a result of a corporate downsizing. Consequently, he falls behind on the mortgage payments on his personal residence. His friend Ted would like to make the delinquent mortgage payments for him. a. Could the payments be structured so that Ray can deduct the mortgage interest? For Ray to deduct the interest: b. Could the payment arrangement deny both Ray and Ted a mortgage interest deduction? For neither Ray nor Ted to deduct the mortgage interest: c. Could the payment arrangement deny both Ray and Ted a mortgage interest deduction? For neither Ray nor Ted to deduct the mortgage interest:

a. Ted should give the money to Ray so that Ray pays the mortgage company. b. Ted pays the mortgage company directly. c. There is no way the payments can be structured so that Ted can deduct the mortgage interest.

Eagle Life Insurance Company pays its employees $.30 per mile for driving their personal automobiles to and from work. The company reimburses each employee who rides the bus $100 a month for the cost of a pass. Tom, in his Mazda 2-seat Roadster, collected $100 for his automobile mileage, and Mason received $100 as reimbursement for the cost of a bus pass. If an amount is zero, enter "0". a. What are the effects of the $100 reimbursement on Tom's and Mason's gross income? b. Assume that Tom and Mason are in the 24% marginal tax bracket and the actual before-tax cost for Tom to drive to and from work is $0.30 per mile. What are Tom's and Mason's after-tax costs of commuting to and from work?

a. Tom includes and Mason excludes the $100 gross income. b. Tom's after-tax cost is $24 and Mason's after-tax cost is $0 for commuting to and from work.

Bronze Corporation has $100,000 of active income, $55,000 of portfolio income, and a $55,000 passive activity loss. Indicate whether the following statements are "True" or "False" regarding whether Bronze Corporation is permitted to deduct the passive activity loss. a. The loss is not deductible if Bronze Corporation is a personal service corporation. b. The loss is deductible against portfolio income if Bronze Corporation is a closely held C corporation that is not a personal service corporation. c. No matter what type of corporation Bronze is, the passive activity loss is not d

a. True b. False c. False

Melissa, the owner of a sole proprietorship, does not provide health insurance for her 20 employees. She plans to spend $1,500 lobbying in opposition to legislation that would require her to provide such insurance. Regarding Melissa's lobbying expenses, indicate whether the following statements are "True" or "False". a. If Melissa spends the $1,500 on in-house lobbying expenditures, the de minimis exception allows her to deduct the entire amount. b. If Melissa pays the $1,500 to a professional lobbyist, the payment is deductible. c. If Melissa spends the $1,500 on in-house lobbying expenditures, she can deduct only $1,000.

a. True b. False c. False

Robert purchased and placed in service $100,000 of 7-year class assets on August 10 of the current year. He also purchased and placed in service $500,000 of 5-year class assets on November 15 of the current year. He does not claim any available additional first-year depreciation. Robert elects to use the MACRS straight-line method of cost recovery on the 7-year class assets. Regarding the calculation of cost recovery for the 5-year class assets, indicate which of the following statements are "True" and which are "False". Robert is a calendar year taxpayer. a. The cost recovery on the five-year class assets is computed using mid-quarter convention MACRS. b. The cost recovery on the five-year class assets is computed using the MACRS straight-line method. c. Neither the seven-year nor five-year assets can use the straight-line method, since those assets are not real property.

a. True b. False c. False

a. If a taxpayer sells goods or provides services on credit and the account receivable subsequently becomes worthless, a bad debt deduction is permitted only if income arising from the creation of the account receivable was previously included in income. b. A bad debt deduction is allowed for a bad debt arising from the sale of a product or service when the taxpayer is on the cash basis. c. Most taxpayers may use only the reserve method in accounting for bad debts. d. One of the more difficult tasks is determining if and when a bad debt is worthless. e. A taxpayer using the specific charge-off method may claim a deduction when a specific business debt becomes either partially or wholly worthless or when a specific nonbusiness debt becomes wholly worthless.

a. True b. False c. False d. True e. True

According to the Supreme Court, would it be good tax policy to use income as computed by financial accounting principles as the correct measure of income for Federal income tax purposes? Select from the dropdown lists either "True" or "False" to indicate if the statement made is true or false regarding the goals and mandates of financial and tax accounting. a. The primary goal of financial accounting is to provide useful information to management, shareholders, creditors, and others properly interested. b. The primary goal of financial accounting is to provide useful information needed to prepare the tax return. c. The primary goal of the tax system is the equitable collection of revenue. d. The primary goal of the income tax system is to collect all taxes.

a. True b. False c. True d. False

Label the following statements as either "True" or "False" as regards the treatment of Section 179 in a year in which a carryforward has occurred. For example, in 2018, which of the following statements would pertain to a Section 179 carryforward from 2017. a. It may be reduced, dollar-for-dollar, if the cost of § 179 property placed in service is in excess of $2,500,000. b. No limit applies to the carryover if the taxpayer is considered a small business and has total assets of less than $1,000,000. c. It may be limited to the business income in the carryforward year. d. It may be limited to the additional first-year depreciation taken in the carryforward year.

a. True b. False c. True d. False

Indicate whether the following statements are "True" or "False" regarding cost recovery for tax purposes. a. Realty (real property) generally includes land and buildings permanently affixed to the land. b. Personal use property is only personalty property (personal property) that is held for personal use rather than for use in a trade or business or an income-producing activity. c. Assets used in a trade or business or for the production of income are eligible for cost recovery if they are subject to wear and tear, decay or decline from natural causes, or obsolescence. d. The key date for the commencement of depreciation is the date an asset is purchased. e. The basis of cost recovery property is reduced by the cost recovery allowed, and not less than the allowable amount.

a. True b. False c. True d. False. e. True

William, a high school teacher, annual AGI = $50,000. In December 2018, he won $1,000,000 in the state lottery. William plans to donate $100,000 to his church. He has asked you, his tax adviser, whether he should donate the $100,000 in 2018 or 2019. Indicate whether the following statements are "True" or "False" regarding the tax issues related to William's decision. a. If he donates the $100,000 in any other tax year, his deduction will be limited to $30,000 plus or minus 60% of other income and adjustments. b. William will not be allowed to carry forward the excess contribution not taken as a deduction in the year of contribution. c. William's AGI in the year of receipt of the lottery winnings would be ample enough for him to take the entire contribution as a deduction in that year. d. The deductions for donations to a qualified public charity (his church) are limited to 60% of adjusted gross income (AGI). William should donate the $100,000 in ___

a. True b. False c. True d. True 2018

Howard buys wrecked cars and stores them on his property. Recently, he purchased a 1990 Ford Taurus for $400. If he can sell all of the usable parts, his total proceeds from the Taurus will be over $2,500. As of the end of the year, he has sold only the radio for $75 and he does not know how many, if any, of the remaining parts will ever be sold. What are Howard's income recognition issues? Regarding Howard's income recognition, select either "True" or "False" to properly classify the statements made. a. He could consider all sales proceeds as a recovery of capital until he receives $400, and then all subsequent proceeds would be included in his gross income. b. He could allocate the cost of the car ($400) among the various parts he sells. This would allow him to reduce his gross receipts by the adjusted basis of the parts sold. c. He can consider all sales proceeds as a recovery of capital until he receives $2,500.

a. True b. True c. False

Indicate whether the following statements are "True" or "False" regarding material participation in an activity. a. Section 469 requires a taxpayer to participate on a regular, continuous, and substantial basis to be a material participant. b. Participation generally includes any work done by an individual in an activity that he or she owns. c. Participation by an owner's spouse does not counts as participation by the owner.

a. True b. True c. False

Indicate whether the following statements are "True" or "False" regarding tax planning strategies. a. Taxpayers who have passive activity losses (PALs) should adopt a strategy of generating passive activity income that can be sheltered by existing passive losses. b. The taxpayer should carefully select the year in which to dispose of a passive activity. It is to the taxpayer's advantage to wait until sufficient passive losses have accumulated to offset any gain recognized on the asset's disposition. c. If money is borrowed to finance the purchase of a passive activity, the associated interest expense is never treated as part of any passive loss.

a. True b. True c. False

Indicate whether the following statements are "True" or "False" regarding tax withholding and payment procedures. a. The employer must match the FICA portion withheld on employees and fully absorb the cost of FUTA tax. b. If a child employed by parent (or partnership in which each partner is a parent of the child), the child is exempt from withholding Social Security and Medicare taxes until age 18. c. If one spouse is employed by the other spouse, there is an exemption regarding the withholding of income taxes and Social Security and Medicare taxes.

a. True b. True c. False

Indicate whether the following statements are "True" or "False" regarding the tax aspect of tax shelters. a. Tax shelters provided a popular way to avoid or defer taxes. b. Many tax shelters merely provided an opportunity for "investors" to buy deductions and credits in ventures that were not expected to generate a profit. c. The typical tax shelter operated as a corporation and relied heavily on recourse financing.

a. True b. True c. False

Harold and Bart own 75% of the stock of Orange Motors. The other 25% of the stock is owned by Jeb. Orange Motors entered into an agreement with Harold and Bart to acquire all of their Orange stock. In addition, Harold and Bart signed a noncompete agreement with Orange Motors. Under the terms of the noncompete agreement, Orange will pay Harold and Bart $15,000 each per year for four years. Help Orange Motors by classifying the following as either "True" or "False". a. The noncompete agreement is considered a § 197 intangible if it is acquired in connection with the acquisition of a business. b. If amortization is permitted, it will be over a 15-year period. c. $15,000 is the basis for purposes of calculating MACRS depreciation. d. If amortization is permitted, it will be ratably amortized over the designated period, beginning in the middle of the quarter it was acquired.

a. True b. True c. False d. False

For each of the following statements, identify each as either "True" or "False" with regard to the significance of the term material participation. a. Losses from a nonrental activity in which the taxpayer materially participates can offset active and portfolio income. b. Material participation is determined by the level of participation of the taxpayer in the activity. c. Section 469 requires a taxpayer to own at least 50% of the business to be a material participant. d. A high enough level of participation indicates that the taxpayer has an interest in the activity other than as an investment.

a. True b. True c. False d. True

Indicate whether the following statements are "True" or "False" regarding capital expenditures incurred for medical purposes. a. The capital expenditures must be deemed medically necessary by a physician. b. The item or facility must be used primarily by the patient alone and the expense must be reasonable. c. Appraisal costs related to capital improvements are deductible as medical expenses. d. The full cost of certain home-related capital expenditures incurred to enable a physically handicapped individual to live independently and productively qualifies as a medical expense. e. 'Additional costs to operate and maintain the item are deductible as medical expenses as long as the medical reason for the capital expenditure continues to exist.

a. True b. True c. False d. True e. True

Indicate whether the following statements are "True" or False" regarding tax planning strategies for minimizing gross income. a. The taxpayer can often defer the recognition of income from appreciated property by postponing the event triggering realization. b. Series EE bonds can be purchased for long-term deferrals of income. c. Because no tax is due until a gain has been recognized, the law does not favor investments that yield appreciation rather than annual income. d. For the accrual basis taxpayer who receives advance payments from customers, the transactions should be structured to avoid payment of tax on income before the time the income is actually earned. e. The tax liability of a family can be minimized by shifting income from higher- to lower-bracket family members through gifts of income-producing property.

a. True b. True c. False d. True e. True

Label the following as "True" or "False" regarding the definition of taxable income as it is used in limiting the § 179 expensing amount. a. The taxable income computation for purposes of the § 179 limit includes the deduction for MACRS. b. Taxable income of a trade or business is computed without regard to the amount expensed under § 179. c. The aggregate amount of taxable income includes net income from a trade or business as well as from the production of income activities. d. The taxable income computation for purposes of the § 179 limit excludes the deduction for additional first-year depreciation.

a. True b. True c. False d. False

Indicate whether the following statements are "True" or "False" regarding the tax credit for the elderly or disabled. a. Taxpayers under age 65 who are retired with a permanent and total disability and who have disability income from a public or private employer on account of the disability qualify for the credit. b. Any taxpayers age 65 or older qualifies for the credit. c. The maximum allowable credit is $3,000. d. The credit generally is based on a qualifying income amount and the filing status of the taxpayer.

a. True b. True c. False - The maximum allowable credit is 15 percent of qualifying retirement income (in general, $5,000 for single taxpayers or married taxpayers where one spouse qualifies; $7,500 for married taxpayers filing jointly where both spouses qualify). This qualifying income amount is reduced by (1) Social Security, Railroad Retirement, and certain excluded pension benefits and (2) one-half of the taxpayer's AGI in excess of a threshold amount based on filing status. d. True - Initial base amount is reduced by (1) Social Security, Railroad Retirement, and certain excluded pension benefits and (2) one-half of the taxpayer's AGI in excess of a threshold amount, which is a function of the taxpayer's filing status.

Regarding how the cost of mineral rights enter into the calculation of cost depletion, classify each statement below as either "True" or "False". a. Cost depletion is determined by using the adjusted basis of the asset, and is a deduction for adjusted gross income. b. For cost depletion of the mineral rights, an economic interest in the resource is required. c. To compute the depletion for mineral rights, the depletion per unit is multiplied by the number of units sold during the year to arrive at the cost depletion allowed.

a. True b. True c. True

Ross would like to dispose of some land he acquired five years ago because he believes that it will not continue to appreciate. Its value has increased by $50,000 over the five-year period. He also intends to sell stock that has declined in value by $50,000 during the eight-month period he has owned it. Ross has four offers to acquire the stock and land: Buyer 1: Exchange land. Buyer 2: Purchase land for cash. Buyer 3: Exchange stock. Buyer 4: Purchase stock for cash. Classify each of the following as either "True" or "False" regarding Ross's tax issues relevant to disposing of this land and stock. a. Only the exchange of the land qualifies for § 1031 deferral treatment. b. Whether he sells or exchanges the stock, the loss is not deferred. c. If he sells the stock, Ross may want to sell the land as well, so that the recognized gain from the land can be offset by the recognized loss on the stock. d. Both the land and the stock qualify for § 1031 treatment, so Ross may defer the loss on the stock and the gain on the land.

a. True b. True c. True d. False

a. Compensatory damages received on account of physical personal injury or physical sickness can be excluded from gross income. b. A payment for damaged or destroyed property is treated as an amount received in a sale or exchange of the property. c. Compensatory damages awarded on account of emotional distress cannot be excluded. d. Punitive damages are excluded from gross income. e. Compensatory damages received for age discrimination or injury to one's reputation cannot be excluded.

a. True b. True c. True d. False e. True

Indicate whether the following statements are "True" or "False" regarding the concept of gross income. a. While the Constitution grants Congress the power to tax income, it does not define the term. b. The Supreme Court has held that there is no income subject to tax until the taxpayer has recovered the capital invested. c. Economists measure income (economic income) by first determining the fair market value of the individual's net assets (assets minus liabilities) at the beginning and end of the year (change in net worth). d. Accounting and tax rules regarding income are the same. e. The accounting concept of income is founded on the realization principle. f. Gross income is not limited to cash received.

a. True b. True c. True d. False e. True f. True

Indicate whether the following statements are "True" or "False" regarding various sources of income. a. The taxpayer can avoid including prizes and awards in gross income by refusing to accept the prize or award. b. Certain employee achievement awards in the form of tangible personal property that are made in recognition of length of service or safety achievement are excludable. c. The fair market value of prizes and awards (other than scholarships exempted under § 117) must be included in gross income. d. The premiums on the first $100,000 of group term life insurance protection are excludable from the employee's gross income. e. After experiencing dissatisfaction with the IRS's treatment of unemployment compensation, Congress amended the Code to make the benefits taxable. f. If the group term life insurance plan discriminates in favor of certain key employees (e.g., officers), the key employees are not eligible for the exclusion.

a. True b. True c. True d. False e. True f. True

Lucy owns a craft store and a dry cleaning business in New York. She also owns a craft store and a dry cleaning business in Miami. Indicate whether the following statements are "True" or "False" regarding possible groupings for tax purposes. Assume reasonable methods of applying the facts and circumstances test prevail. a. All four activities may be grouped into a single activity because of common ownership and control. b. The craft stores may be grouped into an activity, and the dry cleaning stores may be grouped into a separate activity. c. Each of the four activities may be treated as a separate activity. d. The New York activities may be grouped into an activity, and the Miami activities may be grouped into a separate activity.

a. True b. True c. True d. True

In computing the medical expense deduction, a taxpayer may include medical expenses for a person who was a dependent at the time the expenses were paid or incurred. Which of the following requirements apply in determining dependency status for the medical deduction? Select "Yes" or "No", whichever is applicable. a. Member of Household or Relationship Test b. Joint Return Test c. CItizen or Residenct Test d. Age Test e. Support Test f. Gross Income Test

a. Yes b. No c. Yes d. Yes e. Yes f. No

At the entity level, which of the following passive activity loss rules apply. Identify each by selecting either "Yes" they apply or "No" they do not apply. a. Individuals. b. Closely held C corporations. c. S corporations. d. Partnerships. e. Personal service corporations.

a. Yes b. Yes c. No d. No e. Yes

Indicate whether the following would be counted in applying the material participation standard. Classify each item as "Yes" or "No" regarding which would be counted in applying the material participation standard. a. Supervising employees b. Working as a salesperson c. Providing services to customers d. Having your spouse work as a salesperson e. Examining financial statements in a nonmanagerial capacity f. Selling goods g. Any other activity associated with the day-to-day operations of a business

a. Yes b. Yes c. Yes d. Yes e. No f. Yes g. Yes

In connection with the office in the home deduction, complete each section below: a. Based on the following descriptions, classify each as "Yes" it qualifies for the office in the home deduction or "No" it does not. • An exclusive office is the principal place of business for the independent yoga instructor. • The home office is for the convenience of the employer. • The family den is used by the taxpayer to conduct administrative and management activities of their manufacturing business b. Read each statement below regarding the office in the home deduction. Direct expenses benefit only the business part of the home and may be deducted in full, while indirect expenses are for maintaining and operating the home and must be allocated between business and personal use. Direct expenses benefit only the business and indirect expenses are for maintaining and operating the home. Both may be deducted in full if there is an office in the home. Direct expenses benefit only the business part of the home and indirect expenses are for maintaining and operating the home. Both are totaled and allocated based on square footage if there is an office in the home. Select from the dropdown list the statement that accurately outlines the deductibility of direct and indirect expenses. c. Complete the statement regarding the effect a taxpayer's work status (i.e., employed or self-employed) has on the office in the home deduction. ___ classify the deduction as for AGI and as an ___ employee business expense, an employee gets ___ deduction for 2018 through 2025. d. True/False: The office in the home deduction is not available to taxpayers who do not own their home (e.g., who live in rented apartments). e. What is the treatment of office furnishings (e.g. desk, chairs, and file cabinets) on the office in the home deduction? f. Regarding the office in the home deduction, what is the treatment of expenses that exceed gross income from the business?

a. Yes, Yes, No b. Statement 1 c. Self-employed taxpayers, unreimbursed, no d. False e. They are handled separately and are either expensed under §179 or depreciated. f. They are carried to the next year.

A professional consulting business sells professional tools and equipment and provides associated services, such as repair and maintenance, to its customer base. The company's employees include technicians who are required to provide and maintain their own tools and equipment for performing the repairs and maintenance work. The company will reimburse a technician for amounts spent to purchase tools and equipment eligible for a § 179 deduction up to a set amount each year. Any costs for tools and equipment that exceed the set amount will not be reimbursed. John is a technician for the company. During the current year, he purchased equipment that qualifies for the § 179 deduction. John paid $50,000 for the equipment and was reimbursed the set amount of $40,000. Classify for John the following as either "Yes, a relevant tax issue" or "No, not a tax issue" with respect to § 179 and the computation of his taxable income. a. Is John or the professional consulting business entitled to a § 179 deduction? b. Is the § 179 deduction allowed only for employers? c. How much, if any, can John deduct under § 179 on his own tax return? d. What are the tax consequences of the reimbursements that John receives?

a. Yes, a relevant tax issue b. No, not a tax issue c. Yes, a relevant tax issue d. Yes, a relevant tax issue

Does the taxpayer recognize gross income in the following situations? a. Ava is a filing clerk at a large insurance company. She is permitted to leave the premises for lunch, but she usually eats in the company's cafeteria because it is quick and she is on a tight schedule. On average, she pays $2 for a lunch that would cost $12 at a restaurant and it cost her employer $10 to prepare. However, if the prices in the cafeteria were not so low and the food was not so delicious, she would probably bring her lunch at a cost of $3 per day. Ava's meals are provided as ___. Therefore, Ava would include $___ per meal in her gross income. b. Scott is an executive for an international corporation located in New York City. Often he works late, taking telephone calls from the company's European branch. Scott often stays in a company-owned condominium when he has a late-night work session. The condominium is across the street from the company office and has the technology needed to communicate with employees and customers throughout the world. Because the lodging is provided as ___, Scott ___ in gross income. c. Ira recently moved to take a job. For the first month on the new job, Ira was searching for a home to purchase or rent. During this time, his employer permitted Ira to live in an apartment the company maintains for customers during the buying season. The month that Ira occupied the apartment was not during the buying season, and the apartment would not otherwise have been occupied. The use of the apartment should qualifies as ___ and therefore is ___ gross income.

a. a convenience for the employee, $10 b. a convenience for the employer, is not required to include anything c. a no-additional-cost service, excluded from

Faith Godwin is a dealer in securities. She has spotted a fast-rising company and would like to buy and hold its stock for investment. The stock is currently selling for $2 per share, and Faith thinks it will climb to $40 a share within two years. Faith's coworkers have told her that there is "no way" she can get long-term capital gain treatment when she purchases stock because she is a securities dealer. Faith has asked you to calculate her potential gain and tell her whether her coworkers are right. a. If Faith clearly identifies the security as held for investment purposes by the close of business on the ___ date, she will have a $___ per-share ___. b. The purpose of this letter is to discuss the rules for purchases of stock by a securities dealer such as yourself. Your coworkers are ___that a securities dealer may not have a long-term capital gain from the sale of stock. The tax law ___ you to designate stock you purchase as being held for investment. You must make this designation by the end of the business day on which the stock is ___. I suggest you check with your supervisor on how this designation is normally done at your firm. I am sure it is a relatively simple procedure. As long as you continue to hold the designated shares for investment until you ___ them, shares are a ___. When you sell $40 per share, your $___ per-share gain will be long-term capital gain if you have held the shares for more than one year.

a. acquisition, $38, long-term capital gain b. incorrect, allows, acquired, sell, capital asset, $38

In 2018, Kelsey sustained a loss on the theft of a painting. She had paid $20,000 for the painting, but it was worth $40,000 at the time of the theft. a. If the painting is treated as investment property, the loss is ___. Before adjustments (if any), the loss is limited to the ___ of the property at the time of destruction. b. If the painting is treated as personal-use property, the loss is ___. Before adjustments (if any), the loss is limited to the ___ of the property at the time of destruction.

a. an itemized deduction not subject to the $100-per-event and the 10%-of-AGI limitations, adjusted basis b. a casualty and theft loss but not deductible since not related to a Federally declared disaster area, lesser of decline in fair market value or the adjusted basis

If a taxpayer sells property for cash, the amount realized consists of the net proceeds from the sale. For each of the following, indicate the effect on the amount realized. a. The property is sold on credit. The amount realized includes ___. b. A mortgage on the property is assumed by the buyer. The assumption by the buyer of the seller's mortgage ___ the amount realized by ___. c. A mortgage on the property is assumed by the seller. The assumption by the seller of the buyer's mortgage ___ the amount realized by ___. d. The buyer acquires the property subject to a mortgage of the seller. The buyer's acquisition of the property subject to the mortgage of the seller ___ the amount realized by ___. e. Stock that has a basis to the purchaser of $6,000 and a fair market value of $10,000 is received by the seller as part of the consideration. The receipt of the stock by the seller ___ the amount realized by the seller by $___.

a. both the cash received at the time of sale and the cash to be received in the future b. increases, the seller c. decreases, the seller d. increases, the seller e. increases, $10,000

Mortgaged real estate may be received in a like-kind exchange. Answer the following questions regarding mortgaged real estate. a. If the taxpayer assumes a mortgage in connection with a like-kind exchange, the assumption of the mortgage ___ trigger the recognition of realized gain. The amount of the mortgage assumed ___ the basis of the like-kind property given in calculating the basis of the like-kind property received. b. From the perspective of the taxpayer who is being relieved of the mortgage, the amount of the mortgage ___ treated as boot received, and any realized gain ___.

a. does not, is added to b. is, recognized to the extent of the lesser of the boot received or the realized gain

Katrina owns undeveloped land with an adjusted basis of $225,000. She exchanges it for other undeveloped land worth $500,000. Assume that Katrina holds the land as an investment. a. Katrina's realized is ___ $___ and her recognized ___ s $___. b. Katrina's basis in the undeveloped land she receives is $ c. Would the answers for the above questions change if Katrina exchanged the undeveloped land for land and a building?

a. gain, $275,000, gain, $0 b. $225,000 c. No

In January 2018, Sonja deposited $20,000 in a bank in the Bahamas. She earned $500 interest income. She closed the account in December 2018. a. Is Sonja subject to the FBAR reporting requirement? Sonya __ required to file an FBAR form for 2018 because on at least one day during 2018 she had over $___ in a foreign bank account. b. Is the interest income taxable in the United States? ___, ___ of the interest income earned from this account is taxable in the United States.

a. is, $10,000 b. Yes, all

Marilyn owns land that she acquired three years ago as an investment for $250,000. Because the land has not appreciated in value as she had anticipated, she sells it to her brother, Amos, for its fair market value of $180,000. Amos sells the land two years later for $240,000. a. Why is Marilyn's realized loss of $70,000 ($180,000 amount realized - $250,000 adjusted basis) disallowed at the time of the sale to her brother? Amos __ considered a related party under § 267. Therefore, ___ from sales or exchanges of property between certain related parties are not recognized. b. Why does Amos have neither a recognized gain nor a recognized loss on his sale of the land? Although Amos has a realized ___ of ___, he does not recognized a gain or a loss on his sale of the land because it is ___. c. From a tax perspective, how does the related-party disallowance rule affect the total gain or loss recognized by the family unit? d. Which party fares more favorably? ___, because ___. e. How could Marilyn have avoided the loss disallowance on her sale of the land?

a. is, only realized losses b. gain, $60,000, offset by Marilyn's disallowed loss c. In this case the tax results are less favorable because the taxpayers are related d. Amos, Marilyn was not permitted to use any of her realized loss e. By selling the land to an unrelated party

Lisa sells business property with an adjusted basis of $130,000 to her son, Alfred, for its fair market value of $100,000. a. What is Lisa's realized and recognized gain or loss? Lisa's realized ___ ___ recognized. b. What is Alfred's recognized gain or loss if he subsequently sells the property for $138,000? For $80,000? If Alfred sells the property for $138,000, he has a recognized ___ of ___. If Alfred sells the property for $80,000, he has a recognized ___ of ___.

a. loss, $30,000, is not b. gain, $8,000, loss, $20,000

Complete the following statements regarding possible tax planning strategies for minimizing gross income. a. For pre-2019 agreements, the person ___ the alimony payments favors a divorce settlement that includes a provision for alimony payments. b. Being able to distinguish between separate and community property is vital in the estate tax area because the surviving wife's or husband's share of the community property ___ included in the gross estate of the deceased spouse.

a. making b. is not

Donna donates stock in Chipper Corporation to the American Red Cross on September 10, 2018. She purchased the stock for $20,125 on December 28, 2017, and it had a fair market value of $28,750 when she made the donation. a. The stock is treated as ___ property and Donna's charitable contribution deduction is $___ for tax purposes. b. Assume instead that the stock had a fair market value of $17,250 (rather than $28,750) when it was donated to the American Red Cross. Donna's charitable contribution deduction would be $___ for tax purposes.

a. ordinary income, $20,125 b. $17,250

Wanda is considering selling two personal use assets that she owns. One has appreciated in value by $20,000, and the other has declined in value by $17,000. Wanda believes that she should sell both assets in the same tax year so that the loss of $17,000 can offset the gain of $20,000. a. Advise Wanda regarding the tax consequences of her plan by completing the statements below. For tax purposes, the sale of a personal use asset resulting in a gain is ___ and the sale of a personal use asset resulting in a loss is ___. Therefore, Wanda reports for tax purposes ___. b. Could Wanda achieve better tax results by selling the assets in different tax years? ___. If Wanda sells the assets in different years, she reports for tax purposes a gain of ___ and a loss of $0.

a. realized and recognized, realized but not recognized, only her gain b. No, $20,000, $0

Leon sells his interest in a passive activity for $139,500. Determine the tax effect of the sale based on each of the following independent facts: If an amount is zero, enter "0". a. Adjusted basis in this investment is $48,825. Losses from prior years that were not deductible due to the passive activity loss restrictions total $53,708. The ___ is $___. The suspended losses at the end of the year are $___. b. Assume the same sales price but the adjusted basis in this investment is $104,625. Losses from prior years that were not deductible due to the passive activity loss restrictions total $53,708. The ___ is $___. The suspended losses at the end of the year are $___. c. Assume the same sales price but the adjusted basis in this investment is $104,625. Losses from prior years that were not deductible due to the passive activity loss restrictions total $53,708. In addition, suspended credits total $13,950. The ___ is $___. The suspended losses at the end of the year are $___. The suspended credits at the end of the year are ___.

a. taxable gain, $36,967 (139,500 - 53,708 - 48,825), $0 b. deductible loss, $18,833 (139,500 - 104,625 - 53,708), $0 c. deductible loss, $18,833, $0, lost

Blaze operates a restaurant in Cleveland. He travels to Columbus to investigate acquiring a business. He incurs expenses as follows: $1,500 for travel, $2,000 for legal advice, and $3,500 for a market analysis. Based on the different tax consequences listed below, describe the circumstances that were involved in Blaze's investigation of the business. a. Blaze deducts the $7,000 of expenses. He must be investigating a business that is ___ b. Blaze cannot deduct any of the $7,000 of expenses. He must be investigating a business that is ___ c. Blaze acquired a business outside the restaurant industry. He can currently deduct $___ of the expenses and the balance is ___.

a. the same/similar to his current business b. not a restaurant; and did not acquire it c. 5,000, amortized over 180 months

Classify each of the following cases as "Included in" or "Excluded from" gross income. a. Manny was injured while water skiing due to defect in one of the water skis. As a result of a lawsuit filed against the manufacturer of the water skis, Manny received $34,000 in compensatory damages due to the injury. In addition, Manny received $9,000 in reimbursed medical expenses and $7,500 for loss of income associated with the injury. b. Warren sued for slander against a competitor. He received punitive damages of $15,000. c. As a result of a lawsuit, Jasper received $4,000 compensatory damages awarded on account of emotional distress. In addition, Jasper received $1,800 compensatory damages awarded on account of age discrimination. d. As result of a lawsuit filed against his former employer, Zach received $6,200 in compensatory for injury to his reputation and $7,000 for loss of income. e. Josephine received compensatory damages of $40,000 for personal injuries resulting from a car accident. In addition, Josephine received $13,000 for damages to her vehicle. (Assume she has zero basis in the vehicle.)

a1. Excluded from a2. Excluded from b. Included in c. Included in c2. Included in d. Included in e1. Excluded from e2. Included in

After his first business trip to a major city, Herman is alarmed when he reviews his credit card receipts. Both the hotel bill and the car rental charge are in excess of the price he was quoted. Complete the paragraph below to outline what Herman needs to understand about these excess amounts. It appears that Herman was charged a hotel occupancy tax and the rental car tax. These are two examples of __ ______ tax because the taxes are _______ __ _ __________ ___________ __ _______.

an excise, limited to a particular transaction or service

Hakim's rental building (a § 1231 asset) was not insured when it was destroyed by a hurricane. His adjusted basis for the building was substantial but was less than he had paid for the building in 2013. The building was Hakim's only asset that was damaged by the hurricane. How is the loss treated for tax purposes? Hakim has a casualty loss equal to the ___ of the building, and the loss is treated as a deduction ___ AGI.

adjusted basis, for

Is a note receivable that arose in the ordinary course of the taxpayer's retail business a capital asset? A note receivable that arises in the ordinary course of the taxpayer's business of selling inventory or performing services is ___ asset because it is specifically ___ the Section 1221 (a) definition of what is not a capital asset.

an ordinary, included in

Complete the statements below that outline the difference between personal property and personal use property. Personalty is defined as ___. Personal use property is ___that is not held for income production or used in a trade or business. Therefore, cost recovery deductions ___ allowed for personal use assets.

any asset that is not realty, realty or personalty property, are not

In the determination of whether a business expense is deductible, the statutory language of the Code refers to reasonableness only with respect to salaries and other compensation. The courts have held that for ___ the amount must be reasonable.

any business expense to be ordinary and necessary

Complete the statements below regarding the tests for material participation. A general partner may qualify as a material participant only if ___ are met. Generally, a limited partner is not considered a material participant unless he or she qualifies under ___.

any of the seven tests, any of the tests based on prior participation or participates in the activity for more than 500 hours during the year

Carol and Dave each purchase 100 shares of stock of Burgundy, Inc., a publicly owned corporation, in July for $10,000 each. Carol sells her stock on December 31 for $8,000. Because Burgundy's stock is listed on a national exchange, Dave is able to ascertain that his shares are worth $8,000 on December 31. Does the tax law treat the decline in value of the stock differently for Carol and Dave? Select the letter from the dropdown list for the statement that is true regarding the tax treatment of Carol and Dave's stock shares. a. They are treated the same as both experienced a decline in stock value that is considered a realization event. b. They are treated differently because the loss in value of Carol's stock is the result of a sale, while the loss in value of Dave's stock is simply a decline in value. c. They are treated the same because Carol and Dave have a decline in fair market value, which is considered the relevant factor. The correct statement is:

b - They are treated differently because the loss in value of Carol's stock is the result of a sale, while the loss in value of Dave's stock is simply a decline in value.

Mary purchased an annuity that pays her $500 per month for the rest of her life. She paid $70,000 for the annuity. Based on IRS annuity tables, Mary's life expectancy is 16 years. How much of the first $500 payment will Mary include in her gross income (round to two decimals)? a. $0 b. $135.42 c. $364.58 d. $500.00

b. $135.42 Based on IRS tables, Mary is expected to receive 192 (16 years x 12 months) annuity payments. Her investment in the annuity is $70,000 and her return of capital for each annuity payment is $70,000/192 = $364.58. The return of capital portion of each annuity payment is not taxable (not included in gross income). Mary must include the excess received ($500.00 - 364.58) of $135.42 in her gross income.

Gordon, an employee, is provided group term life insurance coverage equal to twice his annual salary of $125,000 per year. According to the IRS Uniform Premium Table (based on Gordon's age), the amount is $12 per year for $1,000 of protection. The cost of an individual policy would be $15 per year for $1,000 of protection. Since Gordon paid nothing towards the cost of the $250,000 protection, Gordon must include in his 2018 gross income which of the following amounts? a. $1,350. b. $2,400. c. $3,000. d. $3,750. e. None of these choices are correct.

b. $2,400. Gordon must include in gross income the Uniform Premium Table amount for $200,000 ($250,000 coverage less the $50,000 exclusion): 200 × $12 = $2,400.

Alice gifted stock to her son, Bob, in year 5. Alice bought the stock in year 1 for $8,300. The value of the stock on the date of gift was $6,400. Bob sold the stock in year 7 for $15,800. What is Bob's recognized gain or loss on the sale in year 7? a. $0 b. $7,500 gain c. $9,400 gain d. $15,800 gain

b. $7,500 gain (15,800 - 8,300)

Natalie is married to Chad, who abandoned her in early June of 2018. She has not seen or communicated with him since then. She maintains a household in which she and her two dependent children live. Which of the following statements about Natalie's filing status in 2018 is correct? a. Natalie can file a joint return with Chad. b. Natalie can file as a head of household. c. Natalie can use the rates for single taxpayers. d. Natalie can file as a surviving spouse. e. None of these statements are appropriate.

b. Natalie can file as a head of household.

Jerry and Ernie are comparing their tax situations. Both are paying all of the nursing home expenses of their parents. Jerry can include the expenses in computing his medical expense deduction, but Ernie cannot. Complete the statements below on why their tax situations may differ. The cost of care in a nursing home, along with ___, ___with the costs for medical or nursing care if the primary reason is ___. If the primary reason for being there is personal, ___ costs for medical or nursing care ___ in the total of deductible medical expenses, ___. Consequently, it appears that Jerry's parents are in the nursing home ___, while Ernie's parents are in the nursing home ___.

both meals and lodging, are included, to get medical care, any, can be included, but meals and lodging must be excluded, to get medical care, for reasons other than medical care

Complete the following statements regarding the treatment of real estate activities. Losses from real estate rental activities are not treated as passive losses for certain real estate professionals. To qualify for nonpassive treatment, a taxpayer must satisfy ___of the following requirements: ___ of the personal services that the taxpayer performs in trades or businesses are performed in real property trades or businesses in which the taxpayer materially participates. The taxpayer performs more than ___ hours of services in these real property trades or businesses as a material participant.

both, more than half, 750

Betty purchased an annuity for $24,000 in 2018. Under the contract, Betty will receive $300 each month for the rest of her life. According to the actuarial estimates, Betty will live to receive 96 payments and will receive a 3% return on her original investment. a. Betty has no gross income until she has collected $24,000. b. If Betty collects $3,000 in 2018, her gross income is $630 (.03 × $21,000). c. If Betty lives to collect more than 96 payments, all of the amounts collected after the 96th payment must be included in taxable income. d. If Betty lives to collect only 60 payments before her death, she will report a $6,000 loss from the annuity [$24,000 - (60 × $300) = $6,000] on her final return. e. None of these choices are correct.

c. If Betty lives to collect more than 96 payments, all of the amounts collected after the 96th payment must be included in taxable income.

On January 1, Father (Dave) loaned Daughter (Debra) $100,000 to purchase a new car and to pay off college loans. There were no other loans outstanding between Dave and Debra. The relevant Federal rate on interest was 6 percent. The loan was outstanding for the entire year. a. Dave must recognize $6,090 of imputed interest income regardless of the amount of Debra's investment income. b. Debra must recognize $6,090 of imputed interest income if Dave has at least $6,090 of investment income. c. If Debra has $15,000 of investment income, Dave must recognize $6,090 of imputed interest income. d. Debra must recognize $6,090 of imputed interest income. e. None of these choices are correct.

c. If Debra has $15,000 of investment income, Dave must recognize $6,090 of imputed interest income. The calculated imputed interest is $6,090 [($100,000 × 6% × .5) + ($103,000 × 6% × .5)]. Under the $100,000 exemption for below-market loans, Dave must recognize $6,090 of imputed interest income because Debra has investment income in excess of the calculated imputed interest income.

Travis and Andrea were divorced in 2016. Their only marital property consisted of a personal residence (fair market value of $400,000, cost of $200,000), and publicly-traded stocks (fair market value of $800,000, cost basis of $500,000). Under the terms of the divorce agreement, Andrea received the personal residence and Travis received the stocks. In addition, Andrea was to receive $50,000 for eight years. I. If the $50,000 annual payments are to be made to Andrea or her estate (if she dies before the end of the eight years), the payments will qualify as alimony. II. Andrea has a taxable gain from an exchange of her one-half interest in the stocks for Travis' one-half interest in the house and cash. III. If Travis sells the stocks for $900,000, he must recognize a $400,000 gain. a. I, II, and III are true. b. Only I and II are true. c. Only III is true. d. Only I and III are true. e. None of these choices are true.

c. Only III is true.

Complete the statements below regarding cafeteria and flexible spending plans. Assume that none of the premiums will be deductible due to AGI limitations for medical expenses. Under a ___ plan, the employee is permitted to choose between cash and ___. Under a ___ plan, the employee accepts ___ in return for ___ paying certain costs that it can pay without the ___ recognizing gross income.

cafeteria, nontaxable benefits, flexible spending, lower cash compensation, employer, employee

Complete the following statements regarding the tax year. Generally, an entity must use the ___ year to report its income. However, a ___ year can be elected if the taxpayer maintains adequate books and records. The ___ year option generally is not available to partnerships, S corporations, and personal service corporations.

calendar, fiscal, fiscal

Sean is in the business of buying and selling stocks and bonds. He has a bond of Green Corporation for which he paid $200,000. The bond is currently worth only $50,000. Can Sean take a $150,000 loss for a business bad debt or for a worthless security? Sean ___ take the loss as a business bad debt. Sean ___ take the loss as a worthless security because losses are allowed ___.

cannot, cannot, only when the security is completely worthless

What is the purpose of the constructive receipt doctrine? The constructive receipt doctrine prevents the ___ basis taxpayers from ___ recognition of income by intentionally ___ the receipt of the income in the current tax year.

cash, deferring, avoiding

If business property or property held for the production of income is ___ destroyed, the loss is equal to the adjusted basis of the property at the time of destruction. The ___ destruction of business property and income-producing property and for ___ destruction of personal use property, the loss is the ___ of the ___ of the property or the difference between the ___ of the property before the event and the ___ immediately after the event.

completely, partial, complete or partial, lesser, adjusted basis, fair market value, fair market value

Bill and Jane Jones were divorced on January 1, 2018. They have no children. In accordance with the divorce decree, Bill transferred the title of their house over to Jane. The home had a fair market value of $250,000 and was subject to a $100,000 mortgage. Under the divorce agreement, Bill is to make $1,000 monthly mortgage payments on the home for the remainder of the mortgage. In the current year, Bill made 12 mortgage payments. What amount is taxable to Jane in the current year? a. $12,000 b. $250,000 c. $100,000 d. $0

d. $0 Because the divorce settlement provides for the payments, no deduction is allowable for payments made and the payments are not includable in gross income of the spouse receiving the payment.

The taxable portion of Social Security benefits may be affected by: a. The taxpayer's itemized deductions. b. The number of quarters the individual worked. c. The individual's standard deduction. d. The individual's tax-exempt interest income. e. None of these choices are correct.

d. The individual's tax-exempt interest income.

A Series EE U.S. government savings bond accrues 3.5% interest each year. The bond matures in three years, at which time the principal and interest will be paid. The bank will pay the taxpayer at a 3.5% interest rate each year if he agrees to leave money on deposit for three years. What tax advantage does the Series EE bond offer that is not available with the bank deposit? Complete the statements below by selecting from the dropdown lists the correct response. The income from the Series EE bond is ___and __ exempt from the OID rules. The interest on the bank account is ___ under the OID rules. Therefore, with the ___, the taxpayer is earning interest on his deferred taxes.

deferred until maturity, is, taxed each year, Series EE bonds

Complete following statements regarding the comparison of the accounting and tax concepts of income. The primary goal of ___ is to provide useful information to management, shareholders, creditors, and others properly interested. The primary goal of ___ is the equitable collection of revenue.

financial accounting, the income tax system

Chloe, a self-employed dentist, tells you "My profits for the year total $300,000, so I don't need to pay any more Federal self-employment taxes." Complete the following statements regarding her comment. Chole's comment is ___. For 2018, the Social Security portion of the self-employment tax ___ and the Medicare portion of the self-employment tax ___.

incorrect, applies to earnings up to $127,200, has no wage limit

Dolly is a cash basis taxpayer. In 2018, she filed her 2017 Virginia income tax return and received a $2,200 refund. Dolly took the standard deduction on her 2017 Federal income tax return, but will itemize her deductions in 2018. Molly, a cash basis taxpayer, also filed her 2017 Virginia income tax return in 2018 and received a $600 refund. Molly had $12,000 in itemized deductions on her 2017 Federal income tax return, but will take the standard deduction in 2018. Complete the statements below regarding how the tax benefit rule applies to Dolly's and Molly's situations. Dolly ___ required to recognize income from the receipt of the state income tax refund of $2,200. The refund merely corrects for her overpayment and the original payment ___ her taxable income. Molly received a tax benefit in the form of a deduction on her 2017 Federal income tax return. Therefore, the $600 refund is the recovery of a tax benefit. Whether Dolly and Molly itemize deductions in the year of recovery ___ relevant to whether they realized gross income from the recovery of 2017 state income taxes.

is not, did not affect, is not

Carey is a waiter at a restaurant that pays a small hourly amount plus tips. Customers are not required to tip the waiter. Carey is especially attentive and friendly, and her tips average 25% of the restaurant charges. Carey ___ to include all of her tips in gross income. Therefore, the payments ___ compensation for services.

is required, are

Bob owns a collection agency. He purchases uncollected accounts receivable from other businesses at 60% of their face value and then attempts to collect these accounts. During the current year, Bob collected $60,000 on an account with a face value of $80,000. Bob has ___ debt deduction. He has ___ of $___.

no bad, income, $12,000

Casper and Cecile are divorced in 2018. As part of the divorce settlement, Casper transferred stock to Cecile. Casper purchased the stock for $25,000, and it had a market value of $43,000 on the date of the transfer. Cecile sold the stock a month after receiving it for $40,000. In addition Casper is required to pay Cecile $1,500 a month in alimony. He made five payment to her during the year. What are the tax consequences for Casper and Cecile regarding these transactions? If an amount is zero, enter "$0". Casper recognizes ___ on the transfer of the stock. He receives ___ for the $7,500 alimony paid. Cecile recognizes ___ on the transfer of the stock. She has ___ from the $7,500 alimony received. When she sells the stock Cecile reports a ___ of $___.

no gain or loss, a deduction, no gain or loss, taxable income, gain, $15,000

In connection with passive activities, complete the statements regarding what a deduction equivalent is and how is it computed. The deduction equivalent is the amount of deduction necessary that ___ by an amount ___. A deduction equivalent is calculated by ___ the taxpayer's ___.

reduces the tax liability for the taxable year, equal to the credit, dividing the tax credits available by, tax bracket percentage

Monty loaned his friend Ned $12,000 three years ago. Ned signed a note and made payments on the loan. Last year, when the remaining balance was $9,000, Ned filed for bankruptcy and notified Monty that he would be unable to pay the balance on the loan. Monty treated the $9,000 as a nonbusiness bad debt. Last year, before considering the tax implications of the nonbusiness bad debt, Monty had capital gains of $3,600 and taxable income of $43,750. During the current year, Ned paid Monty $8,100 in satisfaction of the debt. Determine Monty's tax treatment for the $8,100 received in the current year. The nonbusiness bad debt of $9,000 would have been reported as a ___, and $___ would be included in Monty's gross income.

short-term capital loss, $6,600 ($3,600 capital gain + $3,000 max gain)

Rose dies with passive activity property having an adjusted basis of $150,000, suspended losses of $48,000, and a fair market value at the date of her death of $210,000. Of the $48,000 suspended loss existing at the time of Rose's death, how much is deductible on her final return or by the beneficiary? The basis for the property is ___ to $___; therefore, ___ of the $48,000 suspended loss is deductible on Rose's final return or by the beneficiary.

stepped-up, $210,000, none

Dave uses the second floor of a building for his residence and the first floor for his business. The uninsured building is destroyed by fire. Are the tax consequences the same for each part of the building? If the casualty occurs in a Federally declared disaster area, the casualty loss on the residence portion is ___; otherwise the loss is ___. The casualty loss on the business portion is ___ and is ___.

treated as a deduction from AGI, not deductible, allowed as a business loss, a deduction for AGI

Mary receives tangible personal property as a gift. The property was depreciated by the donor, and Mary will also depreciate it. At the date of the gift, the property was worth more than the donor's adjusted basis. What is the impact of these facts on Mary when she sells the property at a gain several years after she acquired it? Because the donor depreciated the tangible personal property, the ___ depreciation recapture ___ over to Mary. She ___ this potential depreciation recapture to the depreciation she takes on the property.

§ 1245, carries, adds

Ultimately, most taxes are paid by individuals.

True

During the year, Addison is involved in the following transactions. What are the possible income tax ramifications of these transactions?

Transaction 1: Lost money gambling on a trip to a casino. Gambling losses are deductible to the extent of winnings. Transaction 2: Helped pay for her neighbor Howie's dental bills. Howie is a good friend who is unemployed. Barring an exception for dependents, no deduction is allowed for payment of some other person's expenses. Transaction 3: Received from the IRS a tax refund due to Addison's overpayment of last year's Federal income taxes. A Federal income tax refund is not income since it is an adjustment of a prior expenditure that was not deductible. Transaction 4: Paid a traffic ticket received while double parking to attend a business meeting. Fines and penalties are not deductible. Transaction 5: Contributed to the mayor's reelection campaign. The mayor had promised Addison to rezone some of her land. Political contributions are not deductible. Transaction 6: Borrowed money from a bank to make a down payment on an automobile. Borrowing money does not result in income. Transaction 7: Sold a houseboat and a camper on eBay. Both were personal use items, and the gain from one offset the loss from the other. Gains and losses from the sale of personal-use assets cannot offset each other. The gains are taxable and the losses are not deductible. Transaction 8: Paid for Aunt Katherine's funeral expenses. No deduction can be claimed for income tax purposes for the funeral expenses. Transaction 9: Paid premiums on her dependent son's life insurance policy. Premiums on personal life insurance policies are not deductible even when paid on behalf of a dependent.

Even though a client refuses to correct an error on a past return, it may be possible for a practitioner to continue to prepare returns for the client.

True

Mona inherits her mother's personal residence, which she converts to a furnished rental house. These changes should affect the amount of ad valorem property taxes levied on the properties.

True

One of the major reasons for the enactment of the Federal estate tax was to prevent large amounts of wealth from being accumulated within the family unit.

True

Sales made by mail order are not exempt from the application of a general sales (or use) tax.

True

The Federal income tax on individuals generates more revenue than the Federal income tax on corporations.

True

Compute the 2018 tax liability, the marginal, and average tax rates for the following taxpayers. Click here to access the 2018 tax rate schedule. If required, round the tax liability the nearest dollar. When required, round the average rates to four decimal places before converting to a percentage (i.e. .67073 would be rounded to .6707 and entered as 67.07%). a. Chandler, who files single, has taxable income of $94,800. b. Lazare, who files as head of household, has taxable income of $57,050.

a. Tax liability: $17,042 = ($94,800 - 82,500) = $12,300 x 24% = $2,952 + $14,089.50 Marginal rate: 24% Average rate: 17.98% = ($17,042/$94,800) b. Tax liability: $7,099 ($57,050 - $51,800) = $5,250 x 22% = $1,155 + $5,944 Marginal rate: 22% Average rate: 12.44% ($7,099/$57,050)

Mia owns a warehouse that has a cost basis of $80,000. The city condemns the warehouse to make room for a new fire station. It pays Mia $400,000 for the property, its agreed-to fair market value. Shortly after the condemnation, Mia purchases another warehouse as a replacement. If amount is zero, enter "0". a. What is her recognized gain if the new property cost is $280,000? b. What is her recognized gain if the new property cost is $444,000? c. What is her recognized gain if the new property cost is $80,000? d. What, if any, is the justification for deferring the recognition of gain on the involuntary conversion? The involuntary conversion provision is justified under the ___ and the notion that the taxpayer's economic position ___ changed.

a. $120,000 = ($400,000 - $280,000) b. $0 c. $320,000 = ($400,000 - $80,000) d. wherewithal to pay concept, has not

Mia owns a warehouse that has a cost basis of $190,000. The city condemns the warehouse to make room for a new fire station. It pays Mia $950,000 for the property, its agreed-to fair market value. Shortly after the condemnation, Mia purchases another warehouse as a replacement. If amount is zero, enter "0". a. What is her recognized gain if the new property cost is $665,000? b. What is her recognized gain if the new property cost is $1,045,000? c. What is her recognized gain if the new property cost is $190,000? d. What, if any, is the justification for deferring the recognition of gain on the involuntary conversion? The involuntary conversion provision is justified under the ___ and the notion that the taxpayer's economic position ___ changed.

a. $285,000 = ($950,000 - $665,000) b. $0 c. $760,000 = ($950,000 - $190,000) d. wherewithal to pay concept, has not

Answer the following questions regarding the value added tax (VAT). a. What is the popularity of the value added tax (VAT)? Over ___countries impose a value added tax (VAT). b. What is the nature of the tax? A value added tax (VAT) is a tax on ___ c. What effect can a VAT have on government spending? It can ___ government spending.

a. 140 b. consumption c. lead to more

Indicate whether the following items can be "Included in" or "Excluded from" gross income.

a. Alimony payments received (divorce finalized in 2016). Included in b. Award received by the taxpayer for compensatory damages from her broken leg. Excluded from c. A new golf cart won in a church raffle. Included in d. Amount collected on a loan previously made to a college friend. Excluded from e. Insurance proceeds paid to the taxpayer on the death of her uncle—she was the designated beneficiary under the policy. Excluded from f. Interest income on City of Chicago bonds. Excluded from g. Jury duty fees. Included in h. Stolen funds that the taxpayer ostensibly had collected for a local food bank drive. Included in i. Reward paid by the IRS for information provided that led to the conviction of the taxpayer's former employer for tax evasion. Included in j. Rare coins worth $8,000 found in an old trunk purchased by the taxpayer at a garage sale. Included in

Select a response for each of the following questions. a. What is the purpose of the unified transfer tax credit? b. Is the same amount available for both the Federal gift tax and the estate tax? c. Does the use of the credit for a gift affect the amount of credit available for the estate tax?

a. To eliminate the tax on modest gifts and estates. b. Yes c. Yes

Regarding the statute of limitations on additional assessments of tax by the IRS, select the applicable date in each of the following situations. a. The income tax return for 2017 was filed on April 12, 2018. The three-year statute of limitations will begin to run on ___. b. The income tax return for 2017 was filed on October 7, 2018. The statute of limitations will begin to run on ___. c. The income tax return for 2017 was prepared on April 1, 2018, but was never filed. Through some misunderstanding between the preparer and the taxpayer, each expected the other to file the return. The statute of limitations ___. d. The income tax return for 2017 was never filed because the taxpayer thought no additional tax was due. The statute of limitations ___.

a. April 17, 2018 b. October 7, 2018 c. will not begin to run d. will not begin to run

Indicate whether the following items can be "Included in" or "Excluded from" gross income.

a. During the year, shares of stock that the taxpayer had purchased as an investment which doubled in value. Excluded from b. Amount an off-duty motorcycle police officer received for escorting a funeral procession. Included in c. While his mother Shirley was in the hospital, the taxpayer sold some of Shirley's jewelry to help pay for the medical costs. Included in d. Child support payments received. Excluded from e. A damage deposit the taxpayer recovered when he vacated the apartment he had rented. Excluded from f. Interest received by the taxpayer on an investment in school bonds issued by IBM. Included in g. Amounts received by the taxpayer, a baseball "Hall of Famer," for autographing sports equipment (e.g., balls and gloves). Included in h. Tips received by Matt, a bartender, from patrons. (Matt is paid a regular salary by the cocktail lounge that employs him.) Included in i. Sherri sells her Super Bowl tickets for three times what she paid for them. Included in j. Jefferson receives a new BMW from his grandmother when he passes the CPA exam. Excluded from

Choose from the dropdown list the probable justification for each of the following aspects of the tax law: a. A tax credit is allowed for amounts spent to furnish care for minor children while the parent works. b. Deductions for interest on home mortgage and property taxes on a personal residence. c. The income-splitting benefits of filing a joint return. d. Fines and penalties are not deductible. e. Net operating losses of a current year can be carried forward to profitable years. f. A taxpayer who sells property on an installment basis can recognize gain on the sale over the period the payments are received. g. The exclusion from Federal tax of certain interest income from state and local bonds. h. Prepaid income is taxed to the recipient in the year received and not in the year earned.

a. Economic/Social considerations b. Economic/Social considerations c. Equity and Political considerations d. Social considerations e. Equity considerations f. Wherewithal to pay concept g. Political considerations h. Wherewithal to pay concept

Contrast FICA and FUTA regarding the following questions. a. Complete the statement below regarding the purpose of the tax. ____ offers some measure of retirement security. ____ provides a modest source of income. b. Upon whom is the tax imposed? c. Which governmental body administers the tax? d. Which taxes are reduced based on a merit rating system?

a. FICA, FUTA b. FICA is imposed on both the employer and employee, while FUTA is imposed only on the employer. c. FICA is administered by the Federal government. FUTA, however, is handled by both the Federal and state governments. d. Only FUTA

Indicate whether the following statements are "True" or "False" regarding the "Statements on Standards for Tax Services" dealing with CPAs engaged in tax practice. a. Although these pronouncements are part of the AICPA's Code of Professional Conduct, they are not enforceable. b. Any tax position taken should be supported by a good-faith belief that they have a realistic possibility of being sustained, if challenged. c. A practitioner can never use a client's estimates. d. Every effort should be made to answer questions appearing on tax returns.

a. False b. True c. False d. True

Indicate whether the following statements are "True" or "False" regarding the legislative process. a. When the Senate version of the bill differs from that passed by the House, the President resolves these differences. b. Assuming no disagreement between the House and the Senate, a bill passed by the Senate is referred to the President for approval or veto. c. Joint Conference Committee Reports often explain the provisions of the proposed legislation and are therefore a valuable source in ascertaining the intent of Congress.

a. False b. True c. True

Indicate whether the following statements are "True" or "False" regarding the pay-as-you-go feature of the Federal income tax system. a. Employers have the option of withholding income tax on compensation paid to their employees and to pay this tax to the government. b. If taxpayers receive income that is not subject to withholding or income from which not enough tax is withheld, they may have to pay estimated tax. c. The pay-as-you-go feature of the Federal income tax system requires payment of all or part of the taxpayer's income tax liability during the year.

a. False b. True c. True

Indicate whether the following statements are "True" or "False" regarding the statute of limitations. a. Under the general rule, the IRS may assess an additional tax liability against a taxpayer within five years of the filing of the income tax return. b. A statute of limitations is a provision in the law that offers a party a defense against a suit brought by another party after the expiration of a specified period of time. c. If a taxpayer omits an amount of gross income in excess of 25 percent of the gross income reported on the return, the statute of limitations is increased to six years. d. A claim for refund generally must be filed within three years from the date the return was filed or within two years from the date the tax was paid, whichever is later.

a. False b. True c. True d. True

Indicate whether the following statements are "True" or "False" regarding deductions from gross income. a. Nonbusiness expenses are expenses incurred in connection with an income-producing activity that does not qualify as a trade or business. Such expenses are deductions for adjusted gross income. b. As a general rule, personal expenditures are disallowed as deductions in arriving at taxable income. c. AGI is an important subtotal that serves as the basis for computing percentage limitations on certain itemized deductions. d. In lieu of claiming itemized deductions, taxpayers will use the standard deduction.

a. False b. True c. True d. True

Many state income tax returns contain checkoff boxes that allow taxpayers to make donations to a multitude of local charitable causes. On what grounds has this procedure been criticized? The checkoff boxes add complexity to the return and mislead taxpayers into presuming that taxpayers ___ paying for the donation.

are not

For each of the following, select either "True" or "False" regarding tax policy. a. Revenue neutrality. •Taxpayers who earn the same amount of income pay the same amount of tax. •Every new tax law that lowers taxes must include a revenue offset that makes up for the loss. (For every dollar lost, a new dollar is gained.) b. Pay-as-you-go. •Another way of describing revenue neutrality. •Requires employers to withhold for taxes a specified portion of an employee's wages. •Requires the IRS to pay refunds to taxpayers within 12 months of a filed tax return. c. Sunset provision. •Reinstates the law as it existed prior to the tax cut. •Maintains the law in place until Congress has an opportunity to review it and determine whether it should remain in effect. d. Indexation. •The IRS makes annual adjustments to certain key tax components to take into account inflation. •Some of the more important components that are adjusted include the tax brackets and the standard deduction amounts.

a. False, True b. True, True, False c. True, False d. True, True

Serena operates a gift shop. To reduce costs of credit card transactions, she offers customers a discount if they pay in cash. For the holiday rush, she hires some short-term workers, but pays them cash and does not add them to the payroll. a. Indicated whether the following statements are "True" or "False" regarding the tax issues for Serena. She will not have to record the cash transactions. She will owe FICA and FUTA on the short-term workers' wages. Her workers will be classified as casual laborers or independent contractors. She will be required to issue W-2 wage forms to the workers. b. Serena's chances of being audited are ____. This is because she is ___, and appears to operate on ___ basis.

a. False, True, False, True b. high, self-employed, a cash

As to those states that impose an income tax, complete the following: a. Differentiate a "piggyback" approach and "decoupling". For state income tax purposes, "piggyback" means applying a rate to the _______ ______ ___ _________. By "decoupling," a state decides ___ __ allow a particular Federal provision (e.g., exclusion, deduction, credit) for state income tax purposes. b. Using IRS audit results as part of a state tax audit. States _____ use IRS audit results to identify errors that might also exist on the taxpayer's state tax return. c. Credit for taxes paid to other states. Most states _____ their residents some form of tax credit for income taxes paid to other states.

a. Federal income tax liaility, not to b. often c. allow

Indicate whether the following items are "Included" in or "Excluded" from gross income for tax purposes. a. Alimony b. Gifts c. Child support payments d. Life insurance paid upon death e. Inheritances f. Prizes g. Jury duty fees

a. Included b. Excluded c. Excluded d. Excluded e. Excluded f. Included g. Included

Some tax rules can be justified on multiple grounds (e.g., economic and social). Keeping in mind this connection, select either "Yes" or "No" to indicate possible justifications for the rules governing the following items. a. Pension plans. •Private retirement plans are not encouraged since they compete with the Social Security system. •When an employer makes a contribution for an employee to a qualified pension or profit sharing plan, that contribution and any income it generates are not taxed to the employee until the funds are distributed. b. Education. •The various tax credits, deductions, and exclusions that are designed to encourage taxpayers to obtain additional education can be justified on both social and economic grounds. •The education tax incentives cannot be justified on both social and economic grounds since they benefit only the individual. c. Home ownership. •Providing renters with a Federal income tax benefit for the rent they pay encourages home ownership. •Allowing homeowners to deduct interest on their home mortgages encourages home ownership.

a. No, Yes b. Yes, No c. No, Yes

In March 2018, Jim asks you to prepare his Federal income tax returns for tax years 2015, 2016, and 2017. In discussing this matter with him, you discover that he also has not filed for tax year 2014. When you mention this fact, Jim tells you that the statute of limitations precludes the IRS from taking any action as to this year. a. Is Jim correct about the application of the statute of limitations? b. If Jim refuses to file for 2014, should you prepare returns for 2015 through 2017? Select "Yes" or "No", whichever is applicable. Only after you inform the IRS of the error. Because the error may prevent the current year's tax liability from being determined.

a. No. There is no statute of limitations if a return is not filed. b. No, Yes

ndicate whether the following are examples of a "Progressive" or a "Proportional" tax. a. FICA tax b. Federal income tax c. FUTA tax d. Federal gift tax e. Federal estate tax

a. Proportional b. Progressive c. Proportional d. Progressive e. Progressive

Identify the appropriate tax based on the definitions provided. a. A transaction tax that is based on the notion that the state has an interest in its natural resources. b. A tax that is imposed on the right to pass property at death. c. A tax levied on the right to transfer property during a taxpayer's lifetime. d. A tax on the right to receive property from a decedent.

a. Severance tax b. Estate tax c. Gift tax d. Inheritance tax

Indicate whether the following items are examples of "Economic", "Social", "Equity", or "Political" considerations regarding tax laws. a. A contribution made by an employer to a qualified pension or profit sharing plan for an employee may receive special treatment. b. A tax credit is allowed for amounts spent to furnish care for certain minor or disabled dependents to enable the taxpayer to seek or maintain gainful employment. c. Because double taxation results when the same income is subject to both foreign and U.S. income taxes, the tax law permits the taxpayer to choose between a credit or a deduction for the foreign taxes paid. d. To stimulate the manufacturing industry, Congress enacted a domestic production activities deduction. e. The installment method of recognizing gain on the sale of property allows a taxpayer to spread tax consequences over the payout period.

a. Social b. Social c. Equity d. Economic e. Equity

Answer the following questions regarding the pay-as-you-go procedure. a. How does the pay-as-you-go procedure apply to wage earners? b. How does the pay-as-you-go procedure apply to persons who have income from sources other than wages?

a. The tax law requires employers to withhold a specified dollar amount from wages paid to the employee to cover income taxes. b. The tax law requires the taxpayer to make quarterly payments to the IRS for estimated taxes.

Jake (age 72) and Jessica (age 28) were recently married. To avoid any transfer taxes, Jake has promised to leave Jessica all of his wealth when he dies. Indicate whether the following statements are "True" or "False" regarding the operation of the Federal gift and estate taxes. a. The marital deduction allows interspousal transfers (whether by gift or at death) free of any tax (either gift or estate). b. The marital deduction only allows interspousal transfers free of any tax if given as a gift during the donor's lifetime. c. The marital deduction is equal to the annual exclusion of $15,000 per year.

a. True b. False c. False

Indicate whether the following statements are "True" or "False" regarding the history of the Federal income tax. a. The Federal Union and the Confederate States of America used the income tax to raise funds to finance the Civil War. b. The U.S. Supreme Court found the federal corporate income tax to be unconstitutional because it was treated as an excise tax. c. The ratification of the Sixteenth Amendment to the U.S. Constitution in 1913 sanctioned both the Federal individual and corporate income taxes.

a. True b. False c. True

Indicate whether the following statements are "True" or "False" regarding the kiddie tax. a. Unearned income includes income such as taxable interest, dividends, capital gains, rents, royalties, pension and annuity income. b. The kiddie tax, applies to any child who is under age 19 (or under age 24 if a full-time student) and has unearned income of more than $1,050. c. The kiddie tax does not apply if both parents are deceased.

a. True b. False c. True

Indicate whether the following statements are "True" or "False" regarding the influence of the Federal courts on tax law. a. Some court decisions have been of such consequence that Congress has incorporated them into statutory tax law. b. A leading tax concept developed by the courts deals with the interpretation of statutory tax provisions that operate to benefit taxpayers. c. The courts have established the rule that the relief provisions are to be broadly construed if there is any doubt about their application.

a. True b. True c. False

Indicate whether the following statements are "True" or "False" regarding the trends of the Federal income tax. a. The need for revenues to finance the war effort during World War II converted the income tax into a mass tax. b. In 1943, Congress passed the Current Tax Payment Act, which provided for the first pay-as-you-go tax system. c. One trend that has caused considerable concern is the decreasing complexity of the Federal income tax laws.

a. True b. True c. False

Indicate whether the following statements are "True" or "False" regarding IRS audit procedures. a. Only a small number of tax returns are audited. b. The IRS utilizes mathematical formulas and statistical sampling techniques to select tax returns that are most likely to contain errors. c. If an individual's itemized deductions are in excess of norms established for various income levels, the probability of an audit is decreased. d. The tax law permits the IRS to pay rewards to persons who provide information that leads to the detection and punishment of those who violate the tax laws.

a. True b. True c. False d. True

Indicate whether the following statements are "True" or "False" regarding characteristics of ad valorem taxes on realty. a. Some jurisdictions extend immunity from tax for a specified period of time (a tax holiday) to new or relocated businesses. b. Some states partially exempt the homestead, or personal residence, portion of property from taxation. c. Lower taxes may apply to a residence owned by a taxpayer aged 65 or younger. d. Property owned by the Federal government is exempt from this tax.

a. True b. True c. False d. True

Indicate whether the following statements are "True" or "False" regarding whether the breadth and number of Federal excise taxes is increasing or decreasing. a. Federal excise taxes had declined insignificantly as to the number of transactions covered. b. Federal excise taxes on items such as tobacco products, fuel and gasoline sales, and air travel have increased. c. Some Federal excise taxes try to influence social behavior.

a. True b. True c. True

Select "True" or "False" to identify some reasons why taxpayers often have more than one alternative for structuring a business transaction. a. Tax law contains many "gray" areas that are open to interpretation. b. The complexity of the tax laws makes it difficult at times to determine the correct treatment. c. The possibility of different interpretations of the tax law allows for different treatments. d. The simplicity of the Federal tax law makes it easier to come up with different treatments.

a. True b. True c. True d. False

The Benson CPA firm is considering utilizing an offshore service provider to prepare many of its tax returns. Select either "Yes" or "No" to indicate whether the following are ethical considerations that should be taken into account regarding offshore preparation. a. Notify the client, preferably in writing, of the outsourcing. b. Outsourcing tax return preparation is not compatible with accounting ethical guidelines. c. Outsourcing tax return preparation violates Federal law. d. Verifying the accuracy of the work done. e. Maintaining client confidentiality.

a. Yes b. No c. No d. Yes e. Yes

Sophia lives several blocks from her parents in the same residential subdivision. Sophia is surprised to learn that her ad valorem property taxes for the year were raised, while those of her parents were lowered. Select either "Yes" or "No" to identify possible explanation(s) for the difference. a. Sophia made capital improvements to her property. b. Her parents converted their property to income-producing property (e.g., a rental house). c. Lower taxes may apply to a residence owned by older taxpayers (i.e. aged 65 or older).

a. Yes b. No c. Yes

Indicate whether the following are examples of an excise tax (Federal or state). Select "Yes" or "No", whichever is applicable. a. Tax on alcohol b. Tax on air travel c. Sales tax d. Tax on tobacco products e. Hotel occupancy tax

a. Yes b. Yes c. No d. Yes e. Yes

Select either "Yes" or "No" to indicate whether each of the following are penalties imposed upon the preparers of Federal tax returns who violate proscribed acts and procedures. a. Penalty for any willful attempt to understate taxes. b. Penalty for failing to furnish the taxpayer with a copy of the return. c. Penalty for failing to keep copies of returns or maintain a client list. d. Penalty for endorsing a taxpayer's refund check.

a. Yes b. Yes c. Yes d. Yes

With regard to the IRS audit process, complete the following statements: a. The audit is resolved by mail. Referred to as ___ audit, this type of audit ___. b. The audit is conducted at the office of the IRS. Unlike ___ which involves an examination of numerous items reported on the return, ___ is restricted in scope. c. A "no change" RAR results. An RAR (or ___) that results in a "no change" means ___. d. A special agent joins the audit team. A special agent joins the audit team when ___.

a. a correspondence, covers a minor issue b. a field audit, an office audit c. Revenue Agent's Report, that the audit resulted in no additional taxes being due d. taxpayer fraud is suspected

a. Complete the paragraph below that outlines the purpose served by a statute of limitations and how is it relevant in the case of tax controversies. A statute of limitations is a provision in the law that offers a party a defense against a suit brought by another party _____ the expiration of a specified period of time. Therefore, the purpose of a statute of limitations is to ___ stale claims. b. Select either "Yes" or "No" to identify which Federal tax areas that have statutes of limitations. Refund claims by taxpayers Collection of unpaid taxes Additional assessments by the IRS

a. after, preclude parties from prosecuting b. yes, yes, yes

Complete the following statements regarding the Medicare component of FICA. a. Is any dollar limitation imposed on the Medicare tax? Unlike the Social Security portion of FICA, there __ __ dollar limit on the imposition of the Medicare tax. b. Upon whom is the 0.9% additional tax imposed? The 0.9% Medicare addition applies to taxpayers with ___ in excess of $___,___(for single filers) or $___,___(married filing jointly).

a. is no b. earned income such as wages, $200,000, $250,000

In choosing between the standard deduction and itemizing deductions from AGI, what effect, if any, does each of the following variables have? a. The age of the taxpayer(s). The age of the taxpayer ___ if the taxpayer is age 65 or older. b. The health (i.e., physical condition) of the taxpayer(s). If the taxpayer is blind c. Whether the taxpayer(s) rent or own their personal residence. If the taxpayer is still making house payments, the interest deduction may make d. Taxpayer's filing status (e.g., single; married, filing jointly). Because the amount of the ___ varies depending on filing status, this factor is highly relevant to the decision. e. Whether married taxpayers decide to file separate returns. If married persons file separate returns and one spouse itemizes, the other spouse ___. f. The taxpayer's uninsured personal residence that recently was destroyed by a wildfire (the region was declared a disaster area by the Federal government). Because a large casualty loss seems probable, this increases the advantage to be gained ___. g. The number of dependents the taxpayer(s) has. The number of dependents ___.

a. may favor the standard deduction b. an additional standard deduction is available c. itemizing more attractive d. standard deduction e. also must itemize f. itemizing g. has no effect on either of the deductions

Complete the following regarding proportional and progressive taxes. a. A tax is proportional if the rate of tax _______ ________ for any given income level. The tax is progressive if a higher rate of tax applies as the tax base _________. b. Select either "Proportional", "Progressive" or "Neither" to distinguish the items below.

a. remains constant, increases b. Social Security tax - Proportional Federal gift tax -Progressive Federal excise tax on cigarettes - Proportional Federal corporate income tax - Proportional

Answer the following questions regarding estate tax and an inheritance tax. a. Complete the statement below that distinguish between an estate tax and an inheritance tax. If the tax is imposed on the _____ __ ____ ________ __ _____, it is classified as an estate tax. If it taxes the _____ __ _______ ________ ____ _ ________, it is termed an inheritance tax. b. Do some states impose both? Which, if either, does the Federal government impose?

a. right to pass property at death, right to receive property from a decedent b. Yes, both taxes may be imposed, An estate tax

During the year, Tamara had capital transactions resulting in gains (losses) as follows: • Sold stock in ABC Company (acquired two years ago) ($1,500) • Sold collectible coins (held for more than one year) $2,000 • Sold stock in XYZ Company (acquired six months ago) ($4,100) • Sold stock in LMN Company (acquired three years ago) $500 a. As a result of these transactions, Tamara has an overall net b. Is the amount of her overall net gain or loss limited this year? c. If "Yes", what is the limit. If "No", enter"0"

a. short-term capital loss of $3,100 b. Yes c. $3,000

On a Federal income tax return filed five years ago, Andy inadvertently omitted a large amount (more than 25%) of gross income. a. Respond to Andy as he is seeking your advice as to whether the IRS is barred from assessing additional income tax in the event he is audited. Andy, because a ________ statute of limitations applies to a substantial omission of gross income, the IRS ___ assess additional income tax in the event you are audited. b. You were the person who prepared the return. Should you make a disclosure to the IRS regarding the omission on Andy's prior tax return? c. Andy has asked you to prepare his current year's return. Regarding what to do, identify each of the following as either "Yes" or "No". 1. Prepare the return using the information Andy gives you since it is the client's decision to make the disclosure. 2. Withdraw from the engagement if Andy did not make the disclosure and the omission has a carryover effect to the current year. 3. Disclose the omission and then, making the carryover effect correction, file the current year's return.

a. six-year, can b. No, absent Andy's consent, you should not make the disclosure yourself. c. 1 No c. 2 Yes c. 3 No

Regarding the statute of limitations on additional assessments of tax by the IRS, select the applicable date in each of the following situations. Note: Assume a calendar year individual with no fraud or substantial omissions involved. a. The income tax return for 2017 was filed on February 19, 2018. The three-year statute of limitations ____ b. The income tax return for 2017 was filed on June 25, 2018. The statute of limitations will begin to run on ___ c. The income tax return for 2017 was prepared on April 4, 2018, but was never filed. Through some misunderstanding between the preparer and the taxpayer, each expected the other to file the return. The statute of limitations ___ d. The income tax return for 2017 was never filed because the taxpayer thought no additional tax was due. The statute of limitations ___

a. will begin to run on April 17, 2018. b. June 25, 2018 c. will not begin to run d. will not begin to run

Regarding proper ethical guidelines, which (if any) of the following is correct? a. If a client has made a mistake in a prior year's return and refuses to correct it, you should withdraw from the engagement. b. Under no circumstances should a question on a tax return be left unanswered. c. If the exact amount of a deduction is not certain (e.g., around mid-$600s), it should be recorded as an odd amount (i.e., $649) so as to increase the appearance of greater certainty. d. The use of client estimates in preparing a return may be acceptable. e. None of these choices are correct.

d. The use of client estimates in preparing a return may be acceptable.

Complete the following statement regarding the history of income taxes. As a result of the United States' participation in World War II, the income tax was considerably ________, covering a much ______ portion of the population. Hence, it can be described as a ____ tax.

expanded, larger, mass

The wherewithal to pay concept recognizes the ___of taxing a transaction when the taxpayer lacks the means with which to pay the tax. It is particularly suited to situations in which the taxpayer's economic position ___changed significantly as a result of the transaction.

inequity, has not

Complete the paragraph below regarding the statement, "The Sixteenth Amendment to the U.S. Constitution was passed to overturn a Supreme Court decision that had invalidated the Federal income tax." The statement is ___. The Federal income tax on corporations ___a problem as it ___ previously been sanctioned by the Supreme Court. What had been declared unconstitutional was the tax on ___ as it applied to the income from property.

is only partly correct, was not, had, individuals

When enacting tax legislation, a deficit-conscious Congress often has been guided by the concept of ___. The concept means that every new tax law that lowers taxes must include a revenue ___ that makes up for the loss. Revenue ___does not mean that any one taxpayer's tax liability will remain the same.

revenue neutrality, offset, neautrality


Set pelajaran terkait

NUR 210 ATI Custom Practice Quiz No 3

View Set

The Accounting Cycle: Accruals and Deferrals Quiz

View Set

Ch. 12 Gene Expression at a Molecular Level Study Questions and Answers

View Set

Chapter 1: Psychoactive Drugs - Uppers, Downers, All-Arounders

View Set

Practice Test Questions (Cellular Respiration)

View Set